Thoracic Imaging: Case Review Series [3 ed.] 9780323428798

649 336 21MB

English Pages [250] Year 2023

Report DMCA / Copyright

DOWNLOAD FILE

Polecaj historie

Thoracic Imaging: Case Review Series [3 ed.]
 9780323428798

Table of contents :
Front Cover
Thoracic Imaging: Case Review Series, 3ed
Copyright Page
Foreword
Preface
Contents
I. Opening Round
Case 1
Case 2
Case 3
Case 4
Case 5
Case 6
Case 7
Case 8
Case 9
Case 10
Case 11
Case 12
Case 13
Case 14
Case 15
Case 16
Case 17
Case 18
Case 19
Case 20
Case 21
Case 22
Case 23
Case 24
Case 25
Case 26
Case 27
Case 28
Case 29
Case 30
Case 31
Case 32
Case 33
Case 34
II. Fair Game
Case 35
Case 36
Case 37
Case 38
Case 39
Case 40
Case 41
Case 42
Case 43
Case 44
Case 45
Case 46
Case 47
Case 48
Case 49
Case 50
Case 51
Case 52
Case 53
Case 54
Case 55
Case 56
Case 57
Case 58
Case 59
Case 60
Case 61
Case 62
Case 63
Case 64
Case 65
Case 66
Case 67
Case 68
Case 69
Case 70
Case 71
Case 72
Case 73
Case 74
Case 75
Case 76
Case 77
Case 78
Case 79
Case 80
Case 81
Case 82
Case 83
Case 84
Case 85
Case 86
Case 87
Case 88
Case 89
Case 90
Case 91
Case 92
Case 93
Case 94
Case 95
Case 96
Case 97
Case 98
Case 99
Case 100
Case 101
Case 102
III. Challenge
Case 103
Case 104
Case 105
Case 106
Case 107
Case 108
Case 109
Case 110
Case 111
Case 112
Case 113
Case 114
Case 115
Case 116
Case 117
Case 118
Case 119
Case 120
Case 121
Case 122
Case 123
Case 124
Case 125
Case 126
Case 127
Case 128
Case 129
Case 130
Case 131
Case 132
Case 133
Case 134
Case 135
Case 136
Case 137
Case 138
Case 139
Case 140
Case 141
Case 142
Case 143
Case 144
Case 145
Case 146
IV. Answers
Opening Round
Case 1
Squamous Cell Carcinoma of the Lung
Comment
Differential Diagnosis
Lung Cancer
Imaging Findings
REFERENCES
Case 2
Spontaneous Pneumothorax
Comment
Differential Diagnosis
Discussion
Case 3
Pleural Plaques
Comment
Differential Diagnosis
Case 4
Emphysema
Comment
Case 5
Sarcoidosis (Stage I)
Comment
Differential Diagnosis
Discussion
Case 6
Mesothelioma
Comment
Differential Diagnosis
Discussion
Case 7
Junction Lines
Comment
Differential Diagnosis
Discussion
Case 8
Hamartoma
Comment
Differential Diagnosis
Discussion
Case 9
Cystic Fibrosis
Comment
Differential Diagnosis
Discussion
Case 10
Mediastinal Mass (Vascular)
Differential Diagnosis
Discussion
Comment
Differential Diagnosis
Discussion
Comment
Optimal Location of Central Venous Catheter
Azygos Line Placement
Comment
Comment
Differential Diagnosis
Imaging Findings
Comment
Differential Diagnosis
Imaging Findings
Comment
Differential Diagnosis
Imaging Findings
Comment
Differential Diagnosis
Imaging Findings
Comment
Differential Diagnosis
Discussion
Comment
Differential Diagnosis
Discussion
Comment
Comment
Comment
Comment
Comment
Differential Diagnosis
Solitary Pulmonary Nodule
Comment
Differential Diagnosis
Discussion
Comment
Differential Diagnosis
Discussion
Comment
Differential Diagnosis
Discussion
Comment
Differential Diagnosis
Discussion
Comment
Differential Diagnosis
Pericardiophrenic Mass
Imaging Findings
Comment
Differential Diagnosis
Lobar Consolidation
Imaging Findings
Comment
Differential Diagnosis
Discussion
Case 11
Lung Contusion and Laceration
Case 12
Malpositioned Catheter
Case 13
Paraspinal Hematoma
Case 14
Miliary Tuberculosis
Case 15
Neurofibromatosis
Case 16
Left Lower Lobe Atelectasis
Case 17
Pulmonary Hypertension
Case 18
Lung Abscess
Case 19
Pneumomediastinum
Case 20
Subpulmonic Pleural Effusion
Case 21
Pneumothorax on Supine Radiograph
Case 22
Loculated Pleural Fluid in the Major Fissure (Pseudotumor)
Case 23
Esophageal Cancer
Case 24
Benign Calcified Granuloma
Case 25
Radiation Pneumonitis
Case 26
Acute Respiratory Distress Syndrome With Barotrauma
Case 27
Cavity From Postprimary Tuberculosis
Case 28
Mycetoma
Case 29
Pericardial Cyst
Case 30
Right Lower Lobe Pneumonia
Case 31
Empyema
Case 32
Neurogenic Tumor (Schwannoma)
Comment
Differential diagnosis
Discussion
Comment
Differential Diagnosis
Hemothorax
Imaging Findings
Comment
Differential Diagnosis
Neurofibromatosis Type 1 (NF-1)
Imaging Findings
Comment
Differential Diagnosis
Discussion
Comment
Differential Diagnosis
Discussion
Comment
Differential Diagnosis
Discussion
Comment
Differential Diagnosis
Discussion
Comment
Differential Diagnosis
Discussion
Comment
Differential Diagnosis
Discussion
Comment
Differential Diagnosis
Discussion
Comment
Differential Diagnosis
Discussion
Comment
Differential Diagnosis
Discussion
Comment
Differential Diagnosis
Discussion
Comment
Differential Diagnosis
Discussion
Comment
Differential Diagnosis
Discussion
Comment
Differential Diagnosis
Discussion
Comment
Differential Diagnosis
Discussion
Comment
Differential Diagnosis
Discussion
Comment
Differential Diagnosis
Discussion
Comment
Differential Diagnosis
Discussion
Comment
Differential Diagnosis
Discussion
Comment
Differential Diagnosis
Discussion
Comment
Differential Diagnosis
Discussion
Comment
Differential Diagnosis
Discussion
Comment
Differential Diagnosis
Discussion
Comment
Differential Diagnosis
Discussion
Comment
Differential Diagnosis
Discussion
Comment
Comment
Comment
Comment
Comment
Comment
Comment
Comment
Comment
Comment
Comment
Comment
Comment
Comment
Comment
Comment
Comment
Comment
Comment
Comment
Comment
Comment
Comment
Comment
Differential Diagnosis
Discussion
Comment
Differential Diagnosis
Discussion
Comment
Differential Diagnosis
Discussion
Comment
Differential Diagnosis
Discussion
Comment
Differential Diagnosis
Discussion
Comment
Differential Diagnosis
Discussion
Comment
Differential Diagnosis
Discussion
Comment
Differential Diagnosis
Discussion
Comment
Differential Diagnosis
Discussion
Comment
Differential Diagnosis
Discussion
Differential Diagnosis
Discussion
Comment
Differential Diagnosis
Discussion
Comment
Differential Diagnosis
Discussion
Comment
Differential Diagnosis
Discussion
Comment
Comment
Differential Diagnosis
Discussion
Comment
Differential Diagnosis
Discussion
Comment
Differential Diagnosis
Discussion
Comment
Differential Diagnosis
Discussion
Comment
Differential Diagnosis
Discussion
Comment
Differential Diagnosis
Comment
Differential Diagnosis
Discussion
Comment
Differential Diagnosis
Discussion
Comment
Differential Diagnosis
Discussion
Comment
Differential Diagnosis
Discussion
Comment
Differential Diagnosis
Discussion
Comment
Differential Diagnosis
Discussion
Comment
Differential Diagnosis
Discussion
Comment
Differential Diagnosis
Discussion
Comment
Differential Diagnosis
Discussion
Comment
Differential Diagnosis
Discussion
Comment
Differential Diagnosis
Discussion
Comment
Differential Diagnosis
Discussion
Comment
Discussion
Comment
Differential Diagnosis
Discussion
Comment
Differential Diagnosis
Discussion
Comment
Differential Diagnosis
Discussion
Comment
Differential Diagnosis
Discussion
Comment
Differential Diagnosis
Discussion
Comment
Comment
Comment
Comment
Comment
Comment
Comment
Comment
Comment
Comment
Comment
Comment
Comment
Comment
Differential Diagnosis
Comment
Comment
Differential Diagnosis
Discussion
Comment
Differential Diagnosis
Discussion
Comment
Differential Diagnosis
Discussion
Comment
Differential Diagnosis
Comment
Comment
Differential Diagnosis
Discussion
Comment
Differential Diagnosis
Discussion
Comment
Differential Diagnosis
Comment
Comment
Differential Diagnosis
Comment
Comment
Differential Diagnosis
Discussion
Comment
Differential Diagnosis
Comment
Comment
Differential Diagnosis
Discussion
Comment
Differential Diagnosis
Discussion
Case 33
Hemothorax
Case 34
Rib Notching
Fair Game
Case 35
Left Upper Lobe Atelectasis (Lung Cancer)
Case 36
Interstitial Pulmonary Edema
Case 37
Thyroid Goiter
Case 38
Community Acquired Pneumonia
Case 39
Pulmonary Langerhans Cell Histiocytosis
Case 40
Lymphangitic Carcinomatosis
Case 41
Rounded Atelectasis
Case 42
Sarcoidosis (Stage IV)
Case 43
Emphysema (Centrilobular)
Case 44
Anterior Mediastinal Mass
Case 45
Subcarinal Lymph Node Enlargement Secondary to Metastatic Disease
Case 46
Silicosis
Case 47
Connective Tissue Disease-Associated Interstitial Lung Disease (Systemic Sclerosis [Scleroderma] With Nonspecific Interstit ...
Case 48
Ranke Complex
Case 49
Bronchiectasis
Case 50
Arteriovenous Malformation
Case 51
Aortic Aneurysm
Case 52
Pulmonary Infarct
Case 53
Lymphoma
Case 54
Traumatic Aortic Injury (Transection)
Case 55
Lipoid Pneumonia
Case 56
Kartagener Syndrome (Primary Ciliary Dyskinesia)
Case 57
Invasive Aspergillus
Case 58
Granulomatosis With Polyangiitis
Case 59
Lymphangioleiomyomatosis
Case 60
Tuberculosis
Case 61
Pneumocystis jirovecii Pneumonia
Case 62
Metastatic Osteosarcoma
Case 63
Septic Infarcts
Case 64
Bronchopleural Fistula
Case 65
Superior Sulcus Tumor
Case 66
Tuberculosis (Post-Primary Type)
Case 67
Lymphadenopathy From Lung Cancer
Case 68
Subsolid Nodule (Adenocarcinoma)
Case 69
Azygos Continuation of the Inferior Vena Cava
Case 70
Right Upper Lobe Atelectasis (Lung Cancer)
Case 71
Traumatic Rupture of the Left Hemidiaphragm
Case 72
Persistent Left Superior Vena Cava
Case 73
Aspiration
Case 74
Nonspecific Interstitial Pneumonitis (NSIP)
Case 75
Ascending Aortic Aneurysm
Case 76
Carcinoid Tumor
Case 77
Boerhaave Syndrome
Case 78
Bronchiectasis
Case 79
Giant Bulla
Case 80
RML and RLL atelectasis
Case 81
Lipoma
Case 82
Apical Cap Secondary to Extrapleural Hematoma
Case 83
Bleomycin Lung Toxicity
Case 84
Primary Lung Adenocarcinoma With N2 Nodal Disease
Case 85
Superior Vena Cava Syndrome from Lung Cancer
Case 86
Diffuse Alveolar Hemorrhage From Vasculitis
Case 87
Usual Interstitial Pneumonia (UIP) Caused by Idiopathic Pulmonary Fibrosis (IPF)
Case 88
Intramuscular Hematoma From Supratherapeutic Warfarin Therapy
Case 89
Scimitar Syndrome
Case 90
Pericardial Effusion
Case 91
Idiopathic Bronchiectasis
Case 92
Achalasia
Case 93
Cavity Caused by Coccidioidomycosis
Case 94
Hydrostatic Pulmonary Edema
Case 95
Internal Mammary Lymph Node Enlargement
Case 96
CT-Guided Transthoracic Needle Biopsy Procedure
Case 97
Bronchial Atresia
Case 98
Hypersensitivity Pneumonitis
Case 99
Solitary Pulmonary Nodule: Lung Cancer Screening
Case 100
Post-Intubation Tracheal Stenosis
Case 101
Thymic Hyperplasia
Case 102
Part-Solid Nodule (Lepidic Predominant Lung Adenocarcinoma)
Challenge
Case 103
Pulmonary Alveolar Proteinosis
Case 104
Allergic Bronchopulmonary Aspergillosis (ABPA)
Case 105
Mucoid Impaction (Endobronchial Hamartoma)
Case 106
Solitary Fibrous Tumor of the Pleura
Case 107
Complete Lung Collapse Secondary to an Endobronchial Lesion (Lung Cancer)
Case 108
Mycobacterium avium complex (MAC) infection
Case 109
Sarcoidosis (Perilymphatic Nodules)
Case 110
Small Airways Disease (Infectious Bronchiolitis)
Case 111
Mosaic Attenuation (Small Airway Disease)
Case 112
Chronic Beryllium Disease
Case 113
Chronic Eosinophilic Pneumonia
Case 114
Tracheal Diverticulum
Case 115
Sternal Dehiscence After Median Sternotomy
Case 116
Mediastinitis
Case 117
Cryptogenic Organizing Pneumonia
Case 118
Tracheobronchial Amyloidosis
Case 119
Constrictive Bronchiolitis (Swyer-James-MacLeod Syndrome)
Case 120
Metastasizing Leiomyomatosis
Case 121
Asbestosis
Case 122
Mounier-Kuhn Syndrome
Case 123
Tracheomalacia
Case 124
Adenoid Cystic Carcinoma
Case 125
Nocardia
Case 126
Chronic Thromboembolic Disease
Case 127
Metastatic Thyroid Cancer
Case 128
Castleman Disease (Benign Lymph Node Hyperplasia)
Case 129
Aortic dissection
Case 130
Relapsing Polychondritis
Case 131
Swyer-James-Macleod Syndrome
Case 132
Accessory Cardiac Bronchus
Case 133
Intralobar Sequestration
Case 134
Acute Pulmonary Emboli With Right Heart Strain
Case 135
Panlobular Emphysema (Alpha-1 Antitrypsin Deficiency)
Case 136
Thymic Cyst
Case 137
Tracheal Bronchus
Case 138
Amiodarone Toxicity
Case 139
Saphenous Vein Graft Aneurysm Following CABG Surgery
Case 140
Infectious Bronchiolitis
Case 141
Cytomegalovirus Pneumonia in a Lung Transplant Recipient
Case 142
Pulmonary Veno-Occlusive Disease
Case 143
Tracheobronchial Papillomatosis
Case 144
Echinococcal Cyst
Case 145
Congenital Pulmonary Airway Malformation (CPAM)
Case 146
Lymphoid Interstitial Pneumonia (LIP)
V. Supplementary Figures
Index

Citation preview

i w

HHMHIIE

MBIMII

>

fii

w in Sil i

ora i

/ *

«J

Q CASE REVIEW SERIES ELSEVIER

r

3rd EDITION

Any screen. Any time. Anywhere. Activate the eBook version of this title at no additional charge.

Elsevier eBooks+ gives you the power to browse, search, and customize your content, make notes and highlights, and have content read aloud.

Unlock your eBook today. 1. Visit http://ebooks.health.elsevier.com/ 2. Log in or Sign up 3. Scratch box below to reveal your code 4. Type your access code into the “Redeem Access Code” box 5. Click “Redeem”

It’s that easy!

Place Peel Off Sticker Here

For technical assistance: email [email protected] call 1-800-545-2522 (inside the US) call +44 1 865 844 640 (outside the US) Use of the current edition of the electronic version of this book (eBook) is subject to the terms of the nontransferable, limited license granted on http://ebooks.health.elsevier.com/. Access to the eBook is limited to the first individual who redeems the PIN, located on the inside cover of this book, at http://ebooks.health.elsevier.com/ and may not be transferred to another party by resale, lending, or other means. 2022v1.0

Thoracic Imaging Case Review Series

Thoracic Imaging Case Review Series THIRD EDITION

JUSTIN T. STOWELL, MD Assistant Professor of Radiology Consultant Division of Cardiothoracic Imaging, Department of Radiology, Mayo Clinic Jacksonville, Florida

JONATHAN H. CHUNG, MD Professor of Radiology Chief Quality Ofcer, Department of Radiology Section Chief, Thoracic Radiology The University of Chicago Medicine Chicago, Illinois

JEFFREY P. KANNE, MD, FACR, FCCP Professor of Radiology Chief of Thoracic Imaging University of Wisconsin School of Medicine and Public Health Madison, Wisconsin

THERESA C. MCLOUD, MD Professor of Radiology Harvard Medical School Senior Advisor for Faculty Affairs and Thoracic Radiologist Massachusetts General Hospital Boston, Massachusetts

GERALD F. ABBOTT, MD, FACR Associate Professor Harvard Medical School Division of Thoracic Imaging and Intervention Massachusetts General Hospital Boston, Massachusetts

Elsevier 1600 John F. Kennedy Blvd. Ste 1800 Philadelphia, PA 19103-2899 THORACIC IMAGING: CASE REVIEW SERIES, THIRD EDITION

ISBN: 978-0-323-42879-8

Copyright © 2024 by Elsevier Inc. All rights reserved No part of this publication may be reproduced or transmitted in any form or by any means, electronic or mechanical, including photocopying, recording, or any information storage and retrieval system, without permission in writing from the publisher. Details on how to seek permission, further information about the Publisher’s permissions policies, and our arrangements with organizations such as the Copyright Clearance Center and the Copyright Licensing Agency, can be found at our website: www.elsevier.com/permissions. This book and the individual contributions contained in it are protected under copyright by the Publisher (other than as may be noted herein).

Notice Practitioners and researchers must always rely on their own experience and knowledge in evaluating and using any information, methods, compounds, or experiments described herein.Because of rapid advances in the medical sciences in particular, independent verication of diagnoses and drug dosages should be made. To the fullest extent of the law, no responsibility is assumed by Elsevier, authors, editors, or contributors for any injury and/or damage to persons or property as a matter of products liability, negligence, or otherwise, or from any use or operation of any methods, products, instructions, or ideas contained in the material herein.

Previous editions copyrighted 2011, 2001

Senior Content Development Manager: Somodatta Roy Choudhury Senior Content Strategist: Melanie Tucker Senior Content Development Specialist: Priyadarshini Pandey Publishing Services Manager: Shereen Jameel Project Manager: Vishnu T. Jiji Senior Designer: Amy L. Buxton Printed in India Last digit is the print number:

9

8

7

6

5

4

3

2

1

This book is dedicated to radiology residents and other trainees and practitioners in related elds with the hope that the provided cases and discussions will expose them to the rich variety of imaging ndings in thoracic imaging and their relevant clinical features. Justin T. Stowell, MD Jonathan H. Chung, MD Jeffrey P. Kanne, MD, FACR, FCCP Theresa C. McLoud, MD Gerald F. Abbott, MD, FACR

Foreword What is more fundamental to radiology than thoracic imaging? Radiology imaging no doubt starts here for most residents, and yet the depth of investigation into cardiopulmonary pathology stretches into the nal year of residency (or fellowship) training. The lungs and heart yield ever greater challenges to the reader, even as our imaging modalities get more and more sophisticated. So it is that Thoracic Imaging: Case Review Series, Third Edition, serves the needs of all resident levels, fellows, and practicing radiologists. This is a book for the ages. There are new COVID-like pearls and all kinds of new autoimmune and infectious agents that are explored in the latest edition. Congratulations to Drs. Gerald F. Abbott, Jonathan H. Chung, Jeffrey P. Kanne, Theresa C. McLoud (goddess of thoracic radiology), and Justin T. Stowell for their latest

contributions to the ever-vibrant Case Review Series. These books are perfect for the current generation of learners who want the short, pertinent hits that they can enjoy during breaks from clinical cases or video games or TikTok submissions. The series keeps your attention by stimulating you, like with ongoing longitudinal assessments, for a few images, quick questions, short blurb, and relevant reference. Boom. Move on to next case. Thank you to the authors for their hard work. To our readers, I hope you enjoy Thoracic Imaging: Case Review Series, Third Edition, whether you are currently reading hard copy or Soft Copy … or both. Live, love, LEARN, and leave a legacy. David M. Yousem, MD, MBA

vii

Preface The publication of the third edition of Thoracic Imaging: The Requisites in 2019 by our close friend and colleague, Jo-Anne O. Shepard, MD, was a major undertaking and involved the collaboration of numerous other experts in thoracic imaging. The many co-editors and contributors to that work all share close ties to the Massachusetts General Hospital (MGH) Department of Thoracic Imaging and Intervention (Boston, MA). We are pleased to present the long-awaited accompanying Thoracic Imaging: Case Review Series, Third Edition. Similar to the third edition of Thoracic Imaging: The Requisites, this Case Review book has been extensively edited to include the latest updates in the expanding subspecialty of thoracic imaging. This edition has been completely rewritten and supplemented with hundreds of new images. Firstly, it maintains the two primary goals of the previous editions. These include a case format presentation to illustrate and review the imaging features of disorders that span the spectrum of thoracic diseases. Secondly, it is our goal to help the reader develop a sound framework with which to approach imaging interpretation in thoracic radiology. The evolution of complex thoracic diseases and treatments, including lung cancer, interstitial lung disease, and heart and lung transplant, must be emphasized in both residency and fellowship curricula. Lung cancer screening continues to expand to more eligible patients with risk factors for lung cancer. The critical role of radiology has been emphasized, and the art and

science of radiologic staging is illustrated in several cases. Pulmonary nodule management strategies and interstitial lung disease classications have been updated by multidisciplinary societies and are included in this edition. This update is composed of a diverse group of more than 140 cases using multimodality imaging from radiography to magnetic resonance imaging. As is standard for the Case Review Series, the book is organized by the level of difculty: Opening Round, Fair Game, and Challenge cases. Each case is accompanied by a series of multiple-choice questions with answers on the opposite page. Answers are followed by a brief discussion to highlight imaging features, differential diagnostic considerations, and key points from each case. Additional references are provided for those who desire more information about each topic, and cross-references to Thoracic Imaging: The Requisites are also provided. We hope that this latest edition of Thoracic Imaging will continue to serve as a valuable tool for lifelong learning of thoracic imaging. Justin T. Stowell, MD Jonathan H. Chung, MD Jeffrey P. Kanne, MD, FACR, FCCP Theresa C. McLoud, MD Gerald F. Abbott, MD, FACR

ix

Contents Section I

Opening Round, 1

Case 1

Lung Cancer, 1

Case 2

Spontaneous Pneumothorax, 2

Case 3

Pleural Plaques, 3

Case 4

Emphysema: Chest Radiography, 4

Case 5

Case 33

Hemothorax, 33

Case 34

Rib Notching, 34

Section II

Fair Game, 35

Case 35

Left Upper Lobe Atelectasis (Lung Cancer), 35

Sarcoidosis, 5

Case 36

Interstitial Edema, 36

Case 6

Mesothelioma, 6

Case 37

Thyroid Goiter, 37

Case 7

Junction Lines, 7

Case 38

Community-Acquired Bacterial Pneumonia, 38

Case 8

Hamartoma, 8

Case 39

Pulmonary Langerhans Cell Histiocytosis, 39

Case 9

Cystic Fibrosis, 9

Case 40

Lymphangitic Carcinomatosis, 40

Case 10

Mediastinal Mass (Aneurysm or Other Vascular Lesion), 10

Case 41

Rounded Atelectasis, 41

Case 11

Pulmonary Contusion and Laceration, 11

Case 42

Sarcoidosis (Stage IV), 42

Case 12

Malpositioned Catheter, 12

Case 43

Emphysema on CT (Centrilobular), 43

Case 13

Abnormal Paraspinal Line on Radiography, 13

Case 44

Anterior Mediastinal Mass (Thymoma), 44

Case 14

Miliary Tuberculosis, 14

Case 45

Subcarinal Lymph Node Enlargement Secondary to Metastatic Disease, 45

Case 15

Neurobromas, 15

Case 46

Silicosis, 46

Case 16

Left Lower Lobe Atelectasis, 16

Case 47

Case 17

Pulmonary Hypertension, 17

Progressive Systemic Sclerosis (Scleroderma), 47

Case 18

Lung Abscess, 18

Case 48

Ranke Complex, 48

Case 19

Pneumomediastinum, 19

Case 49

Bronchiectasis, 49

Case 20

Subpulmonic Pleural Effusion, 20

Case 50

Arteriovenous Malformation, 50

Case 21

Pneumothorax on Supine Radiograph, 21

Case 51

Aortic Aneurysm, 51

Case 22

Loculated Pleural Fluid in the Major Fissure, 22

Case 52

Pulmonary Infarction, 52

Case 53

Lymphoma, 53

Case 23

Esophageal Cancer, 23

Case 54

Traumatic Aortic Injury, 54

Case 24

Calcied Granuloma, 24

Case 55

Lipoid Pneumonia, 55

Case 25

Radiation Pneumonitis, 25

Case 56

Kartagener Syndrome, 56

Case 26

Acute Respiratory Distress Syndrome (ARDS), 26

Case 57

Invasive Aspergillosis, 57

Case 27

Cavitary Tuberculosis, 27

Case 58

Granulomatosis With Polyangiitis, 58

Case 28

Mycetoma, 28

Case 59

Lymphangioleiomyomatosis, 59

Case 29

Pericardial Cyst, 29

Case 60

Tuberculosis (Lymphadenopathy), 60

Case 30

Pneumonia, 30

Case 61

Pneumocystis jirovecii Pneumonia, 61

Case 31

Empyema, 31

Case 62

Ossied Lymph Nodes Secondary toMetastatic Osteosarcoma, 62

Case 32

Neurogenic Tumor (Schwannoma), 32

Case 63

Septic Infarcts, 63 xi

xii

Contents

Case 64

Bronchopleural Fistula, 64

Case 99

Case 65

Superior Sulcus Tumor, 65

Case 66

Post-Primary “Reactivation” Tuberculosis (TB), 66

Case 67

Mediastinal and Hilar Lymphadenopathy (Lung Cancer), 67

Case 68

Subsolid Nodule (Adenocarcinoma), 68

Case 69

Azygos Continuation of the Inferior Vena Cava, 69

Case 70

Right Upper Lobe Atelectasis, 71

Case 104 Allergic Bronchopulmonary Aspergillosis, 105

Case 71

Traumatic Rupture of the Left Hemidiaphragm, 72

Case 106 Solitary Fibrous Tumor of the Pleura, 107

Case 72

Persistent Left Superior Vena Cava, 73

Case 107 Complete Lung Collapse (Lung Cancer), 108

Case 73

Aspiration, 74

Case 74

Nonspecic Interstitial Pneumonia(NSIP), 75

Case 108 Mycobacterium Avium Complex (MAC) Infection, 109

Case 75

Ascending Aortic Aneurysm, 76

Case 76

Carcinoid, 77

Case 77

Boerhaave Syndrome, 78

Case 78

Bronchiectasis, 79

Case 111 Mosaic Attenuation (Small Airways Disease), 112

Case 79

Bulla (Giant), 80

Case 112 Chronic Beryllium Disease, 113

Case 80

Combined RML and RLL Atelectasis (Endobronchial Metastases), 81

Case 113 Chronic Eosinophilic Pneumonia, 114

Case 81

Lipoma, 82

Case 82

Apical Cap, 83

Case 115 Sternal Dehiscence Following MedianSternotomy, 116

Case 83

Bleomycin Drug Toxicity, 84

Case 116 Mediastinitis, 117

Case 84

Lung Cancer With N2 Nodal Disease, 85

Case 117 Organizing Pneumonia, 118

Case 85

Superior Vena Cava (SVC) Syndrome, 86

Case 118 Tracheobronchial Amyloidosis, 119

Case 86

Pulmonary Hemorrhage, 87

Case 119 Constrictive Bronchiolitis, 120

Case 87

Idiopathic Pulmonary Fibrosis (IPF), 88

Case 120 Metastasizing Leiomyoma, 121

Case 88

Chest Wall Mass: Hematoma, 89

Case 121 Asbestosis, 122

Case 89

Scimitar Sign (Hypogenetic Lung Syndrome), 90

Case 122 Mounier-Kuhn Syndrome, 123

Case 90

Pericardial Effusion, 91

Case 91

Bronchiectasis, 92

Case 92

Achalasia, 93

Case 93

Coccidioidomycosis, 94

Case 94

Hydrostatic Pulmonary Edema, 95

Case 95

Internal Mammary Lymph Node Enlargement, 96

Case 96

Computed Tomography (CT)-Guided Transthoracic Needle Biopsy (TTNB), 97

Case 97

Bronchial Atresia, 98

Case 130 Relapsing Polychondritis, 131

Case 98

Hypersensitivity Pneumonitis, 99

Case 131 Swyer-James-Macleod Syndrome, 132

Solitary Pulmonary Nodule: Lung Cancer Screening, 100

Case 100 Tracheal Stenosis, 101 Case 101 Thymic Hyperplasia, 102 Case 102 Part Solid Nodule, 103

Section III Challenge, 104 Case 103 Pulmonary Alveolar Proteinosis, 104

Case 105 Mucoid Impaction (Obstructing Tumor), 106

Case 109 Sarcoidosis (Perilymphatic Nodules), 110 Case 110 Small Airways Disease (InfectiousBronchiolitis), 111

Case 114 Tracheal Diverticulum, 115

Case 123 Tracheomalacia, 124 Case 124 Adenoid Cystic Carcinoma of the Trachea, 125 Case 125 Nocardia Infection (Heart TransplantPatient), 126 Case 126 Chronic Pulmonary Embolism, 127 Case 127 Metastatic Thyroid Carcinoma, 128 Case 128 Castleman Disease (Benign Lymph Node Hyperplasia), 129 Case 129 Aortic Dissection, 130

Contents

xiii

Case 132 Accessory Cardiac Bronchus, 133

Case 142 Pulmonary Venoocclusive Disease, 143

Case 133 Intralobar Sequestration, 134

Case 143 Tracheobronchial Papillomatosis, 144

Case 134 Acute Pulmonary Embolism, 135

Case 144 Echinococcal Cysts, 145

Case 135 Panlobular Emphysema (Alpha1-Antitrypsin Deciency), 136

Case 145 Congenital Pulmonary Airway Malformation (CPAM), 146

Case 136 Thymic Cyst, 137

Case 146 Lymphoid Interstitial Pneumonia (LIP), 147

Case 137 Tracheal Bronchus, 138 Case 138 Amiodarone Drug Toxicity, 139 Case 139 Saphenous Vein Graft Aneurysm Following Coronary Artery Bypass Grafting (CABG) Surgery, 140

Section IV Answers, 148 Section V

Supplementary Figures, e1

Index, 232

Case 140 Infectious Bronchiolitis, 141 Case 141 Viral Pneumonia (Organ Transplant Patient), 142

Supplemental gures can be accessed at Elsevier eBooks+ (eBooks.Health.Elsevier.com)

SECTION I

Opening Round

Case 1 History: Hemoptysis. 1. Which of the following should be included in the differential diagnosis? (Figs. 1.1, 1.2, 1.3 and 1.4) (Choose all that apply.) A. Lung carcinoma B. Infection C. Silicosis D. Pulmonary sequestration 2. What is the most common histologic type of lung carcinoma? A. Adenocarcinoma B. Squamous cell carcinoma C. Small cell carcinoma D. Carcinoid 3. What is the greatest risk factor for developing lung carcinoma? A. Coal B. Asbestos

C. Smoking D. Obesity 4. Which factor best predicts 5-year survival for patients with lung carcinoma? A. Smoking history B. Histologic type C. Grade of neoplasm D. Stage at diagnosis

Fig. 1.1

Fig. 1.2

Fig. 1.3

Fig. 1.4

1

Case 2 History: A 28-year old man has acute chest pain while playing basketball. 1. What is the most likely diagnosis (Figs. 2.1 and 2.2)? A. Traumatic pneumatocele B. Pneumomediastinum C. Spontaneous tension pneumothorax D. Swyer-James-McLeod syndrome 2. What is the likely mechanism involved in producing a traumatic pneumatocele? A. Penetrating trauma to the lung B. Compression-decompression trauma causing rupture of small airways C. Extension of pneumomediastinum into lung parenchyma D. Expansion of a preexisting bulla 3. What is the causative pathologic process in Swyer-JamesMcLeod syndrome? A. Blunt force trauma B. Postinfectious bronchiolitis C. Congenital bronchial atresia D. Incomplete interlobar ssures 4. Which radiographic nding occurs in tension pneumothorax? (Choose all that apply.) A. Contralateral shift of the mediastinum B. Downward displacement of the ipsilateral hemidiaphragm C. Ipsilateral widening of intercostal space (between ribs) D. All of the above

Fig. 2.1

2

Fig. 2.2

Case 3 History: Asymptomatic 63-year-old man. 1. Based on the radiographic and computed tomography CT ndings (Fig.3.1), what is the most likely diagnosis? A. Fibrothorax B. Previous pleurodesis C. Asbestos-related pleural plaques D. Metastases 2. What percentage of patients diagnosed with the pleural malignancy caused by mesothelioma will have concomitant pleural plaques? A. 0% B. 5% C. 25% D. 100%

3. What is the characteristic latency period between a person’s occupational exposure to asbestos and the development of mesothelioma? A. 1–2 years B. 5–10 years C. 10–15 years D. 30–40 years 4. Which of the following occupations has been associated with asbestos exposure in past decades? (Choose all that apply.) A. Construction worker B. Fireghter C. Shipyard worker D. Textile mill worker E. All of the above

Fig. 3.1

3

Case 4 History: 60-year-old man with dyspnea. 1. Which of the following conditions is associated with hyperinated lungs? (Choose all that apply.) A. Centrilobular emphysema B. Fibrotic hypersensitivity pneumonitis C. Lymphangioleiomyomatosis D. Asthma 2. What is the characteristic etiology of Swyer-James-McLeod syndrome? A. Follicular bronchiolitis B. Postinfectious bronchiolitis C. Broncholithiasis with air trapping D. Bronchial atresia

3. Which of the following is not always a smoking-related disease? A. Centrilobular emphysema B. Paraseptal emphysema C. Panlobular emphysema D. Desquamative interstitial pneumonia (DIP) 4. Which of the following are characteristic radiographic ndings of advanced centrilobular emphysema in a patient who smokes (Fig.4.1)? (Choose all that apply.) A. Hyperination B. Hyperlucency of upper lung zones C. Rapid tapering and attenuation of pulmonary vessels in affected lung D. Flattened hemidiaphragms

Fig. 4.1

4

Case 5 History: Asymptomatic 29-year-old woman. 1. Based on the clinical setting and imaging ndings (Fig.5.1), what is the most likely diagnosis? A. Castleman disease B. Lymphoma C. Sarcoidosis D. Silicosis 2. Which of the following malignancies may manifest with metastatic mediastinal and hilar lymphadenopathy? (Choose all that apply.) A. Head and neck malignancies B. Melanoma C. Breast cancer

D. Genitourinary malignancies E. All of the above 3. Which of the following endemic fungal infections is most associated with lymphadenopathy? A. Blastomycosis B. Coccidioidomycosis C. Histoplasmosis 4. Which of the following may manifest with asymmetric bilateral hilar lymphadenopathy? (Choose all that apply.) A. Lymphoma B. Sarcoidosis C. Metastatic disease D. All of the above

Fig. 5.1

5

Case 6 History: 66-year-old man with weight loss. 1. Which of the following imaging features are characteristic of pleural malignancy (Figs.6.1A and 6.1B)? (Choose all that apply.) A. >1 cm in thickness B. Nodularity C. Circumferential growth pattern within the involved hemithorax D. Involvement of the mediastinal pleural 2. Which of the following is most common? A. Mesothelioma B. Pleural lymphoma C. Pleural metastasis D. Solitary brous tumor of the pleura

Fig. 6.1A

6

3. What percentage of patients with pleural mesothelioma also have pleural plaques as a manifestation of prior asbestos exposure? A. 10% B. 25% C. 50% D. 100% 4. Which of the following is the most common histologic subtype of mesothelioma? A. Epithelioid B. Sarcomatoid C. Adenomatoid D. Mixed (biphasic)

Fig. 6.1B

Case 7 History: 28-year-old man with cough. 1. Based on the structures indicated (arrow and arrowhead Fig.7.1), what is the most likely diagnosis? A. Pneumomediastinum B. Pneumothorax C. Esophageal dilatation D. Normal study 2. Which of the following etiologies are associated with pneumomediastinum? (Choose all that apply.) A. Inhalational drug use B. Weightlifting C. Obstructive lung disease D. Pulmonary brosis

3. In Dr. Benjamin Felson’s original system for evaluating mediastinal abnormalities on lateral chest radiography, which of the following courses of a boundary line separates the anterior mediastinum from the middle mediastinum? A. Anterior to the trachea and anterior to the heart B. Anterior to the trachea and posterior to the heart C. Posterior to the trachea and anterior to the heart D. Posterior to the trachea and posterior to the heart 4. Which of the following radiographic ndings typically occur in total atelectasis of the right or left lung? A. Contralateral displacement of the anterior junction line B. Ipsilateral displacement of the anterior junction line C. Downward displacement of the ipsilateral hemidiaphragm D. Downward displacement of the contralateral hemidiaphragm

Fig. 7.1

7

Case 8 History: 70-year-old man with 30-pack year history of cigarette smoking. Finding detected on initial lung cancer screening computed tomography (CT) study (Fig.8.1). 1. What is the most likely diagnosis? A. Lepidic adenocarcinoma B. Carcinoid tumor C. Hamartoma D. Lipoid pneumonia 2. Which of the following may show uorodeoxyglucose (FDG)-avidity on positron emission tomography (PET) imaging? (Choose all that apply.) A. Lepidic adenocarcinoma B. Carcinoid tumor C. Lipoid pneumonia

D. Hamartoma E. All of the above 3. What is the range of fat attenuation on the Hounseld scale of CT numbers? A. -300 to -200 B. -200 to -100 C. -100 to -50 D. -40 to 0 4. What is the percentage of hamartomas detected by thinsection CT exhibit fat attenuation? A. 9% B. 29% C. 59% D. 79%

Fig. 8.1

8

Case 9 History: 22-year-old man with chronic cough. 1. Based on the radiographic ndings (Fig.9.1), what is the most likely diagnosis? A. Lymphangioleiomyomatosis B. Cystic brosis C. Centrilobular emphysema D. Pulmonary Langerhans cell histiocytosis 2. Which of the following computed tomography (CT) techniques is most helpful in the detection of bronchiectasis? A. Maximum-intensity projection images (MIPs) B. Minimum-intensity projection images (MinIPs) C. Multiplanar reformations D. Expiratory images

3. Which of the following conditions are associated with bronchial artery dilatation? (Choose all that apply.) A. Bronchiectasis B. Cavitary infection C. Cavitary sarcoidosis D. Lung cancer E. All of the above 4. Which of the following abnormalities are associated with cystic brosis? (Choose all that apply.) A. Mediastinal/hilar lymphadenopathy B. Pansinusitis C. Biliary cirrhosis D. Infertility E. All of the above

Fig. 9.1

9

Case 10 History: 66-year-old man with substernal chest pain. 1. Dr. Benjamin Felson introduced the concept of mediastinal compartments by using normal anatomic landmarks on the lateral chest radiograph. What is the course of the line Felson used to separate the anterior mediastinum from the middle mediastinum? A. Anterior to the trachea and anterior to the heart B. Anterior to the trachea and posterior to the heart C. Posterior to the trachea and anterior to the heart D. Posterior to the trachea and posterior to the heart 2. Which of the following diagnoses is most consistent with the radiographic nding of diffuse mediastinal widening (Fig.10.1)? A. Thymoma B. Bronchogenic cyst

C. Metastatic lymphadenopathy D. Esophageal varices 3. What percentage of mediastinal abnormalities are vascular lesions? A. 10% B. 20% C. 30% D. 40% 4. Which of the following are most characteristic of patient age encountered in cases of thymoma and teratoma? A. Over and under the age of 20 years, respectively B. Under and over the age of 40 years, respectively C. Over and under the age of 40 years, respectively D. Under and over the age of 20 years, respectively

Fig. 10.1

10

Case 11 History: 46-year-old man was struck by a motorcycle. 1. What is the most likely cause of the ground-glass opacity and consolidation in the left lung (Figs.11.1A and 11.1B)? A. Aspiration B. Atelectasis C. Pulmonary contusion D. Lung cancer 2. What is the most likely cause of the focal lucency within the parenchymal consolidation? A. Bronchiectasis B. Bullous emphysema C. Post-traumatic pneumatocele D. Cystic neoplasm

Fig. 11.1A

3. Pulmonary contusion should begin to resolve within how much time from injury? A. 4 hours B. 12 hours C. 24 hours D. 48 hours 4. Which of the following may manifest as cysts on computed tomography (CT)? A. Desquamative interstitial pneumonia (DIP) B. Lymphocytic interstitial pneumonia (LIP) C. Pulmonary Langerhans cell histiocytosis (PLCH) D. All of the above

Fig. 11.1B

11

Case 12 History: Recent central line placement. 1. Where does the central line terminate (Figs.12.1 and 12.2)? A. Superior vena cava (SVC) B. Azygos vein C. Great cardiac vein D. Brachiocephalic vein 2. What is the ideal location for the termination of a central venous line in most cases? A. Subclavian vein B. Azygos vein C. Brachiocephalic vein D. Cavoatrial junction 3. What is the most likely complication of a central venous line placed in the right atrium? A. Cardiac arrhythmia B. Cardiac thrombus

Fig. 12.1

12

C. Cardiac rupture D. Cardiac tamponade 4. A subclavian central catheter is placed for administration of chemotherapy for lung cancer. Pulsatile, bright, red blood is noted from the catheter immediately after placement. Of the listed choices, what is the next best step in management? A. Use the catheter for drug administration B. Remove the line immediately C. Contact vascular surgery D. Vigorously ush the catheter See Supplemental Figures section for additional gures for this case. [E-book/Inkling: Link to supplemental gures, Case 12]

Fig. 12.2

Case 13 History: History withheld. 1. Which of the following could cause this imaging nding (Figs. 13.1, 13.2 and 13.3)? A. Achalasia B. Spinal fracture C. Metastatic disease D. Aortic aneurysm 2. What classic sign of posterior mediastinal lesions is present in this case? A. Cervicothoracic sign B. Thoracoabdominal sign C. Cervicoabdominal sign D. Cervicothoracoabdominal sign 3. What is the most sensitive modality to detect acute spinal fractures? A. Radiography B. Fluoroscopy C. Computed tomography (CT) D. Magnetic resonance imaging (MRI) 4. An increase in tube potential does what to radiation dose? A. Increase on radiography and CT B. Decrease on radiography and CT C. Increase on radiography but decrease on CT D. Decrease on radiography but increase on CT

Fig. 13.2

Fig. 13.1

Fig. 13.3

13

Case 14 History: 53-year-old man with cough and shortness of breath. 1. Which of the following should be included in the differential diagnosis for this patient (Figs.14.1 and 14.2)? (Choose all that apply.) A. Emphysema B. Pulmonary edema C. Tuberculosis D. Metastases 2. The computed tomography (CT) chest pattern is best described as: A. Tree-in-bud B. Random C. Perilymphatic D. Centrilobular 3. A tree-in-bud pattern on CT is most suggestive of which of the below conditions? A. Aspiration B. Edema

Fig. 14.1

14

C. Metastases D. Hemorrhage 4. What does a random pattern of diffuse nodular lung disease imply? A. Lymphatic spread B. Hematogenous spread C. Aerogenous spread D. Mixed spread See Figure S14.1. See Supplemental Figures section for additional gures for this case. [E-book/Inkling – Link to supplemental gures Case 14]

Fig. 14.2

Case 15 History: History withheld. 1. Which of the following should be included in the differential diagnosis for this patient (Figs.15.1A and 15.1B)? (Choose all that apply.) A. Pulmonary metastases B. Neurobromatosis C. Pleural metastases D. Pulmonary histoplasmosis 2. What imaging sign is exemplied in this case example? A. Incomplete border B. Hilar overlay C. Cervicothoracic D. Monod

Fig. 15.1A

3. Which of the following is least common in neurobro matosis 1? A. Caféaulait spots B. Scoliosis C. Acoustic neuromas D. Optic gliomas 4. Two symmetrically located nodules are noted overlying the lower thorax; both are welldened laterally but poorly dened medially. What is the next step in management? A. Inandout of phase magnetic resonance imaging (MRI) B. Chest computed tomography (CT) with contrast C. Chest radiograph with BB markers D. Positron emission tomography (PET)/CT

Fig. 15.1B

15

Case 16 History: Outpatient radiograph in a 55-year-old man with subacute cough. 1. Based on the radiograph (Fig. 16.1), which of the following should be included in the differential diagnosis? A. Pneumonia B. Atelectasis C. Pleural effusion D. Chest wall mass 2. What is the most likely cause of this condition in this man (Fig. 16.2)? A. Aspiration B. Hypoventilation C. Lung cancer D. Carcinoid tumor

3. What would be the most likely cause of this condition in a child? A. Aspiration B. Hypoventilation C. Lung Cancer D. Carcinoid tumor 4. Which of the following signs is associated with this condition? A. Cervicothoracic sign B. S sign of Golden C. Luftsichel sign D. Flat waist sign

Fig. 16.2

Fig. 16.1

16

Case 17 History: Chronic dyspnea. 1. What is the diagnosis (Fig.17.1)? A. Pulmonary hypertension B. Aortic dissection C. Pulmonary stenosis D. Sarcoidosis 2. Which of the following is not a cause for this condition? A. Left-to-right shunt B. Mitral stenosis C. Calcium channel blockers D. Sleep apnea 3. What is the upper limit of normal for main pulmonary artery size on computed tomography (CT)? A. 2 cm B. 2.5 cm C. 3 cm D. 3.5 cm 4. Which of the following is a secondary cause of pulmonary hypertension? A. Portopulmonary hypertension B. Renopulmonary hypertension C. Adrenopulmonary hypertension D. Splenopulmonary hypertension Fig. 17.1

17

Case 18 History: Middle-aged man with fever and cough. 1. What is the differential diagnosis for cavitary nodule(s) (Fig.18.1)? A. Vasculitis B. Necrobiotic nodules C. Metastatic disease D. Abscess 2. Which primary tumor is least likely to cause cavitary lung metastases? A. Sarcoma B. Transitional cell carcinoma C. Melanoma D. Squamous cell carcinoma

3. What is the most common predisposing cause or factor in formation of a lung abscess (Fig.18.2)? A. Trauma B. Pulmonary embolism C. Aspiration D. Air-trapping 4. In which lung segment is a lung abscess least likely to develop? A. Anterior segment, right upper lobe B. Superior segment, right lower lobe C. Posterior segment, right upper lobe D. Lateral basilar segment, right lower lobe

Fig. 18.1

Fig. 18.2

18

Case 19 History: Patient with long history of asthma with new-onset chest pain. 1. Which of the following could be a cause this imaging nding (Figs.19.1 and 19.2)? A. Asthma B. Esophageal rupture C. Ruptured bleb D. Tracheal rupture 2. Which of the following ectopic gas collections will not shift with patient position? A. Pneumomediastinum B. Pneumoperitoneum C. Pneumothorax D. Pneumopericardium

Fig. 19.1

3. Which anatomic landmark represents the upper limit of visualized gas in a pneumopericardium? A. Tracheal carina B. Left atrial appendage C. Aortic arch D. Coronary sinus 4. Which of the following signs can be seen in either pneumomediastinum or pneumopericardium? A. Naclerio V sign B. Continuous diaphragm sign C. Deep sulcus sign D. Ring around the artery sign

Fig. 19.2

19

Case 20 History: Acute onset shortness of breath. 1. Which of the following is the most likely cause this imaging nding (Fig.20.1)? A. Liver mass B. Pleural effusion C. Diaphragmatic paralysis D. Diaphragmatic eventration 2. What is the most sensitive radiographic technique to detect pleural effusion? A. Oblique B. Posteroanterior (PA) C. Anteroposterior (AP) D. Lateral decubitus 3. What is the most common cause of transudative pleural effusion? A. Congestive heart failure B. Nephrotic syndrome C. Hepatic hydrothorax D. Hypoalbuminemia 4. What is the most common cause of exudative pleural effusion? A. Pneumonia B. Lung cancer C. Pulmonary infarct D. Collagen vascular disease

20

Fig. 20.1

Case 21 History: Decreased left-sided breath sounds on auscultation. 1. Which of the following could cause this imaging nding (Figs.21.1 and 21.2)? A. Pneumothorax B. Pleural effusion C. Pneumoperitoneum D. Basilar emphysema 2. What is the most sensitive radiographic technique to detect pneumothorax? A. Oblique B. Posteroanterior (PA) C. Expiration PA D. Lateral decubitus

Fig. 21.1

3. In a young patient, what is the most common cause of pneumothorax? A. Primary spontaneous B. Secondary spontaneous C. Traumatic D. Iatrogenic 4. Which sign is diagnostic of a pneumothorax on supine radiographs? A. Ring around the artery sign B. Deep sulcus sign C. Westermark sign D. Air crescent sign

Fig. 21.2

21

Case 22 History: History of pulmonary hypertension. 1. What is the location of the abnormality on the right (Figs.22.1, 22.2 and 22.3)? A. Right upper lobe B. Right middle lobe C. Right lower lobe D. None of the above 2. Which of the following is the most likely diagnosis? A. Lung cancer B. Pulmonary hamartoma C. Pseudotumor D. Metastatic disease 3. What is the most common malignancy to involve the pleura? A. Mesothelioma B. Solitary brous tumor C. Metastatic disease D. Sarcoma 4. What is the most common type of accessory pulmonary ssure? A. Azygos ssure B. Superior accessory ssure C. Inferior accessory ssure D. Left minor ssure Fig. 22.2

Fig. 22.1

22

Fig. 22.3

Case 23 History: History of dysphagia and gastroesophageal reux. 1. What diagnoses should be included in the differential diagnosis based on the scout image (Figs.23.1 and 23.2)? A. Esophageal cancer B. Lung cancer C. Bronchogenic cyst D. Thymoma 2. Which of the following is true regarding uorodeoxyglucose (FDG)-positron emission tomography (PET) FDGPET imaging in the setting of esophageal cancer? A. High accuracy for T staging B. High accuracy for N staging C. High accuracy for M staging D. Not useful for TNM staging 3. What is the most common process causing abnormal convexity along the inferior aspect of the azygoesophageal recess? A. Hiatal hernia B. Left atrial dilation C. Esophageal cancer D. Lymphadenopathy 4. Regarding the differences between squamous cell carcinoma (SCC) and adenocarcinoma (AC) of the esophagus, which of the following is true? A. SCC is more common in the lower third of the esophagus than AC. B. AC is more common than SCC world-wide. C. The most common risk factor for AC is gastroesophageal reux. D. SCC has a better prognosis than AC.

Fig. 23.1

Fig. 23.2

Fig. 23.3

23

Case 24 History: Preoperative evaluation. 1. Which of the following should be included in the differential diagnosis for this patient (Fig.24.1)? (Choose all that apply.) A. Hamartoma B. Adenocarcinoma C. Calcied granuloma D. Calcied pleural plaque 2. Which of the following is not a benign pattern of calcication in smoothly marginated lung nodules? A. Diffuse B. Central C. Stippled D. Laminar 3. Which of the following features on computed tomography (CT) is the most suggestive that a solitary lung nodule is benign? A. Eccentric calcication B. Smooth margins C. Mixed solid and ground-glass attenuation D. Fat attenuation 4. Which of the following characteristics is associated with the highest likelihood ratio that a solitary lung nodule is malignant? A. Size >3 cm B. Spiculated margins C. Doubling time less than 30 days D. Upper lobe location

24

Fig. 24.1

Case 25 History: Lung cancer. 1. Which of the following should be included in the differential diagnosis for this patient (Figs.25.1 and 25.2)? (Choose all that apply.) A. Sarcoidosis B. Idiopathic pulmonary brosis C. Radiation pneumonitis D. Infectious pneumonia 2. At what time point following completion of radiation therapy is radiation pneumonitis usually detectable by chest radiography? A. >1 week B. 2–4 weeks C. 6–8 weeks D. >8 weeks

3. Which of the following factors plays the least direct role in contributing to risk for developing radiation pneumonitis? A. Tumor histology B. Concomitant chemotherapy C. Radiation dose D. Size of tumor 4. Which of the following statements regarding radiationinduced lung disease is true? A. Radiation pneumonitis is a prerequisite for developing radiation brosis. B. A normal chest radiograph excludes the diagnosis of radiation pneumonitis. C. Radiation brosis is a progressive condition that often results in respiratory failure. D. Radiation brosis usually develops 6 to 12 months after the completion of therapy.

Fig. 25.1

Fig. 25.2

25

Case 26 History: Hypxia and failure t wean frm ventilatr. 1. Which f the fllwing shuld be included in the differential diagnsis fr this patient (Fig.26.1)? (Chse all that apply.) A. Acute respiratry distress syndrme (ARDS) B. Aspiratin C. Sarcidsis D. Bacterial pneumnia 2. Aside frm diffuse lung pacities, what ther abnrmality is present n this patient’s chest radigraph? (Chse all that apply.) A. Pneummediastinum B. Pneumpericardium C. Pneumperitneum D. Pneumthrax

Fig. 26.1

26

3. What is the mst likely cause f pneumthrax in this patient? A. Central venus catheter placement B. Bartrauma C. Brnchial rupture D. Pulmnary laceratin 4. Which f the fllwing is not a criterin fr the diagnsis f ARDS? A. Pao2/Fio2 ≤300 mmHg B. Bilateral lung pacity n chest radigraphy C. Hypxemic respiratry failure within ne week f insult D. Pleural effusin

Case 27 History: 55-year-old man with worsening cough and weight loss. Query lung carcinoma. 1. Which of the following should be included in the differential diagnosis of this patient (Figs.27.1 and 27.2)? (Choose all that apply.) A. Lung cancer B. Granulomatosis with polyangiitis C. Sarcoidosis D. Tuberculosis 2. Which cell type of lung cancer is most closely associated with the presence of cavitation? A. Adenocarcinoma B. Small cell carcinoma C. Squamous cell carcinoma D. Large cell carcinoma

Fig. 27.1

3. Which one of the following is most suggestive of a malignant cavitary lesion? A. Thick (>4 mm) wall B. Intracavitary uid level C. Development in a preexisting area of consolidation D. Lower lobe location 4. Which of the following is the next best step in the management of this patient? A. Bronchoscopy with bronchoalveolar lavage B. Transthoracic ne needle aspiration C. Respiratory isolation D. Fluorodeoxyglucose (FDG)-positron emission tomography (PET)/computed tomography (CT)

Fig. 27.2

27

Case 28 History: Hemoptysis. 1. Which of the following should be included in the differential diagnosis for this patient (Figs.28.1, 28.2 and 28.3)? (Choose all that apply.) A. Lung carcinoma B. Progressive massive brosis C. Mycetoma D. Pulmonary Langerhans cell histiocytosis 2. Which organism is most commonly associated with mycetoma? A. Aspergillus B. Blastomyces C. Candida D. Coccidioides 3. What one of the following is the most common complication of a mycetoma? A. Spread of infection to others B. Hemoptysis C. Dissemination of infection D. Lung carcinoma 4. Which of the following is preferred rst-line therapy for patients with mycetoma and acute severe hemoptysis? A. Antifungal therapy B. Pneumonectomy C. Bronchial artery embolization D. Radiation therapy Fig. 28.2

Fig. 28.1

28

Fig. 28.3

Case 29 History: 33-year-old man presenting for employment physical examination. 1. Based on the chest radiograph (Fig.29.1), which of the following should be included in the differential diagnosis for this patient? (Choose all that apply.) A. Pericardial cyst B. Lymphoma C. Paraganglioma D. Morgagni hernia 2. Based on the computed tomography (CT) scan image, what one of the following is the best diagnosis for this patient (Fig.29.2)? A. Pericardial cyst B. Lymphoma C. Paraganglioma D. Morgagni hernia

Fig. 29.1

3. Which of the following is true regarding pericardial cysts? A. Pericardial cysts occur more commonly in the left pericardiophrenic space. B. Most pericardial cysts communicate with the pericardium. C. Most pericardial cysts attach to the parietal pericardium. D. Most pericardial cysts cause symptoms. 4. Which of the following MR = magnetic resonance ndings of pericardial cysts is correct? A. Most pericardial cysts have high signal intensity on T1-weighted imaging. B. T2-signal intensity is heterogeneous. C. T1 postcontrast images show thin capsular enhancement. D. T1 postcontrast images show no internal enhancement.

Fig. 29.2

29

Case 30 History: 20-year-old female with fever and cough. 1. Which of the following should be included in the differential diagnosis for this patient? (Choose all that apply.) A. Acute histoplasmosis B. Community-acquired pneumonia C. Pleural effusion D. Septic emboli 2. What is the term used to describe the air-lled, tubular, branching structures that are visible within the right lower lobe (Figs.30.1 and 30.2)? A. Air bronchogram B. Pseudocavitation C. Pulmonary interstitial emphysema D. Black bronchus sign

Fig. 30.1

30

3. Which organism is most often associated with lobar pneumonia in the normal (immunocompetent) host? A. Aspergillus fumigatus B. Legionella pneumophila C. Staphylococcus aureus D. Streptococcus pneumoniae 4. What sign is used to describe obscuration of the right hemidiaphragm on the posteroanterior (PA) view? A. Sail sign B. Spine sign C. Silhouette sign D. Continuous hemidiaphragm sign

Fig. 30.2

Case 31 History: Leukocytosis and cough. 1. Which of the following should be included in the differential diagnosis for this patient (Fig.31.1)? (Choose all that apply.) A. Chylothorax B. Empyema C. Malignant effusion D. Hemothorax 2. Which of the following best suggests empyema on computed tomography (CT)? A. Gravitational layering of uid B. Lenticular shape of pleural collection C. Gas within the pleural collection D. Compressive atelectasis of adjacent lung 3. What is the most likely cause of an air uid level within a pleural uid collection? A. Gas-forming organisms B. Pulmonary gangrene C. Pleurocutaneous stula D. Bronchopleural stula 4. What is the most likely type of organism causing this abnormality? A. Bacterial B. Fungal C. Parasitic D. Viral

Fig. 31.1

31

Case 32 History: Chest pain. 1. Which of the following should be included in the differential diagnosis for this patient (Figs.32.1, 32.2 and 32.3)? (Choose all that apply.) A. Schwannoma B. Thymoma C. Ganglioneuroma D. Germ cell tumor 2. In which mediastinal compartment is the mass? A. Anterior mediastinum B. Middle mediastinum C. Posterior mediastinum D. Inferior mediastinum 3. Which of the following is the next best step in management of this patient? A. Chest computed tomography (CT) B. Thoracic spine magnetic resonance imaging (MRI) C. Fluorodeoxyglucose-positron emission tomography (FDG-PET)/CT D. Biopsy 4. Which of the following regarding neurogenic tumors is true? A. Most neurogenic tumors are malignant. B. Rib spreading and erosions imply malignancy. C. Tumors of the sympathetic chain often widen the neural foramen. D. Calcication is more common in sympathetic chain tumors than peripheral nerve tumors.

Fig. 32.1

Fig. 32.3

Fig. 32.2

32

Case 33 History: 50-year-old male with dyspnea and cough with increasing chest pain three days after slipping on ice. 1. Which of the following should be included in the differential diagnosis for this patient (Fig.33.1)? (Choose all that apply.) A. Hemothorax B. Empyema C. Chylothorax D. Solitary brous tumor 2. Which of the following most likely suggests hemothorax? A. Rapidly enlarging pleural collection following heart surgery B. Slowly enlarging pleural collection over several weeks C. Pleural collection loculated in the major ssure D. Homogeneous, low-attenuation pleural collection on computed tomography (CT)

3. What sign describes layers of different attenuation in a hemothorax? A. Split pleura sign B. Dependent viscera sign C. Collar sign D. Hematocrit sign 4. What is the next best management for this hemodynamically stable patient? A. Thoracotomy B. Tube thoracostomy C. Transcatheter embolization D. Talc pleurodesis

Fig. 33.1

33

Case 34 History: Multiple skin lesions. 1. Which of the following should be included in the differential diagnosis for this patient (Fig.34.1)? (Choose all that apply.) A. Tuberous sclerosis complex B. Metastases C. Neurobromatosis type 1 (NF-1) D. Takayasu arteritis 2. Which of the following is not an osseous manifestation of NF-1? A. Widened neural foramen B. Rib erosion C. Vertebra plana D. Scoliosis 3. What is the most common cause of destruction of a rib in an adult? A. Primary bone sarcoma B. Metastases C. Multiple myeloma D. Metabolic bone disease 4. What congenital aortic anomaly is associated with rib notching? A. Right aortic arch B. Cervical aortic arch C. Aberrant subclavian artery D. Aortic coarctation

34

Fig. 34.1

SECTION II

Fair Game

Case 35 History: 60-year-old man with cough. 1. Which radiographic sign is demonstrated (Fig.35.1)? A. S-sign of Golden B. Rock of Gibraltar C. Luftsichel sign D. Flat waist sign 2. In an adult outpatient, which is the most likely cause for lobar atelectasis? A. Aspirated foreign body B. Endobronchial mucus plug C. Lung cancer D. Bronchomalacia

Fig. 35.1

3. In an adult inpatient, which is the most likely cause for lobar atelectasis? A. Aspirated foreign body B. Endobronchial mucus plug C. Lung cancer D. Bronchomalacia 4. The revised 8th edition TNM Staging Classication for Lung Cancer considers any degree of post-obstructive atelectasis or pneumonitis extending to the hilum to represent at least what T (tumor) stage? A. T1 B. T2 C. T3 D. T4

Fig. 35.2

35

Case 36 History: Shortness of breath in 48-year-old man. 1. Based on the radiographic appearance, what is the most likely diagnosis (Fig.36.1)? A. Cardiogenic pulmonary edema B. Noncardiogenic pulmonary edema C. Lymphangitic carcinomatosis D. Community acquired pneumonia 2. What of the following is not a typical radiologic feature of this entity? A. Architectural distortion B. Perihilar haze C. Septal thickening D. Cardiomegaly 3. What is the most common cause for this radiographic appearance? A. Drug toxicity B. Left heart disease

Fig. 36.1

36

C. Neurologic disease D. Inhalational injury 4. Which of the following is true regarding the imaging features of this entity? (Choose all that apply.) A. Associated with increased hydrostatic pressure in pulmonary capillaries B. Affects the axial, peripheral, and parenchymal interstitium C. Widening of the vascular pedicle to greater than 58 mm is a marker for increased central vascular pressure D. Radiographic improvement may lag behind clinical parameters E. All of the above

Fig. 36.2

Case 37 History: Elderly woman with incidental nding on chest radiography. 1. What radiographic sign is demonstrated (Fig.37.1)? A. Thoracoabdominal sign B. Hilum convergence sign C. Hilum overlay sign D. Cervicothoracic sign 2. In which radiographic space is the abnormality located on the lateral radiograph (Fig.37.2)? A. Retrotracheal space B. Retrosternal space C. Infrahilar window D. Paravertebral space 3. In which anatomic compartment is the abnormality located? A. Prevascular mediastinum B. Visceral mediastinum C. Paravertebral mediastinum D. Pleural space

Fig. 37.1

4. Which of the following imaging features of this entity might prospectively predict the need for sternotomy in the surgical management of this lesion? A. Tracheal deviation B. Visceral mediastinal extension C. Paravertebral mediastinal extension D. Subcarinal extension E. Intrathoracic component larger than thoracic inlet F. Dumbbell or “conical” shape G. B, C, D, E, and F H. All of the above

Fig. 37.2

37

Case 38 History: 68-year-old female with otherwise noncontributory medical history presents with 3 days of fever and cough. 1. In which lobe of the lung is the abnormality located (Figs.38.1 and 38.2)? A. Right upper lobe B. Right lower lobe C. Right middle lobe D. Left upper lobe E. Left lower lobe 2. Which imaging sign is demonstrated on the radiograph (Figs.38.1 and 38.2)? (Choose all that apply.) A. Spine sign B. Halo sign C. Reverse halo sign D. Silhouette sign

3. Which of the following organisms is most likely responsible for these patterns? A. Mycoplasma pnemoniae B. Bacillus anthracis C. Histoplasma capsulatum D. Streptococcus pneumoniae

Fig. 38.1

Fig. 38.2

38

Case 39 History: 33-year-old male smoker with progressive cough. 1. What is the most likely cause for the high-resolution computed tomography (HRCT) ndings (Figs.39.1 and 39.2)? A. Metastatic disease B. Usual interstitial pneumonia (UIP) C. Pulmonary Langerhans cell histiocytosis D. Sarcoidosis 2. Which pair of associations is typical of this entity? A. Skin folliculomas and renal cell carcinoma B. Intracranial subependymal tubers and renal angiomyolipomas C. Perilymphatic micronodules and hilar lymphadenopathy D. Smoking history and younger age

3. Which portion of the lung is typically spared in this disorder? A. Upper lobes B. Costophrenic sulci C. Apices D. Terminal bronchioles 4. Which of the following choices list the two most common complications of the disease? A. Emphysema and lung cancer B. Chylothorax and pneumothorax C. Pneumothorax and pulmonary hypertension D. Recurrent infections and bronchiectasis

Fig. 39.2 Fig. 39.1

39

Case 40 History: 62-year-old former smoker presents with dyspnea. 1. Based on the CT ndings, what is the most likely diagnosis (Fig.40.1)? A. Lymphangitic carcinomatosis B. Sarcoidosis C. Idiopathic pulmonary brosis D. Lymphoma 2. Which of the following is a high-resolution computed tomography (HRCT) feature of this condition? (Choose all that apply.) A. Smooth or nodular axial interstitial thickening B. Smooth or nodular interlobular septal thickening C. Smooth or nodular ssural thickening D. Preservation of normal lung architecture E. All the above

Fig. 40.1

40

3. Which of the following primary malignant neoplasms is associated with lymphangitic carcinomatosis? (Choose all that apply.) A. Colon B. Lung C. Breast D. Stomach E. All of the above 4. Which of the following histologic subtypes of malignancy is most likely to exhibit a lymphangitic pattern? A. Squamous cell carcinoma B. Sarcoma C. Adenocarcinoma D. Neuroendocrine cell

Case 41 History: 57-year-old asymptomatic man. 1. What should be included in the differential diagnosis for the parenchymal abnormality (Fig.41.1)? A. Pneumonia B. Pulmonary infarct C. Lung cancer D. Rounded pneumonia E. All of the above 2. Which computed tomography (CT) sign is demonstrated (Fig.41.2)? A. Reverse halo sign B. Comet tail sign C. Bronchus cut-off sign D. Dark bronchus sign 3. What is the most likely diagnosis for the parenchymal abnormality? A. Pneumonia B. Rounded atelectasis

Fig. 41.1

C. Lung cancer D. Pulmonary infarct 4. Which of the following features must be present to make a denitive imaging diagnosis? A. Rounded, wedge-shaped, or lentiform morphology of mass-like opacity B. Pleural abnormality in contact with the subpleural masslike opacity C. Signs of volume loss in the affected lobe D. Traction and convergence of adjacent bronchovascular structures E. All of the above

Fig. 41.2

41

Case 42 History: 44-year-old female with dyspnea on exertion. 1. Which of the following entities is associated with upper lung zone architectural distortion? A. Sarcoidosis B. Tuberculosis C. Pneumoconiosis (e.g., silicosis, berylliosis) D. Chronic hypersensitivity pneumonitis E. All of the above 2. Which radiographic stage of the diagnosis is depicted (Fig.42.1)? A. Stage 0 B. Stage I C. Stage II D. Stage III E. Stage IV

3. Which term is used to describe the airway abnormalities associated with pulmonary brosis? A. Traction bronchiectasis B. Honeycombing C. Bronchomalacia D. Bronchial stenosis E. Obliterative bronchiolitis 4. Which laboratory value is often elevated in patients with this condition? A. Alpha-1 antitrypsin B. Angiotensin converting enzyme (ACE) C. pCO2 D. Eosinophil count E. Cytoplasmic anti-neutrophil cytoplasmic antibody (c-ANCA)

Fig. 42.1

Fig. 42.2

Fig. 42.3

42

Case 43 History: 66-year-old female smoker. 1. What is the anatomic distribution of the abnormality (Figs.43.1 and 43.2)? A. Paraseptal B. Centrilobular C. Panlobular D. Paracicatricial 2. What is the most common cause of the abnormality? A. Autoimmune disease B. Drug reaction C. Smoking D. Positive airway pressure

3. What distribution is typical for this condition? A. Lower lobes B. Upper lobes C. Asymmetric D. Random 4. True or False? This condition is reversible with smoking cessation. A. True B. False

Fig. 43.1 Fig. 43.2

43

Case 44 History: Asymptomatic 66-year-old female with incidental nding on routine imaging. 1. In what mediastinal compartment is the abnormality located (Figs.44.1, 44.2 and 44.3)? A. Prevascular (anterior) B. Visceral (middle) C. Paravertebral (posterior) D. Superior 2. What is the most common primary neoplasm of the prevascular mediastinum? A. Thymoma B. Thymic carcinoma C. Thymic neuroendocrine tumor D. Lymphoma E. Germ cell neoplasm 3. At which age are thymomas typically encountered? A. 5 to 10 years B. 20 to 30 years C. 30 to 40 years D. >40 years 4. Which paraneoplastic syndrome is most commonly associated with thymoma? A. Myasthenia gravis B. Lambert-Eaton syndrome C. Syndrome of inappropriate antidiuretic hormone D. Serotonin syndrome

Fig. 44.1

44

Fig. 44.2

Fig. 44.3

Case 45 History: 67-year-old male smoker with weight loss. 1. What is the most likely diagnosis (Figs. 45.1A and 45.1B)? A. Bronchogenic cyst B. Hiatal hernia C. Mediastinal hematoma D. Lymphadenopathy 2. In which anatomic space is the abnormality located? A. Paraesophageal B. Subaortic C. Right paratracheal D. Subcarinal 3. In a patient with right upper lobe lung cancer, involvement of this level represents, at a minimum, which tumor, node, metastasis (TNM) nodal classication? A. N0 B. N1 C. N2 D. N3

4. Which interventional staging procedure(s) can be used to access and sample abnormalities in this location? A. Bronchoscopy B. Endobronchial ultrasound (EBUS) C. Esophageal endoscopic ultrasound (EUS) D. Mediastinoscopy E. all of the above

Fig. 45.1A

Fig. 45.1B

45

Case 46 History: Asymptomatic male. 1. What is the predominant imaging abnormality (Figs. 46.1A, 46.1B, 46.1C, 46.1D, and 46.1E)? A. Pleural plaques B. Nodules C. Masses D. Architectural distortion 2. Which of the following is most likely to have been the patient’s occupation? A. Business executive B. Airplane pilot C. Fireman D. Quarry worker

3. In the simple form of this disease, which represents the typical temporal relationship from the exposure to the disease manifestation? A. 1 to 2 hours B. 1 to 2 weeks C. 1 to 2 years D. 10 to 20 years 4. The International Labor Ofce has established guidelines for the objective classication of this entity using which of the following? A. Pulmonary function test B. Whispered pectoriloquy C. Digital chest radiography D. Chest computed tomography

Fig. 46.1C

Fig. 46.1A

Fig. 46.1D

46

Fig. 46.1B

Fig. 46.1E

Case 47 History: 50-year-old woman with progressive decline in lung function. 1. Which of the following descriptors should be excluded as a predominant nding in this case (Fig.47.1)? A. Peripheral B. Nodules C. Basilar predominant D. Ground-glass E. Subpleural sparing 2. Which of the following is the most likely cause for this computed tomography (CT) pattern in this patient? A. Cigarette smoking B. Connective tissue disease C. Hot tub use D. Asbestos exposure

3. Which of the following associated features shown may help narrow the differential diagnosis for the CT pattern in this patient? A. Traction bronchiectasis B. Coronary artery calcication C. Dilated pulmonary artery D. Esophageal dilation 4. Elevation of which serologic markers would be characteristic in this disease? A. Rheumatoid factor, anti-CCP B. Anti-ribonuclear protein C. Anti-centromere antibody, anti-SCL-70 D. Anti-SS-A (Ro), anti-SS-B (La)

Fig. 47.1

47

Case 48 History: 44-year-old asymptomatic female with positive puried protein derivative (PPD) skin test during pre-employment physical. 1. What is the most likely cause of the imaging abnormality (Figs. 48.1A and 48.1B)? A. Asbestosis B. Broncholith C. Calcied metastasis D. Healed tuberculosis 2. Which historical eponym is given to the parenchymal nding? A. Fleischner sign B. Ghon focus C. S-sign of Golden D. Hampton hump

Fig. 48.1A

48

3. Which historical eponym is given to the combination of the parenchymal and hilar nding? A. Ranke complex B. Ghon focus C. Kerley’s line D. Naclerio V-sign 4. Which of the following is included in the differential diagnosis for a calcied pulmonary nodule? (Choose all that apply.) A. Healed granulomatous disease B. Hamartoma C. Pneumoconiosis D. Carcinoid E. Calcic metastases F. Pulmonary ossication G. All of the above

Fig. 48.1B

Case 49 History: 49-year-old woman with chronic cough. 1. What is the most prominent imaging abnormality (Fig.49.1A)? A. Bronchiectasis B. Dilated pulmonary artery C. Middle lobe atelectasis D. Lingular atelectasis 2. Which of the following denes bronchiectasis by imaging criteria? A. Bronchoarterial ratio >0.5 B. Bronchoarterial ratio >1.0 C. Bronchoarterial ratio >1.5 D. Greater than 70% luminal cross-sectional area collapse on expiration E. Greater anteroposterior airway diameter than transverse diameter

Fig. 49.1A

3. Which computed tomography (CT) imaging sign is associated with this condition (Fig.49.1B)? A. Signet ring sign B. Three density sign C. Fallen viscera sign D. Reverse CT halo sign 4. Which of the following belong in the differential diagnosis? (Choose all that apply) A. Cystic brosis B. Allergic bronchopulmonary aspergillosis (ABPA) C. Chronic aspiration D. Chronic or recurrent infection E. Williams-Campbell syndrome F. Primary ciliary dyskinesia G. All of the above

Fig. 49.1B

49

Case 50 History: 29-year-old woman with an incidental nding on chest radiography. 1. Which of the following should be included in the differential diagnosis of the chest radiographic nding (Figs. 50.1 and 50.2)? (Choose all that apply.) A. Lung cancer B. Septic embolus C. Arteriovenous malformation D. Pulmonary artery pseudoaneurysm E. All of the above 2. What percent of patients with this abnormality have the inherited disorder known as hereditary hemorrhagic telangiectasia? A. Less than 1% B. 1% to 20%

C. 20% to 40% D. 40% to 60% E. 60% to 80% F. Greater than 80% 3. Which of the following is the most common presentation for persons with pulmonary arteriovenous malformation (AVMs)? A. Cerebral abscess B. Hemoptysis C. Cough D. Asymptomatic

Fig. 50.2

Fig. 50.1

50

Case 51 History: 52-year-old woman with an abnormal chest radiograph (Figs. 51.1A and 51.1B). 1. What is the most likely cause of the mediastinal mass? A. Mediastinal hematoma B. Lymphadenopathy C. Ascending aortic aneurysm D. Thymic neoplasm 2. What percentage of mediastinal masses are vascular in etiology? A. Less than 1% B. 10%

Fig. 51.1A

C. 20% D. 50% E. 75% 3. In a patient with Marfan syndrome, what is the threshold dimension of luminal dilatation that prompts surgical repair of an aortic aneurysm? A. 4.0 cm B. 4.5 cm C. 5.0 cm D. 5.5 cm E. 6.0 cm

Fig. 51.1B

51

Case 52 History: 32-year-old man with right chest pain and dyspnea. 1. Which of the following should be included in the differential diagnosis of the chest radiographic nding (Fig. 52.1)? (Choose all that apply.) A. Lung cancer B. Pneumonia C. Pulmonary infarct D. Pulmonary contusion E. All of the above 2. Which of the following indicates a radiographic manifestation of pulmonary infarction? A. Wells score B. Hampton hump

Fig. 52.1

52

C. Westermark sign D. Knuckle sign E. Fleishner sign 3. Which of the following is a poor prognostic indicator in patients with acute pulmonary emboli? A. Pulmonary infarct B. Hemoptysis C. RV:LV ratio >1.0 D. Subsegmental DECT perfusion defects

Case 53 History: 25-year-old reghter with fever, fatigue, and night sweats. 1. Which of the following should be included in the differential diagnosis of the chest radiographic nding (Fig. 53.1)? (Choose all that apply) A. Metastasis B. Lymphoma C. Germ cell neoplasm D. Thymic epithelial neoplasm E. All of the above 2. Which of the following is true regarding “bulky” disease for Hodgkin lymphoma (Figs.53.2 and 53.3)? A. Portends a favorable prognosis B. Dened by chest radiograph extent C. Dened by a single nodal mass measuring greater than 3 cm on computed tomography (CT) D. Dened by a single nodal mass occupying greater than 1/3 of thoracic diameter 3. Which of the following is considered the gold standard imaging modality for initial, interim, and end-of-treatment assessment of lymphoma? A. Chest radiograph B. Non-contrast chest CT C. Fluorodeoxyglucose -positron emission tomography (18FDG-PET)-CT D. Contrast-enhanced thoracic magnetic resonance imaging (MRI)

Fig. 53.2

Fig. 53.1

Fig. 53.3

53

Case 54 History: 35-year-old man after high-speed motorcycle accident. 1. Which of the following are the most common sites of traumatic aortic injury (TAI). (Choose all that apply.) A. Aortic root B. Aortic isthmus C. Sinotubular junction D. Branch vessel origins E. Diaphragmatic hiatus 2. Which of the following ndings on the radiograph supports the diagnosis of TAI over other mimics (Fig.54.1)? A. Atherosclerotic plaque B. Sinotubular junction dilation

Fig. 54.1

54

C. Rib notching D. Periaortic hemorrhage 3. What is the typical in-the-eld mortality associated with TAI? A. 10% to 20% B. 20% to 30% C. 40% to 50% D. 50% to 60% E. 80% to 90%

Case 55 History: 88-year-old woman referred to CT for an abnormal chest radiograph (Figs.55.1 and 55.2). 1. Which of the following should be included in the differential diagnosis for chronic airspace opacity? (Choose all that apply.) A. Lung adenocarcinoma B. Lymphoma C. Alveolar proteinosis D. Organizing pneumonia E. Lipoid pneumonia 2. Which of the following ndings supports the diagnosis of lipoid pneumonia in the setting of a chronic consolidation? A. Spiculated margin B. Long-term stability

Fig. 55.1

C. Macroscopic fat attenuation D. Central cavitation 3. Which of the following may predispose a patient to developing lipoid pneumonia? A. Young age B. Smoking C. Positive pressure ventilation D. Zenker diverticulum E. Intravenous (IV) drug abuse

Fig. 55.2

55

Case 56 History: 33-year-old man referred for an abnormal chest radiograph (Fig.56.1) and infertility. 1. What is the most likely diagnosis (Figs.56.2 and 56.3)? A. Allergic bronchopulmonary Aspergillosis B. Kartagener syndrome C. Cystic brosis D. Recurrent aspiration E. Bronchial atresia 2. Which of the following comprise the triad associated with this entity? (Choose three.) A. Bronchiectasis B. Situs solitus C. Situs inversus D. Sinusitis E. Extralobar sequestration 3. Which of the following is the typical pattern of inheritance of this disorder? A. Autosomal dominant B. Autosomal recessive C. X-linked dominant D. X-linked recessive E. Mosaicism Fig. 56.2

Fig. 56.1

56

Fig. 56.3

Case 57 History: 59-year-old woman on chronic steroids for asthma presents with hemoptysis. 1. Which of the following should be included in the differential diagnosis for the radiographic ndings (Fig.57.1)? (Choose all that apply.) A. Pulmonary infarct B. Lung cancer C. Invasive aspergillus D. Rounded atelectasis E. Organizing pneumonia 2. What is the most likely diagnosis in this case? A. Pulmonary infarct B. Lung cancer C. Invasive aspergillus D. Rounded atelectasis E. Organizing pneumonia

Fig. 57.1

3. What clinical information in this case supports the diagnosis? A. Female sex B. History of asthma C. Chronic steroid therapy D. Peripheral location 4. Which computed tomography (CT) sign is demonstrated (Fig.57.2)? A. Signet ring B. Air bronchogram C. Comet tail D. Halo E. Reverse halo

Fig. 57.2

57

Case 58 History: 75-year-old man with epistaxis. 1. What should be included in the differential diagnosis of the imaging ndings (Figs.58.1 and 58.2)? (Choose all that apply.) A. Septic emboli B. Lung abscess C. Multifocal pneumonia D. Metastatic disease E. Vasculitis 2. What is the most likely diagnosis in this case? A. Septic emboli B. Pulmonary abscesses C. Multifocal pneumonia D. Metastatic disease E. Vasculitis

Fig. 58.1

58

3. What clinical information would support the diagnosis? A. Male sex B. Intravenous (IV) drug abuse C. Epistaxis D. Immunosuppression 4. What computed tomography (CT) sign is demonstrated? A. Signet ring B. Air bronchogram C. Comet tail D. CT halo E. Reverse CT halo

Fig. 58.2

Case 59 History: 52-year-old woman with chronic dyspnea (Figs. 59.1 and 59.2). 1. Which of the following should be included in the differential diagnosis for this patient? (Choose all that apply.) A. Langerhans cell histiocytosis B. Lymphangioleiomyomatosis C. Tuberous sclerosis D. Hermansky-Pudlak syndrome 2. What is a characteristic computed tomography (CT) nding in Langerhans cell histiocytosis? A. Ground-glass opacity B. Costophrenic angle sparing C. Consolidation D. Apical sparing

Fig. 59.1

3. Which of the conditions listed is associated with chylous pleural effusions? A. Lymphangioleiomyomatosis B. Langerhans cell histiocytosis C. Amyloidosis D. Lymphocytic interstitial pneumonitis 4. Which of the following ndings are not usually present in the setting of tuberous sclerosis? A. Fatty cardiac lesions B. Multinodular multifocal pneumocyte hyperplasia C. Hepatocellular carcinoma D. Angiomyolipomas

Fig. 59.2

59

Case 60 History: Young man with history of chest pain and cough (Fig.60.1). 1. Which of the following should be included in the differential diagnosis for this patient? (Choose all that apply.) A. Lymphoma B. Tuberculosis C. Metastatic disease D. Sarcoidosis 2. In the setting of tuberculosis, which pulmonary nding is more common in children than adults? A. Fibrocavitary disease B. Ground-glass opacity C. Bronchiectasis D. Consolidation 3. Tuberculous lymphadenopathy is common in which of the following conditions? A. Acquired immune deciency syndrome B. Rheumatoid arthritis C. Common variable immune deciency D. Lymphocytic interstitial pneumonitis 4. In which demographic is tuberculous pleuritis least common? A. Infants B. Adolescents C. Young adults D. Elderly

60

Fig. 60.1

Case 61 History: 49-year-old man with human immunodeciency virus (HIV) and shortness of breath (Fig.61.1). 1. Which of the following should be included in the differential diagnosis for this patient? (Choose all that apply.) A. Pulmonary edema B. Diffuse alveolar hemorrhage C. Pneumocystis jirovecii pneumonia D. Pulmonary alveolar proteinosis 2. Which of the following is another common computed tomography (CT) nding in patients with Pneumocystis jirovecii pneumonia? A. Pleural effusions B. Cysts C. Lymphadenopathy D. Tree-in-bud nodules 3. Which of the conditions listed below is associated with chylous pleural effusions? A. Lymphangioleiomyomatosis B. Langerhans cell histiocytosis C. Amyloidosis D. Lymphocytic interstitial pneumonitis 4. What is rst-line therapy for Pneumocystis jirovecii pneumonia? A. Trimethoprim-sulfamethoxazole B. Amoxicillin/clavulanic acid C. Linezolid D. Azithromycin

Fig. 61.1

61

Case 62 History: 25-year-old man; history withheld (Fig.62.1). 1. Which of the following should be included in the differential diagnosis for the appearance of the mediastinum and hila? (Choose all that apply.) A. Silicosis B. Tuberculosis C. Histoplasmosis D. Sarcoidosis 2. Calcied lymphadenopathy following treatment is seen most frequently in which disease? A. Cat scratch disease B. Acquired immunodeciency syndrome (AIDS) C. Systemic sclerosis D. Lymphoma

3. Which of the following conditions is least likely to cause eggshell calcication in hilar and mediastinal lymphadenopathy? A. Silicosis B. Sarcoidosis C. Histoplasmosis D. Coal workers pneumoconiosis 4. Garland’s triad is highly associated with which of the following diseases? A. Sarcoidosis B. Lymphoma C. Tuberculosis D. Leukemia

Fig. 62.1

62

Case 63 History: 52-year-old man with fever, chills, and pleuritic chest pain (Fig.63.1). 1. Which of the following should be included in the differential diagnosis for this patient? (Choose all that apply.) A. Metastatic disease B. Septic infarcts C. Granulomatosis with polyangiitis D. Acute respiratory distress syndrome 2. What is the likely etiology for chronic cavitary lung nodules in a patient with joint pain and subcutaneous nodules? A. Tuberculosis B. Rheumatoid arthritis C. Granulomatosis with polyangiitis D. Invasive aspergillosis 3. Which the following is not a risk factor for development of septic infarcts? A. Central venous catheter B. Periodontal disease C. Pulmonary hypertension D. Intravenous drug use

4. What is the most common imaging manifestation of fat embolism? A. Ground-glass opacity B. Fatty attenuation pulmonary emboli C. Tree-in-bud nodules D. Consolidation

Fig. 63.1

63

Case 64 History: History of recent pneumonectomy. Two chest radiographs are shown acquired 2 days apart (Figs.64.1 and 64.2). 1. What is the diagnosis? A. Extrapleural hematoma B. Hemothorax C. Bronchopleural stula D. Expected postoperative evolution 2. What is likely underlying reason for a higher incidence of bronchopleural stula after right pneumonectomy as compared to left pneumonectomy? A. Obtuse angle of the bronchial stump B. Shorter length of bronchial stump C. Technically more difcult surgery D. Greater frequency of right pneumonectomies

Fig. 64.1

64

3. Which lobe is most prone to lobar torsion after ipsilateral lobectomy? A. Right middle lobe B. Right lower lobe C. Left upper lobe D. Left lower lobe 4. In the setting of suspected bronchopleural stula after pneumonectomy, what is the next step in management? A. Chest tube drainage B. Watchful waiting C. Surgical exploration D. Bronchoscopy

Fig. 64.2

Case 65 History: 59-year-old man with left shoulder pain and hand weakness. 1. The imaging ndings in this case are most commonly associated with which cell type of lung cancer (Figs.65.1, 65.2 and 65.3)? A. Small cell carcinoma B. Squamous cell carcinoma C. Large cell carcinoma D. Adenocarcinoma 2. If this tumor invaded the chest wall, what staging factor would make him an inoperable candidate? A. T3 disease B. N2 disease C. N3 disease D. Any nodal status

3. In the staging of lung cancer invasion of which of the following is considered T4 (non-operable)? A. Phrenic nerve B. Recurrent laryngeal nerve C. Diaphragm D. Parietal pericardium 4. What is the current standard therapy for most operable superior sulcus tumors? A. Chemotherapy B. Chemotherapy and radiotherapy C. Chemotherapy and radiotherapy with subsequent surgery D. Surgery with subsequent chemotherapy and radiotherapy

Fig. 65.3

Fig. 65.1

Fig. 65.2

65

Case 66 History: 26-year-old with fever, night sweats, and cough (Fig.66.1). 1. Which of the following should be included in the differential diagnosis for this patient? (Choose all that apply.) A. Lung cancer B. Fungal pneumonia C. TB D. Pulmonary infarct 2. Which lung cancer cell type is associated most frequently with cavitation? A. Small cell lung cancer B. Adenocarcinoma C. Large cell carcinoma D. Squamous cell carcinoma 3. In adult patients with tuberculosis, tree-in-bud opacities indicate involvement in which anatomic structures? A. Arterioles B. Lymphatics C. Bronchioles D. Alveoli 4. In the setting of acute pulmonary embolism, which comorbid condition is most frequently associated with the development of pulmonary infarction? A. Congestive heart failure B. Asthma C. Pulmonary brosis D. Tricuspid regurgitation

66

Fig. 66.1

Case 67 History: 63-year-old man with long history of smoking and chronic cough. 1. Which of the following lymph node stations demonstrate uorodeoxyglucose (FDG) avidity in this patient with non-small cell lung cancer (Figs.67.1, 67.2 and 67.3)? (Check all that apply.) A. Subaortic B. Low paratracheal C. High paratracheal D. Para-aortic E. Hilar 2. What nding makes this cancer unresectable? A. T3 disease B. N2 disease C. N3 disease D. M1 disease

3. What is the most common lobe affected by lung cancer? A. Right upper lobe B. Right middle lobe C. Right lower lobe D. Left lower lobe 4. What feature of lung cancer is least likely to be associated with a false negative result on positron emission tomography/ computed tomography (PET/CT)? A. Small size B. Carcinoid histology C. Ground-glass attenuation D. Spiculated margins

Fig. 67.1 Fig. 67.3

Fig. 67.2

67

Case 68 History: 57-year-old man with cough. 1. Which of the following should be included in the differential diagnosis for the nodule shown in (Fig. 68.1)? (Choose all that apply.) A. Pneumonia B. Aspiration C. Adenocarcinoma D. Scar or focal brosis 2. Which attenuation characteristic of a focal pulmonary nodule is most concerning for primary lung cancer? A. Ground-glass B. Part-solid C. Solid D. Solid with cavitation

Fig. 68.1

68

3. A solitary 4 to 5 mm ground-glass nodule is identied incidentally on a chest computed tomography (CT) for evaluation of chronic dyspnea. According to current guidelines, what is the next step in management? A. CT follow-up in 3 months B. CT follow-up in 6 months C. CT follow-up in 12 months D. No CT follow-up 4. Which of the following is least likely to cause diffuse centrilobular ground-glass nodules throughout the lungs? A. Hypersensitivity pneumonitis B. Metastatic disease C. Pulmonary hemorrhage D. Respiratory bronchiolitis

Case 69 History: 45-year-old woman with chronic dyspnea. 1. Which of the following should be included in the differential diagnosis for this patient based solely on the radiograph (Fig.69.1)? (Choose all that apply.) A. Lymphoma B. Azygos continuation of the inferior vena cava (IVC) C. Superior vena cava obstruction D. Partial anomalous pulmonary venous return 2. What is the most likely diagnosis based on all imaging available (Figs.69.1, 69.2 and 69.3)? A. Lymphoma B. Azygos continuation of the inferior vena cava C. Superior vena cava obstruction D. Partial anomalous pulmonary venous return

3. In the setting of chronic superior vena cava obstruction, which collateral veins allow drainage of abdominal wall veins into the anterior aspect of the liver, which may produce enhancing pseudo-hepatic lesions? A. Veins of Sappey B. Veins of Marshall C. Veins of Galen D. Veins of Mayo 4. When compared to asplenia heterotaxy syndrome, polysplenia is more often associated with which of the following? A. Higher prevalence of cyanotic congenital heart disease B. Higher prevalence of midgut malrotation C. Lower prevalence of azygos continuation of IVC D. Lower prevalence of bilateral hyparterial bronchi

Fig. 69.1

69

70

SECTION II

Fair Game

Fig. 69.2

Fig. 69.3

Case 70 History: 75-year-old man with cough and dyspnea (Figs. 70.1 and 70.2). 1. Which of the following should be included in the differential diagnosis for this patient based solely on the radiograph? (Choose all that apply.) A. Pneumonia B. Aspiration C. Lung cancer D. Lymphoma 2. What imaging sign is demonstrated in this case? A. Flat-waist sign B. Hilar-overlay sign C. Ivory-heart sign D. S sign of Golden

Fig. 70.1

3. What tumor is most likely in a young adult with lobar collapse associated with a calcied endobronchial lesion? A. Hamartoma B. Carcinoid tumor C. Adenoid cystic carcinoma D. Mucoepidermoid carcinoma 4. Based solely on the presence of postobstructive atelectasis present, what is the correct T (tumor) stage if the underlying lesion were a non–small cell lung cancer (NSCLC)? A. T1 B. T2 C. T3 D. T4

Fig. 70.2

71

Case 71 History: 45-year-old man with history of falling off a ladder one-year ago. 1. Which of the following should be included in the differential diagnosis for this patient? (Choose all that apply.) A. Large pleural effusion B. Diaphragmatic rupture C. Diaphragmatic paralysis D. No abnormality 2. What imaging sign is present on the axial computed tomography (CT) images (Figs.71.1 and 71.2)? A. Thoracoabdominal sign B. Cobweb sign C. Fleischner sign D. Dependent viscera sign 3. In diaphragmatic rupture from blunt trauma, what location of injury is most common? A. Anterolateral right hemidiaphragm B. Anterolateral left hemidiaphragm C. Posterolateral right hemidiaphragm D. Posterolateral left hemidiaphragm 4. A patient presents with marked elevation of the right hemidiaphragm on radiography after thoracic surgery. What is the most appropriate next imaging modality? A. Magnetic resonance imaging (MRI) B. CT C. Fluoroscopy D. Ultrasound

Fig. 71.2 Fig. 71.1

72

Case 72 History: 38-year-old man with shortness of breath (Figs.72.1 and 72.2). 1. Which of the following should be included in the differential diagnosis for this patient? (Choose all that apply.) A. Left superior vena cava (SVC) B. Azygos continuation of the inferior vena cava (IVC) C. Partial anomalous pulmonary venous return (PAPVR) D. Abnormal left superior intercostal vein 2. Into which vessel do most left-sided SVCs drain? A. Right SVC B. Hemiazygos vein C. Coronary sinus D. Right atrium

Fig. 72.1

3. In the setting of right superior PAPVR, what is the most common associated anomaly? A. Left SVC B. Azygos continuation of the IVC C. Atrial septal defect (ASD) D. Patent ductus arteriosus (PDA) 4. An unroofed coronary sinus is a type of: A. Ventricular septal defect B. PAPVR C. PDA D. ASD

Fig. 72.2

73

Case 73 History: 56-year-old man with chronic cough (Fig.73.1). 1. Which of the following should be included in the differential diagnosis for this patient? (Choose all that apply.) A. Aspiration B. Non-tuberculous mycobacterial infection C. Primary ciliary dyskinesia D. Diffuse panbronchiolitis 2. Which diffuse lung disease is least likely to be associated with aspiration? A. Usual interstitial pneumonitis B. Diffuse alveolar damage C. Lymphocytic interstitial pneumonitis D. Organizing pneumonia 3. Which of the following pulmonary segments is least likely to be affected by aspiration? A. Right upper lobe anterior B. Right upper lobe posterior C. Right lower lobe superior D. Right lower lobe basal posterior 4. Of the listed choices, which is the least common sequelae of foreign body aspiration? A. Pneumothorax B. Pneumonia C. Atelectasis D. Bronchiectasis

74

Fig. 73.1

Case 74 History: 48-year-old man with shortness of breath and crackles on physical examination (Figs.74.1 and 74.2). 1. What is the most likely diagnosis? A. Usual interstitial pneumonitis (UIP) B. Nonspecic interstitial pneumonitis (NSIP) C. Acute interstitial pneumonitis (AIP) D. Desquamative interstitial pneumonitis (DIP) 2. Which nding is inconsistent with UIP? A. Lower lung zone preponderance B. Peripheral lung preponderance C. Honeycombing D. Predominant ground-glass opacity

Fig. 74.1

3. What is the most specic differentiator of NSIP from UIP? A. Upper lung zone distribution B. Mid lung zone distribution C. Subpleural sparing D. Reticulation 4. What is the least common cause of NSIP pattern of lung disease? A. Collagen vascular disease B. Drugs C. Hypersensitivity pneumonitis D. Idiopathic

Fig. 74.2

75

Case 75 History: 42-year-old man with pleuritic chest pain (Fig.75.1). 1. Which of the following are likely causes of the abnormal aorta? (Choose all that apply). A. Atherosclerosis B. Hypertension C. Marfan syndrome D. Ehlers Danlos syndrome 2. What is the size criteria for operative repair in ascending aortic aneurysms? A. >2.5 cm B. >3.5 cm C. >4.5 cm D. >5.5 cm 3. Which of the following is least often associated with bicuspid aortic valves? A. Aortic aneurysm B. Intracranial arterial aneurysm C. Turner syndrome D. Partial anomalous pulmonary venous return 4. There is an enlarged ascending aorta with loss of the normal concave junction between the sinus of Valsalva and tubular ascending aorta. This imaging nding is most specic for which condition? A. Syphilis B. Marfan syndrome C. Bicuspid aortic valve D. Atherosclerosis

76

Fig. 75.1

Case 76 History: 30-year-old woman with chronic cough (Figs. 76.1, 76.2 and 76.3). 1. What is the most likely diagnosis? A. Small cell carcinoma B. Lymphoma C. Squamous cell carcinoma D. Carcinoid tumor 2. What is the most common benign pulmonary tumor? A. Lipoma B. Hamartoma C. Chondroma D. Papilloma 3. What percentage of carcinoid tumors are calcied? A. 10% B. 20% C. 30% D. 40% 4. What is a major nding in diffuse idiopathic pulmonary neuroendocrine cell hyperplasia (DIPNECH)? A. Consolidation B. Fibrosis C. Pneumothorax D. Air-trapping

Fig. 76.2

Fig. 76.1

Fig. 76.3

77

Case 77 History: 58-year-old woman with acute chest pain (Figs. 77.1 and 77.2). 1. Which of the following etiologies should be considered? (Choose all that apply.) A. Childbirth B. Asthma C. Esophageal rupture D. Extension of pneumothorax 2. What condition is thought to be related to the development of asthma-related pneumomediastinum? A. Pneumothorax B. Pulmonary interstitial emphysema C. Honeycombing D. Centrilobular emphysema 3. In which anatomic area of the esophagus does rupture from vomiting usually occur? A. Left mid B. Right mid C. Right lower D. Left lower 4. What is the most serious complication of esophageal rupture? A. Pneumonia B. Pneumothorax C. Mediastinitis D. Empyema

Fig. 77.1

78

Fig. 77.2

Case 78 History: 67-year-old woman with chronic cough and malaise (Fig.78.1). 1. Which of the following should be included in the differential diagnosis for this patient? (Choose all that apply.) A. Nontuberculous mycobacterial infection B. Allergic bronchopulmonary aspergillosis C. Cystic brosis D. Usual interstitial pneumonitis 2. Which computed tomography (CT) postprocessing tool is most helpful in differentiating bronchiectasis from cystic lung disease? A. Maximum intensity projection B. 3D volume rendering C. Surface rendering D. Minimum intensity projection 3. Which of the following is not an imaging subtype of bronchiectasis? A. Spherical B. Varicoid C. Cystic D. Cylindrical 4. Excluding the pancreas, which is the most frequently involved abdominal organ in cystic brosis? A. Kidneys B. Liver C. Spleen D. Stomach

Fig. 78.1

79

Case 79 History: 43-year-old man with dyspnea. 1. Which of the following should be included in the differential diagnosis for this patient based on the chest radiograph (Fig.79.1)? (Choose all that apply.) A. Pneumothorax B. Giant bulla C. Poland syndrome D. Langerhans cell histocytosis 2. What is the size criterion that distinguishes a bulla from a bleb? A. ≥0.5 cm B. ≥1 cm C. ≥1.5 cm D. ≥2 cm 3. Which of the following tests is most sensitive for the detection of early emphysema? A. Tomosynthesis B. Pulmonary function tests C. Hyperpolarized He magnetic resonance imaging (MRI) D. Chest computed tomography (CT) 4. Which illicit drug has been associated with basilar predominant panlobular emphysema? A. Smoked crack cocaine B. Smoked marijuana C. Intravenous (IV) heroin D. IV Ritalin

Fig. 79.1

80

Case 80 History: 61-year-old woman with history of breast cancer (Figs.80.1 and 80.2). 1. Which is collapsed? A. Right upper lobe B. Right middle lobe C. Right lower lobe D. Combined right middle and lower lobes 2. Which bronchus is obstructed? A. Right upper lobe bronchus B. Right middle lobe bronchus C. Bronchus intermedius D. Right lower lobe bronchus

Fig. 80.1

3. In a recently intubated patient, what is the most likely cause of complete right lung atelectasis? A. Foreign body aspiration B. Mucous plugging C. Endobronchial tumor D. Endotracheal tube misplacement 4. Which of the listed primary tumors are least likely to cause central airway metastases? A. Melanoma B. Renal cell carcinoma C. Breast cancer D. Hepatocellular carcinoma

Fig. 80.2

81

Case 81 History: 41-year-old woman with chest asymmetry on examination (Figs.81.1, 81.2 and 81.3). 1. Which of the following should be included in the differential diagnosis for this patient? (Choose all that apply.) A. Lipoma B. Hamartoma C. Liposarcoma D. Fat necrosis 2. Which of the following material is not inherently T1 hyperintense on magnetic resonance imaging (MRI)? A. Fat B. Melanin C. Methemoglobin D. Cerebral spinal uid

Fig. 81.1

82

Fig. 81.2

3. Which of the follow-up ndings is more suggestive of liposarcoma rather than lipoma? A. Larger size B. Amorphous shape C. Greater amount of soft tissue D. Greater T2 signal 4. A bilobed fat attenuation mass is noted in the interatrial septum with high uorodeoxyglucose (FDG) uptake on positron-emission tomography–computed tomography (PET-CT). What is most likely diagnosis? A. Teratoma B. Liposarcoma C. Lipomatous hypertrophy D. Thymolipoma

Fig. 81.3

Case 82 History: Found down and unresponsive (Figs.82.1 and 82.2). 1. Which of the following should be included in the differential diagnosis for this patient? (Choose all that apply.) A. Extrapleural hematoma B. Pulmonary contusion C. Asbestos pleural thickening D. Hemothorax 2. Which of the following ndings suggest primary lung cancer as the most likely cause of a unilateral apical cap? A. Hilar lymphadenopathy B. Pleural effusion C. Bone destruction D. Mediastinal widening

3. Which one of the following is the most likely to cause bilateral apical caps? A. Trauma associated with mediastinal hematoma B. Radiation for breast carcinoma C. Advancing age D. Head and neck carcinoma 4. Which one of the following is the best next step in the management of this patient? A. Computed tomography (CT) B. Magnetic resonance imaging (MRI) C. Ultrasound D. Fluorodeoxyglucose–positron emission tomography (FDGPET) (FDG-PET)

Fig. 82.2

Fig. 82.1

83

Case 83 History: 35-year-old man with testicular cancer treated with chemotherapy. 1. Which of the following should be included in the differential diagnosis for this patient (Figs. 83.1 and 83.2)? (Choose all that apply.) A. Bleomycin lung toxicity B. Congestive heart failure C. Collagen vascular disease D. Fungal pneumonia 2. Approximately what percent of patients receiving bleomycin develop lung toxicity? A. < 5% B. 15% C. 25% D. 35%

Fig. 83.1

84

3. Which one of the following does not increase the risk of developing bleomycin-related lung toxicity? A. Lymphoma B. Concurrent radiation therapy C. Renal insufciency D. Advanced age 4. Which one of the following regarding treatment of bleomycinrelated lung toxicity is not true? A. The drug must be discontinued. B. Corticosteroids are usually administered. C. Most patients improve within a few days. D. Some patients take up to 2 years to recover.

Fig. 83.2

Case 84 History: Hemoptysis. 1. Which of the following should be included in the differential diagnosis for this patient (Figs.84.1, 84.2 and 84.3)? (Choose all that apply.) A. Lung Cancer B. Metastatic disease C. Silicosis D. Tuberculosis 2. Biopsy showed the left upper lobe mass to be primary lung adenocarcinoma. Which one of the following is the likely N staging of this patient based on the most recent tumor, node metastasis (TNM) staging system? A. N0 B. N1 C. N2 D. N3

3. Which one of the following regarding lymph node metastases is true? A. Ipsilateral axillary lymph node metastases are staged as N3. B. N2 disease precludes surgical resection. C. Fluorodeoxyglucose–positron emission tomography (FDG-PET) is the standard for lymph node staging. D. N2 disease results in a minimal stage of IIIB. 4. Which one of the following ndings is a contraindication for resection of non–small cell lung carcinoma? A. Chest wall invasion B. Tumor metastasis to the same lobe C. Ipsilateral pleural metastasis D. Post obstructive pneumonia involving the entire lung

Fig. 84.2

Fig. 84.1

Fig. 84.3

85

Case 85 History: Swelling of the head and neck. 1. What is the most likely cause of the ndings on the computed tomography (CT) scan (Fig.85.1)? A. Superior vena cava syndrome B. Arteriovenous malformation C. Contrast extravasation D. None of the above 2. What is the most common cause of superior vena cava (SVC) syndrome? A. Long-term intravenous device B. Fibrosing mediastinitis C. Lung cancer D. Radiation therapy

Fig. 85.1

86

3. Which one of the following is not a common clinical manifestation of SVC syndrome? A. Head and face edema B. Extremity edema C. Visual disturbances D. Dural venous sinus thrombosis 4. What is the best initial treatment for patients with non–small cell lung carcinoma who present with acute SVC syndrome? A. Chemotherapy B. Surgical resection C. SVC stenting D. Radiation therapy

Case 86 History: Acute onset dyspnea and hypoxia. 1. Which of the following should be included in the differential diagnosis for this patient (Figs. 86.1 and 86.2)? (Choose all that apply.) A. Pulmonary hemorrhage B. Langerhans cell histiocytosis C. Diffuse pneumonia D. Silicosis 2. What is the most common cause of diffuse alveolar hemorrhage? A. Hemophilia B. Drug toxicity C. Vasculitis D. Infection

3. Which of the following has the greatest association with diffuse alveolar hemorrhage? A. Granulomatosis with polyangiitis B. Microscopic polyangiitis C. Eosinophilic granulomatosis with polyangiitis D. Takayasu arteritis 4. What procedure best conrms the diagnosis of diffuse alveolar hemorrhage? A. Bronchoalveolar lavage B. Transbronchial biopsy C. Surgical biopsy D. Sputum analysis

Fig. 86.2

Fig. 86.1

87

Case 87 History: 72-year-old woman with progressive dyspnea on exertion. 1. Which of the following should be included in the differential diagnosis for this patient (Figs.87.1, 87.2 and 87.3)? (Choose all that apply.) A. Desquamative interstitial pneumonia (DIP) B. Usual interstitial pneumonia (UIP) C. Nonspecic interstitial pneumonia (NSIP) D. Lymphoid interstitial pneumonia (LIP) 2. What is the next best approach to tissue diagnosis? A. Surgical lung biopsy B. Transbronchial biopsy C. Computed tomography (CT) guided needle biopsy D. No tissue sampling needed

3. Which of the following collagen vascular diseases has the highest association with usual interstitial pneumonia? A. Rheumatoid arthritis B. Progressive systemic sclerosis C. Polymyositis D. Sjögren syndrome 4. Which of the following occupational exposures has a strong association with usual interstitial pneumonia? A. Coal B. Silica C. Beryllium D. Asbestos

Fig. 87.1 Fig. 87.3

Fig. 87.2

88

Case 88 History: Chest pain. History of stroke. 1. Which of the following should be included in the differential diagnosis for this patient (Figs. 88.1 and 88.2)? (Choose all that apply.) A. Chest wall metastasis B. Abscess C. Hematoma D. Elastobroma 2. The high-attenuation component of the mass on this computed tomography (CT) scan of a patient on warfarin most likely represents which one of the following? A. Tumor B. Blood C. Calcium D. Fat

Fig. 88.1

3. Which one of the following is not a potential complication of intramuscular hemorrhage? A. Malignant degeneration B. Ischemic myopathy C. Neuropathy D. Pressure necrosis of adjacent bone 4. Which of the following is a common cause of spontaneous intramuscular hematoma? A. Aspirin therapy B. Intravenous (IV) contrast administration C. Vitamin K supplements D. Hemophilia

Fig. 88.2

89

Case 89 History: Occupational screening. Patient is asymptomatic. 1. Which of the following should be included in the differential diagnosis for this patient (Figs. 89.1 and 89.2)? (Choose all that apply.) A. Scimitar syndrome B. Congenital pulmonary airway malformation C. Pulmonary arteriovenous malformation D. Bronchial atresia 2. What type of shunt is most likely present? A. Left-to-left B. Left-to-right C. Right-to-right D. Right-to-left

3. What other ndings are associated with this diagnosis? A. Hypoplastic lung with abnormal airway branching B. Lung cancer C. Kidney agenesis D. Muscular dystrophy 4. Which of the following is the best management for this patient? A. Right lower lobectomy B. Endovascular coiling C. Bronchoscopy D. Observation

Fig. 89.2 Fig. 89.1

90

Case 90 History: Chest pain and pericardial rub on auscultation. 1. Which of the following should be included in the differential diagnosis of pericardial effusion (Figs.90.1, 90.2 and 90.3)? (Choose all that apply.) A. Myocardial infarction B. Systemic lupus erythematosus C. Relapsing polychondritis D. Acute pulmonary thromboembolism 2. The lateral view shows (Figs.90.2) which of the following? A. “Oreo cookie sign” B. Myocardial fat stripe C. Epicardial fat stripe D. A and C

Fig. 90.1

3. Which of the following is most sensitive for detecting a pericardial effusion? A. Chest radiograph B. Transthoracic echocardiogram C. Cardiac magnetic resonance imaging (MRI) D. Transesophageal echocardiogram 4. Which of the following denes Dressler syndrome? A. Rheumatoid arthritis-associated pleural and pericardial effusions B. Viral myocarditis-related pleural and pericardial effusions C. Radiation-induced pleural and pericardial effusions D. Myocardial infarction-related pleural and pericardial effusions

Fig. 90.3

Fig. 90.2

91

Case 91 History: Chronic cough and recurrent pneumonia. 1. Which of the following should be included in the differential diagnosis for this patient (Figs.91.1 and 91.2)? (Choose all that apply.) A. Ciliary dyskinesia B. Allergic bronchopulmonary aspergillosis C. Idiopathic pulmonary brosis D. Lymphangioleiomyomatosis 2. What is the most common cause of this abnormality? A. Congenital condition B. Interstitial pneumonia C. Cigarette smoking D. Infection

3. Which of the following is not a specic sign of bronchiectasis on high-resolution computed tomography (HRCT)? A. Bronchial diameter greater than that of the adjacent artery B. Lack of normal bronchial tapering C. Bronchial wall thickening D. Bronchus visible in peripheral 1 cm of lung 4. Which of the following is not a complication of bronchiectasis? A. Hemoptysis B. Atelectasis C. Pneumonia D. Pleural effusion

Fig. 91.1

Fig. 91.2

92

Case 92 History: Dysphagia. 1. Which of the following should be included in the differential diagnosis for this patient (Fig.92.1)? (Choose all that apply.) A. Esophageal duplication cyst B. Achalasia C. Esophageal diverticulum D. Esophageal carcinoma 2. Which one of the following structures is most likely responsible for displacement of the azygoesophageal contour in this patient? A. Esophagus B. Azygos vein C. Right atrium D. Left atrium

3. Which one of the following imaging studies would be most helpful for further evaluation of this nding? A. Computed tomography (CT) scan B. Magnetic resonance imaging (MRI) C. Esophagram D. Lateral radiograph 4. Which one of the following does not result in a chronically constricted lower esophageal sphincter? A. Progressive systemic sclerosis (scleroderma) B. Achalasia C. Chagas disease D. Pseudoachalasia

Fig. 92.1

93

Case 93 History: Cough. 1. Which of the following should be included in the differential diagnosis for this patient (Figs. 93.1 and 93.2)? (Choose all that apply.) A. Granulomatosis with polyangiitis B. Sarcoidosis C. Coccidioidomycosis D. Septic infarct 2. In which geographic location is Coccidioides endemic? A. New England B. Mississippi River valley C. Pacic Northwest D. American Southwest

3. Which one of the following regarding coccidioidomycosis is true? A. Most affected patients are symptomatic. B. Cavities are typical of chronic infection. C. Disseminated disease is common. D. The pneumonic form requires antifungal therapy. 4. What pattern is most associated with disseminated coccidioidomycosis? A. Multifocal lung consolidation B. Numerous small nodules C. Diffuse ground-glass opacity D. Septic pulmonary infarcts

Fig. 93.2

Fig. 93.1

94

Case 94 History: Dyspnea. 1. Which of the following should be included in the differential diagnosis for this patient (Fig.94.1)? (Choose all that apply.) A. Hydrostatic pulmonary edema B. Usual interstitial pneumonia C. Lymphangitic carcinomatosis D. Sarcoidosis 2. Which one of the following chest radiographic nding correlates with the computed tomography (CT) nding of thickened interlobular septa? A. Kerley B lines B. Thickened ssures C. Bronchovascular bundle thickening D. Lung consolidation

3. Which one of the following are required components of high-resolution computed tomography (HRCT) imaging? A. Thin collimation (≤1.5 mm) B. Prone imaging C. Multiplanar reformations D. Smoothing reconstruction kernel 4. Which of the following is the best next step in management of this patient in whom congestive heart failure is not suspected? A. Fluorodeoxyglucose–positron emission tomography (FDG-PET) B. Repeat chest imaging after diuresis C. Transbronchial biopsy D. Surgical lung biopsy

Fig. 94.1

95

Case 95 History: Chest pain (Figs.95.1 and 95.2). 1. Based on the lateral chest radiograph (Fig. 95.1), which of the following should be included in the differential diagnosis for this patient? (Choose all that apply.) A. Aortic coarctation B. Lymphoma C. Breast carcinoma D. Tuberculosis 2. Which one of the following is the most common site of lymph node metastases in patients with breast carcinoma? A. Internal mammary B. Axillary C. Hilar D. Subcarinal

3. True or false? A normal chest radiograph excludes internal mammary lymphadenopathy. A. True B. False 4. Which type of lymphoma is most commonly associated with neoplastic tissue conned to the anterior mediastinum? A. Classical Hodgkin lymphoma B. Follicular lymphoma C. Diffuse large B cell lymphoma D. Small lymphocytic lymphoma

Fig. 95.2

Fig. 95.1

96

Case 96 History: Lung nodule (Fig.96.1). 1. Which of the following is the greatest risk factor for developing pneumothorax as a result of needle biopsy? A. Older age B. Emphysema C. Lower lobe location D. Subpleural location 2. What is the sensitivity of transthoracic needle biopsy (TTNB) for malignant nodules? A. 50% B. 70% C. 80% D. >90%

3. Which one of the following is the most common complication of TTNB? A. Hemoptysis B. Pneumothorax C. Air embolism D. Seeding of biopsy track 4. Which one of the following improves the diagnostic yield of TTNB for benign lesions? A. Use two needles to increase volume of aspirate. B. Have a cytopathologist on-site. C. Use a cutting needle to obtain core needle biopsy specimens. D. Use CT uoroscopy.

Fig. 96.1

97

Case 97 History: Abnormal chest radiograph. 1. Which of the following should be included in the differential diagnosis for this patient (Figs.97.1 and 97.2)? (Choose all that apply.) A. Carcinoid tumor B. Constrictive bronchiolitis C. Bronchial atresia D. Lung abscess 2. What allows aeration of lung distal to a bronchocele? A. Bronchopleural stula B. Incomplete obstruction C. Anomalous bronchus D. Collateral air ow through adjacent lung

3. What is the most common site of bronchial atresia? A. Left upper lobe B. Left lower lobe C. Right upper lobe D. Right lower lobe 4. Which of the following is the best treatment for bronchial atresia? A. Surgical resection B. Endobronchial stenting C. Observation D. Endobronchial laser ablation

Fig. 97.1

Fig. 97.2

98

Case 98 History: Dyspnea on exertion and intermittent fever. 1. Which of the following should be included in the differential diagnosis for this patient (Figs. 98.1 and 98.2)? (Choose all that apply.) A. Sarcoidosis B. Silicosis C. Hypersensitivity pneumonitis D. Respiratory bronchiolitis 2. What anatomic structures are located in the central core of the pulmonary lobule? A. Pulmonary artery and vein B. Pulmonary artery and bronchiole C. Pulmonary vein and bronchiole D. Pulmonary vein and lymphatics

3. Which of the following is not associated with cigarette smoking? A. Respiratory bronchiolitis B. Pulmonary Langerhans cell histiocytosis C. Desquamative interstitial pneumonia D. Chronic eosinophilic pneumonia 4. Which type of immunologic reaction is most strongly associated with hypersensitivity pneumonitis? A. Type 1 (IgE mediated) B. Type 2 (cytotoxic: IgG or IgM mediated) C. Type 3 (immune complex) D. Type 4 (delayed-type hypersensitivity)

Fig. 98.1 Fig. 98.2

99

Case 99 History: Baseline lung cancer screening. 1. Which of the following should be included in the differential diagnosis for this patient (Fig.99.1)? (Choose all that apply.) A. Lung carcinoma B. Metastasis C. Granuloma D. Streptococcal pneumonia 2. What is the appropriate Lung-RADS category for this patient? A. Lung-RADS 1 B. Lung-RADS 2 C. Lung-RADS 3 D. Lung-RADS 4a

Fig. 99.1

100

3. What is the next best management? A. Computed tomography (CT) follow-up in 6 months B. Fluorodeoxyglucose–positron emission tomography (FDG-PET)/CT C. Transthoracic needle biopsy D. Resection 4. What is the probability that this nodule is malignant? A. 20%

Case 100 History: Stridor and cough. 1. Which of the following should be included in the differential diagnosis for this patient (Figs.100.1 and 100.2)? (Choose all that apply.) A. Double aortic arch B. Previous intubation C. Granulomatosis with polyangiitis D. Mounier-Kuhn syndrome 2. Which of the following is not a risk factor for tracheal stenosis? A. Prolonged intubation B. Tracheostomy C. Balloon cuff overdistention D. Tracheal bronchus

3. Which of the following causes of tracheal stenosis characteristically spares the posterior tracheal wall? A. Relapsing polychondritis B. Inammatory bowel disease C. Granulomatosis with polyangiitis D. Amyloidosis 4. Which of the following is the least common sign or symptom of central airway stenosis? A. Cough B. Stridor C. Wheeze D. Hemoptysis

Fig. 100.1

Fig. 100.2

101

Case 101 History: Fatigue. 1. Based on the computed tomography (CT) image (Fig. 101.1), which of the following should be included in the differential diagnosis for this patient? (Choose all that apply.) A. Thymoma B. Thymolipoma C. Thymic hyperplasia D. Thymic cyst 2. Which of the following CT attenuation values is not typical of normal thymus? A. Soft tissue B. Fat and soft tissue C. Fatty replacement D. Calcication

Fig. 101.1

102

3. Pure red cell aplasia is most commonly associated with which of the following thymic lesions? A. Thymic carcinoma B. Thymoma C. Thymic carcinoid D. Thymolipoma 4. Which imaging examination is most useful for distinguishing thymic hyperplasia from thymic neoplasia? A. Fluorodeoxyglucose–positron emission tomography (FDG-PET)/CT B. Magnetic resonance imaging C. Contrast-enhanced CT D. Gallium scan

Case 102 History: Cough. 1. Which of the following should be included in the differential diagnosis for this patient (Fig.102.1)? (Choose all that apply.) A. Respiratory bronchiolitis B. Focal brosis C. Lung carcinoma D. Histoplasmoma 2. For nodules smaller than 10 mm, which of the following is most likely to be malignant? A. Solid nodule B. Ground-glass nodule C. Solid and ground-glass nodule D. Calcied nodule

3. What type of lung cancer most commonly presents as a ground-glass attenuation or part solid nodule? A. Adenocarcinoma B. Squamous cell carcinoma C. Small cell carcinoma D. Large cell carcinoma 4. Which of the following is the next best step in the follow-up of this patient? A. Fluorodeoxyglucose–positron emission tomography (FDG-PET)/CT B. Follow-up computed tomography (CT) C. Left upper lobectomy D. Bronchoscopy

Fig. 102.1

103

SECTION III

Challenge

Case 103 History: Patient with chronic cough and an abnormality on chest radiography. 1. What should be included in the differential diagnosis of the imaging abnormalities (Figs. 103.1 and 103.2)? (Choose all that apply.) A. Pulmonary edema B. Pneumocystic jirovecii pneumonia C. Diffuse alveolar hemorrhage D. Diffuse alveolar damage E. Pulmonary alveolar proteinosis 2. What is the most likely diagnosis in this case? A. Pulmonary edema B. Pneumocystic jirovecii pneumonia C. Diffuse alveolar hemorrhage

Fig. 103.1

104

D. Diffuse alveolar damage E. Pulmonary alveolar proteinosis 3. What computed tomography (CT) sign is demonstrated? A. Signet ring B. Crazy-paving C. Comet tail D. CT halo E. Reverse CT halo 4. What clinical information would support the diagnosis? A. Female sex B. Intravenous (IV) drug abuse C. Chronicity D. Immunosuppression

Fig. 103.2

Case 104 History: Patient with asthma and an abnormality on chest radiography (Fig.104.1). 1. What imaging sign is demonstrated (Figs.104.1 and 104.2)? A. Signet ring B. Crazy-paving C. Finger-in-glove D. Computed tomography (CT) halo E. Reverse CT halo 2. Which of the following organisms is often implicated in this condition? A. Human papillomavirus type 6 B. Mycobacterium tuberculosis

Fig. 104.1

C. Staphylococcus aureus D. Aspergillus fumigatus 3. Patients with which of the following diseases are predisposed to develop this condition? A. Tracheobronchopathia osteochondroplastica B. Primary lung cancer C. Cystic brosis D. Carcinoid tumor 4. What clinical information would support the diagnosis? A. Recurrent aspiration B. Asthma C. Smoking D. Immunosuppression

Fig. 104.2

105

Case 105 History: 60-year-old woman with cough and dyspnea. 1. What is the salient radiographic abnormality (Fig.105.1)? A. Right-sided pneumothorax B. Left lower lobe pneumonia C. Left upper lobe atelectasis D. Left pneumonectomy 2. What is the most likely cause of the branching low-attenuation opacities within the collapsed left upper lobe (Fig.105.2)? A. Obstruction by tumor B. Bronchial atresia C. Allergic bronchopulmonary aspergillosis D. Cystic brosis

Fig. 105.1

106

3. Which of the following would favor a benign cause for the computed tomography (CT) ndings? A. Smoking history B. Hemoptysis C. Enhancing mass D. Mediastinal inltration E. Macroscopic fat and calcium

Fig. 105.2

Case 106 History: 22-year-old female with abnormality on chest radiograph. 1. In which anatomic compartment is the lesion arising (Figs.106.1 and 106.2)? A. Parenchyma B. Pleura C. Chest wall D. Lymphatics 2. Which of the following imaging features may help determine lesion location? A. Ipsilateral pleural effusion B. Obtuse angles with parenchyma C. Chest wall invasion D. Rib erosion 3. Which of the following extrathoracic manifestations might occur in conjunction with this lesion? A. Digital clubbing B. Hypertrophic osteoarthropathy

Fig. 106.1

C. Episodic hypoglycemia D. Galactorrhea E. All of the above 4. Which of the following reects typical magnetic resonance imaging (MRI) features of small, uncomplicated lesions with this histology? A. Hyperintense signal on T1- and T2-weighted images, no enhancement B. Heterogeneous signal on T1- and T2-weighted images, no enhancement C. Hypointense signal on T1- and T2-weighted images, no enhancement D. Hypointense signal on T1- and T2-weighted images, progressive enhancement

Fig. 106.2

107

Case 107 History: 85-year-old man with weight loss undergoes outpatient evaluation. 1. What is the most likely cause of this radiographic abnormality (Fig.107.1)? A. Right-sided pneumothorax B. Left pleural effusion C. Left lung collapse D. Left pneumonectomy 2. Which of the following would be the most likely cause for this pattern in an intubated patient in the intensive care unit? A. Left-sided infusothorax B. Left lung re-expansion edema C. Left lung hemorrhage D. Left main bronchus mucus plug 3. What term is applied to the preservation of enhancing vessels within the collapsed lung (Figs.107.2 and 107.3)? A. Computed tomography (CT) halo sign B. Air bronchogram sign C. CT angiogram sign D. Juxtaphrenic peak sign 4. Which of the following would raise suspicion for a malignant etiology of the abnormality? (Choose all that apply.) A. Smoking history B. Weight loss C. Age of 85 D. Outpatient setting E. Lymphadenopathy

Fig. 107.1

108

Fig. 107.2

Fig. 107.3

Case 108 History: 79-year-old female never smoker with chronic cough. 1. In which anatomic compartment is the computed tomography (CT) abnormality predominantly located (Fig.108.1)? A. Trachea B. Bronchi C. Parenchyma D. Lymphatics 2. Which organism is most likely responsible for the clinical and imaging abnormalities? A. Inuenza-A B. Mucormycosis

C. Aspergillus fumigatus D. Mycobacterium avium complex (MAC) 3. Which form of this disease is most likely in this case? A. Cavitary (classic) B. Bronchiectatic (non-classic) C. Immunocompromised host D. Nodular E. Hypersensitivity

Fig. 108.1

109

Case 109 History: 47-year-old asymptomatic man. 1. Which of the following best describes the pattern of nodules shown (Figs.109.1 and 109.2)? A. Random, diffuse B. Centrilobular, branching C. Centrilobular, non-branching D. Perilymphatic 2. Which of the following should be included in the differential diagnosis for this pattern? A. Sarcoidosis B. Lymphangitic carcinomatosis C. Silicosis D. Lymphoma E. Miliary tuberculosis

3. Which term is used to describe computed tomography (CT) nding of conuent perilymphatic sarcoid granulomas organized into a focal nodule with surrounding smaller nodules and reticulation? A. Comet tail sign B. Mosaic attenuation C. Reverse CT halo sign D. Galaxy sign 4. Which is the most common clinical presentation of patients with this disease? A. Asymptomatic B. Bird fancier with chronic cough C. Profound weight loss D. History of malignancy

Fig. 109.2

Fig. 109.1

110

Case 110 History: 22-year-old male presents with cough and fever. 1. Which computed tomography (CT) pattern is shown (Fig.110.1)? A. Interstitial B. Centrilobular, nonbranching C. Centrilobular, branching D. Perilymphatic 2. Which anatomic structure(s) may be abnormal? A. Alveolar septae B. Interlobular septae C. Centrilobular arteriole D. Centrilobular bronchiole

3. What is the most common cause of this imaging pattern? A. Infectious bronchiolitis B. Aspiration bronchiolitis C. Diffuse panbronchiolitis D. Excipient lung disease E. Cellulose granulomatosis

Fig. 110.1

111

Case 111 History: 42-year-old male with chronic cough. 1. Which computed tomography (CT) pattern is shown (Fig. 111.1)? (Choose all that apply.) A. Crazy paving B. Mosaic attenuation C. Air trapping D. Tree-in-bud 2. Which anatomic structure may be abnormal? (Choose all that apply.) A. Alveolar septae B. Interlobular septae C. Centrilobular arteriole D. Centrilobular bronchiole 3. What is the most common cause of this imaging pattern? A. Infectious bronchiolitis B. Aspiration bronchiolitis C. Diffuse panbronchiolitis D. Excipient lung disease E. Chronic thromboembolism

Fig. 111.1

112

Case 112 History: 90-year-old male former lightbulb assembly worker with dyspnea. 1. Which indirect sign(s) of volume loss are demonstrated on the radiographs (Figs. 112.1 and 112.2)? (Choose all that apply.) A. Superior hilar displacement B. Increased retrosternal clear space C. Juxtaphrenic peak D. Reticulonodular opacities 2. What is the primary distribution of abnormality? A. Upper lung zone B. Lower lung zone C. Diffuse D. Subpleural 3. Which of the following should be included in the differential diagnosis? (Choose all that apply.) A. Tuberculosis B. Chronic (cluster 2) hypersensitivity pneumonitis C. Sarcoidosis (stage IV) D. Coal worker’s pneumoconiosis E. Silicosis 4. Which descriptor is given for the lymph node calcications (Fig. 112.3)? A. Popcorn B. Toothpaste C. Stippled D. Onion skin E. Egg shell

Fig. 112.1

Fig. 112.2

Fig. 112.3

113

Case 113 History: 56-year-old woman asthmatic patient with fevers and cough (Figs.113.1, 113.2 and 113.3). 1. Which entities may manifest with a peripheral consolidation pattern? (Choose all that apply.) A. Cryptogenic organizing pneumonia B. Chronic eosinophilic pneumonia C. Granulomatosis with polyangiitis D. Pulmonary infarcts E. Simple pulmonary eosinophilia 2. Which cell type is implicated in this disorder? A. Neutrophil B. Erythrocyte C. Langerhans cell D. Eosinophil 3. More than 50% of patients with this disorder will have what associated condition? A. Rheumatoid arthritis B. Takayasu arteritis C. Asthma D. Sarcoidosis 4. What is the most appropriate treatment for the airspace opacities in this condition? A. Doxycycline B. Oseltamivir C. Corticosteroids D. Pembrolizumab E. Montelukast

Fig. 113.1

114

Fig. 113.2

Fig. 113.3

Case 114 History: 77-year-old man with chronic obstructive pulmonary disease (COPD) and incidental nding on computed tomography (CT). 1. What is the salient CT abnormality (Fig. 114.1)? A. Tracheal diverticulum B. Apical paraseptal emphysema C. Tracheal rupture D. Pneumomediastinum 2. What is the most common location of this nding? A. Left paratracheal, T1-T2 B. Left paratracheal, T10-T11 C. Right paratracheal, T1-T2 D. Right paratracheal, T10-T11 3. With which clinical scenario is this nding most commonly associated? A. Recurrent aspiration B. Reduced forced expiratory volume/forced vital capacity (FEV1/FVC) ratio C. Smoking D. Histoplasmosis infection

114.1

115

Case 115 History: 66-year-old man undergoing standard postoperative evaluation after coronary artery bypass. 1. What is the salient radiographic abnormality (Figs. 115.1 and 115.2)? A. Sternal wire fracture B. Sternal wire unraveling C. Sternal wire displacement D. Mediastinal hematoma 2. The detection of “wandering wires” implies what underlying sternotomy complication? A. Sternal hematoma B. Mediastinitis C. Sternal dehiscence D. Sternal non-union

Fig. 115.1

116

3. During which postoperative time period may this complication be detected? A. First week B. First month C. 6 months to 1 year D. Greater than 1 year E. Any 4. Which of the following may increase the risk for this complication? A. Reoperation B. Internal mammary bypass graft C. Diabetes mellitus D. Chronic obstructive pulmonary disease E. All of the above

Fig. 115.2

Case 116 History: 61-year-old male with prior tonsillar abscess and worsening fevers. 1. What is the concerning abnormality depicted on computed tomography (CT) (Figs. 116.1 and 116.2)? A. Malpositioned endotracheal tube B. Malpositioned enteric tube C. Mediastinal fat inammation D. Aortic dissection 2. Which of the following are potential causes for the imaging ndings? (Choose all that apply.) A. Sternal osteomyelitis B. Mediastinal hemorrhage

Fig. 116.1

C. Mediastinal radiation D. Esophageal perforation E. Retropharyngeal infection 3. What is the most likely diagnosis? A. Post-traumatic mediastinal hemorrhage B. Diffuse lymphangiomatosis C. Acute descending mediastinitis, retropharyngeal extension D. Acute mediastinitis after median sternotomy E. Acute mediastinitis after esophageal trauma

Fig. 116.2

117

Case 117 History: 56-year-old otherwise healthy, afebrile woman with cough. 1. What is the distribution of the parenchymal abnormalities in the rst image (Fig. 117.1)? A. Peribronchovascular, focal B. Peripheral, focal C. Peribronchovascular, diffuse D. Peripheral, diffuse 2. What imaging sign is demonstrated in the second image after the patient was treated with corticosteroids (Fig. 117.2)? A. Computed tomography (CT) halo sign B. Reverse CT halo sign C. Air bronchogram sign D. Random

3. What is the most likely diagnosis in this case? A. Bacterial pneumonia B. Aspiration pneumonia C. Pulmonary infarcts D. Cryptogenic organizing pneumonia E. Multifocal mucinous adenocarcinoma 4. What clinical entities may be associated with this imaging pattern? (Choose all that apply.) A. Drug reaction B. Collage vascular disorders C. Vasculitis D. Aspiration pneumonia E. Eosinophilic pneumonia

Fig.117.2

Fig. 117.1

118

Case 118 History: 53-year-old male with history of asthma, restrictive cardiomyopathy, and incidental nding on chest computed tomography (CT) (Fig. 118.1). 1. What of the following entities affect only the cartilaginous portions of the tracheal wall? A. Relapsing polychondritis B. Tracheobronchopathia osteochondroplastica C. Granulomatosis with polyangiitis D. Chondrosarcoma E. Amyloidosis 2. What is a possible complication of this condition? A. Hemoptysis B. Reduced diffusing capacity for carbon monoxide (DLCO) C. Swelling of ears D. Protein-losing enteropathy 3. What is the histopathologic abnormality? A. Submucosal deposition of abnormal protein B. Cartilage inammation and destruction C. Necrotizing granulomatosis vasculitis D. Non-caseating granulomas in airway wall 4. What is the most likely diagnosis in this case? A. Tracheobronchial amyloidosis B. Relapsing polychondritis C. Granulomatosis with polyangiitis D. Adenoid cystic carcinoma E. Endobronchial metastases

Fig. 118.1

119

Case 119 History: 43-year-old otherwise healthy man with recurrent episodes of dyspnea (Figs. 119.1 and 119.2). 1. What should be included in the differential diagnosis for a unilateral hyperlucent hemithorax? (Choose all that apply.) A. Pneumothorax B. Mastectomy C. Endobronchial obstruction D. Bronchial atresia E. Fibrosing mediastinitis 2. What is the most likely underlying pathophysiology of the imaging ndings in this case? A. Autosomal recessive mutation in cystic brosis transmembrane conductance regulator (CFTR) gene B. X-linked agammaglobulinemia C. Constrictive bronchiolitis, post-infectious D. Constrictive bronchiolitis, transplant-related 3. Which feature of clinical history is typically associated with these ndings? A. Acute chest pain B. Childhood viral bronchiolitis C. Chest wall radiation D. Inhabitation of Ohio River Valley 4. By which eponym is this condition known? A. Poland syndrome B. Swyer-James-MacLeod syndrome C. Westermark D. Williams-Campbell

Fig. 119.2

120

Fig. 119.1

Case 120 History: 53-year-old woman with remote history of hysterectomy. 1. Based on the chest radiographic ndings (Fig. 120.1), what is the distribution pattern of the pulmonary nodules? A. Peripheral predominant B. Upper lung zone predominant C. Lower lung zone predominant D. Central predominant 2. What is the anatomic distribution of nodules on the corresponding computed tomography (CT) (Fig. 120.2)? A. Centrilobular, branching B. Centrilobular, non-branching

Fig. 120.1

C. Perilymphatic D. Random 3. What is the most common cause of this imaging pattern? A. Hematogenous metastases B. Lymphatic metastases C. Aspiration bronchiolitis D. Respiratory bronchiolitis E. Excipient lung disease

Fig. 120.2

121

Case 121 History: 68-year-old man with crackles on auscultation (Figs. 121.1 and 121.2). 1. What is the best diagnosis? A. Idiopathic pulmonary brosis B. Asbestosis C. Hypersensitivity pneumonitis (HP) D. Sarcoidosis 2. What proportion of usual interstitial pneumonia (UIP) cases can be condently diagnosed solely on chest computed tomography (CT)? A. 20% B. 30% C. 50% D. 70%

Fig. 121.1

122

3. What is the most helpful ndings in differentiating brotic HP from UIP and nonspecic interstitial pneumonitis (NSIP)? A. Subpleural sparing B. Basilar sparing C. Reticulation D. Traction bronchiectasis 4. Which of the following features occurs with equal frequency in chronic beryllium disease and sarcoidosis? A. Isolated lymphadenopathy B. Pulmonary nodularity C. Pulmonary brosis D. Airway involvement

Fig. 121.2

Case 122 History: 45-year-old woman with chronic cough and sputum production. 1. What is the best diagnosis (Figs.122.1 and 122.2)? A. Mounier-Kuhn syndrome B. Tracheomalacia C. Tracheobronchopathia osteochondroplastica D. Relapsing polychondritis 2. Which of the following is not a common nding in MounierKuhn syndrome? A. Bronchiectasis B. Tracheal diverticula C. Hyperination D. Nodules 3. What is the treatment for tracheobronchopathia osteochondroplastica? A. Surgical resection B. Laser ablation C. Immunotherapy D. No treatment 4. What portion of the tracheal wall is free of cartilaginous support? A. Anterior wall B. Lateral walls C. Posterior wall D. Trachea has circumferential cartilaginous rings

Fig. 122.2

Fig. 122.1

123

Case 123 History: 59-year-old woman with chronic dyspnea and history of “asthma.” 1. What is the best diagnosis (Figs.123.1 and 123.2)? A. Saber sheath trachea B. Tracheomalacia C. Relapsing polychondritis D. Tracheal rupture 2. During which phase of the respiratory cycle is tracheomalacia most evident? A. Dynamic inspiration B. Dynamic expiration C. End-inspiration D. End-expiration

3. If the tracheal wall is thickened circumferentially but the posterior wall is spared what other condition (other than relapsing polychondritis) should be considered in the differential diagnosis? A. Amyloidosis B. Tuberculosis C. Granulomatosis with polyangiitis D. Tracheobronchopathia osteochondroplastica 4. What is the name ascribed to a trachea in which the transverse diameter is wider than the anterior-posterior diameter? A. Bowed trachea B. Parabolic trachea C. Lunate trachea D. Quadrilateral trachea

Fig. 123.2 Fig. 123.1

124

Case 124 History: Middle-aged man with shortness of breath and wheezing. 1. Which of the following diagnoses should be included on the differential diagnosis (Figs.124.1 and 124.2)? (Choose all that apply.) A. Metastasis B. Squamous cell carcinoma C. Adenoid cystic carcinoma D. Leiomyoma 2. What degree of tracheal narrowing is typically present before airway symptoms develop in primary tracheal malignancy? A. 25% B. 50% C. 75% D. 90%

3. Which of the listed primary malignancies is most likely to spread hematogenously to the trachea? A. Melanoma B. Testicular cancer C. Thyroid cancer D. Pancreatic cancer 4. In addition to lipomas, what benign airway tumor commonly contains fat? A. Leiomyoma B. Hamartoma C. Adenoma D. Papilloma

Fig. 124.2

Fig. 124.1

125

Case 125 History: 53-year-old man with history of heart transplant with new fever (Fig.125.1). 1. Which of the following diagnoses infectious etiologies are most likely in this setting? A. Histoplasmosis B. Nocardiosis C. Tuberculosis D. Aspergillosis 2. What is the most common cause of death in the rst 30 days after cardiac transplant? A. Graft failure B. Infection C. Acute allograft rejection D. Cardiac allograft vasculopathy 3. Which of the following entities is least likely to manifest with a tree-in-bud pattern on computed tomography (CT)? A. Acute tuberculosis B. Invasive aspergillosis C. Chronic aspiration D. Cystic brosis

Fig. 125.1

126

Case 126 History: 69-year-old man with chronic shortness of breath (Figs.126.1 and 126.2). 1. What is the most likely diagnosis? A. Proximal interruption of the pulmonary artery B. Pulmonary artery sarcoma C. Primary pulmonary hypertension D. Chronic thromboembolic disease 2. Which of the following is the likely cause of mosaic attenuation? A. Air-trapping B. Left-sided heart failure C. Vasculitis D. Infection

3. Which of the following computed tomography (CT) ndings is specic for chronic as opposed to acute thromboembolic disease? A. Mosaic attenuation B. Ground-glass opacity C. Central arterial lling defect D. Decreased arterial caliber 4. What pulmonary nding is invariably present in long-standing proximal interruption of pulmonary artery? A. Mosaic attenuation B. Consolidation C. Bronchiectasis D. Fibrosis/scarring

Fig. 126.1

Fig. 126.2

127

Case 127 History: Middle-aged man with shortness of breath (Figs.127.1 and 127.2). 1. What is the most likely diagnosis? A. Sarcoidosis B. Silicosis C. Langerhans cell histiocytosis D. Metastatic disease 2. Which of the following disease processes most commonly present with a random pattern of diffuse nodular lung disease on computed tomography (CT)? A. Disseminated infection B. Silicosis C. Hypersensitivity pneumonitis D. Chronic beryllium disease

Fig. 127.1

128

3. What is the typical zonal distribution for silicosis? A. Upper B. Mid C. Lower D. Diffuse 4. A woman with an occupational history of nuclear reactor manufacturing presents with chronic shortness of breath. What is the next step in management? A. Puried protein derivative (PPD) skin test B. Chest CT C. Beryllium lymphocyte proliferation test (BeLPT) D. Ventilation/perfusion scan

Fig. 127.2

Case 128 History: 35-year-old woman with an incidental computed tomography (CT) nding (Fig.128.1). 1. What should be included in the differential diagnosis? A. Sarcoidosis B. Tuberculosis C. Castleman’s disease D. Hypervascular metastasis 2. Lymph node metastases from which of the following primary malignancies are typically not hypervascular? A. Melanoma B. Renal cell carcinoma C. Colon cancer D. Thyroid cancer 3. Which of the following is not common in the plasma cell variant of Castleman’s disease? A. Avid nodal enhancement B. Systemic symptoms C. Multicentric involvement D. Progression to lymphoma 4. Which chronic infection predisposes to the development of human herpesvirus 8 (HHV-8) associated Castleman’s disease? A. Tuberculosis B. Human immunodeciency virus (HIV) C. Hydatid disease D. Nocardiosis Fig. 128.1

129

Case 129 History: 37-year-old man with chest tightness and dyspnea. 1. What is the diagnosis (Figs.129.1 and 129.2)? A. Penetrating aortic ulceration B. Intramural hematoma C. Aortic dissection D. Motion artifact 2. What is the next step in management? A. Surgery B. Watchful waiting C. Blood pressure control D. B and C 3. Which portion of the aorta is most commonly affected by penetrating aortic ulcerations? A. Sinus of Valsalva B. Ascending thoracic aorta

Fig. 129.1

130

C. Aortic arch D. Descending thoracic aorta 4. In the setting of ascending aortic intramural hematoma, into which other vessel wall can hematoma extend? A. Right pulmonary artery B. Superior vena cava C. Left pulmonary vein D. Azygous vein

Fig. 129.2

Case 130 History: 53-year-old woman with chronic dyspnea. 1. What is the best diagnosis (Fig.130.1)? A. Granulomatosis with polyangiitis B. Relapsing polychondritis C. Sarcoidosis D. Amyloidosis 2. What portion of the central airways is usually involved in the setting of granulomatosis with polyangiitis? A. Subglottic B. Carina C. Mid-trachea D. Proximal bronchi 3. Which of the following can present with nodular hyperdense foci within the tracheal walls? A. Sarcoidosis B. Relapsing polychondritis C. Amyloidosis D. Granulomatosis with polyangiitis 4. Which of the following conditions can lead to diffuse tracheal dilation? A. Emphysema B. Asthma C. Pulmonary brosis D. Pleural thickening

Fig. 130.1

131

Case 131 History: 23-year-old man with shortness of breath and chronic cough. History of severe childhood pulmonary infection. 1. What of the following should be included in the differential diagnosis (Figs.131.1 and 132.2)? A. Congenital lobar emphysema B. Swyer-James-Macleod syndrome C. Bronchial atresia D. Obliterative bronchiolitis 2. Hyperdense bronchial mucus plugging is typical of which condition? A. Bronchial atresia B. Obliterative bronchiolitis C. Williams-Campbell syndrome D. Allergic bronchopulmonary aspergillosis 3. Computed tomography (CT) performed in expiration is characterized by inward bowing of which? A. Posterior wall of the trachea B. Anterior wall of the trachea

C. Posterior wall of the bronchus intermedius D. Anterior wall of the bronchus intermedius 4. In an older female patient with long history of asthma, a chest CT showed severe air-trapping and multiple, well-dened pulmonary nodules. Biopsy of the dominant nodule was diagnostic of a well-differentiated carcinoid tumor. What is the best diagnosis? A. Granulomatous lymphocytic interstitial lung disease B. Carney complex C. Erdheim-Chester disease D. Diffuse idiopathic pulmonary neuroendocrine cell hyperplasia

Fig. 131.2

Fig. 131.1

132

Case 132 History: Middle-aged man with chronic shortness of breath. 1. What is the correct diagnosis (Figs.132.1 and 132.2)? A. Tracheal bronchus B. Bridging bronchus C. Bronchial atresia D. Cardiac bronchus 2. Which condition is associated with a bridging bronchus? A. Right pulmonary artery atresia B. Interruption of the right pulmonary artery C. Pulmonary artery sling D. Patent ductus arteriosus

3. Which lobe is most commonly affected by bronchial atresia? A. Left upper lobe B. Right upper lobe C. Left lower lobe D. Right lower lobe 4. In the setting of a tracheal bronchus (arising from the distal trachea), an endotracheal tube position close to the carina would most likely lead to atelectasis of which pulmonary lobe? A. Left upper lobe B. Right upper lobe C. Left lower lobe D. Right lower lobe

Fig. 132.2

Fig. 132.1

133

Case 133 History: Young adult with recurrent pneumonia. 1. What is the most likely diagnosis (Figs.133.1 and 133.2)? A. Pulmonary sequestration B. Congenital pulmonary airway malformation C. Bronchogenic cyst D. Pneumatocele 2. Patients with extralobar sequestrations typically present at which age? A. Infancy B. Adolescence C. 20 to 30 years of age D. 30 to 40 years of age

3. What is the typical treatment for congenital pulmonary airway malformations that have come to clinical attention? A. Long-term antibiotics B. Surgical resection C. Annual imaging surveillance D. No treatment 4. Which pulmonary lobe is most commonly affected by bronchogenic cysts? A. Upper lobes B. Right middle lobe/lingula C. Lower lobes D. All lobes are equally affected

Fig. 133.2 Fig. 133.1

134

Case 134 History: Dyspnea. 1. Which of the following should be included in the differential diagnosis for this patient (Figs.134.1 and 134.2)? (Choose all that apply.) A. Acute pulmonary embolism B. Flow artifact C. Pulmonary artery sarcoma D. Pulmonary artery pseudoaneurysm 2. Which other important nding is present? A. Pulmonary infarct B. Pulmonary hemorrhage C. Right heart strain D. Aortic dissection

Fig. 134.1

3. What is the radiographic sign used to describe oligemia distal to an obstructing embolus? A. S sign of Golden B. Hampton hump C. Fleischner sign D. Westermark sign 4. What nding on magnetic resonance imaging (MRI) best distinguishes pulmonary thromboembolism from pulmonary artery sarcoma? A. High T2 signal intensity B. Enhancement with gadolinium C. Shape of lling defect D. High T1 signal intensity

Fig. 134.2

135

Case 135 History: Dyspnea. 1. Which of the following should be included in the differential diagnosis for this patient (Figs. 135.1, 135.2 and 135.3)? (Choose all that apply.) A. Coal-worker’s pneumoconiosis B. Alpha-1 Antitrypsin deciency C. Intravenous drug abuse D. Nontuberculous mycobacterial infection 2. What type of emphysema predominates in this patient? A. Centrilobular B. Paraseptal C. Panlobular D. Paracicatricial 3. What is the pattern of inheritance of alpha-1 antitrypsin deciency? A. Autosomal dominant B. Autosomal recessive C. X-linked dominant D. X-linked recessive 4. Which of the following is another established complication of alpha-1 antitrypsin deciency? A. Liver cirrhosis B. Pancreatitis C. Renal cell carcinoma D. Aortic aneurysm

Fig. 135.2

Fig. 135.1

Fig. 135.3

136

Case 136 History: Abnormal chest radiograph. 1. Based on the computed tomography (CT) ndings (Fig.136.1), which of the following should be included in the differential diagnosis for this patient. (Choose all that apply.) A. Thymoma B. Lymphoma C. Thymic cyst D. Normal thymus 2. Which of the following is not a common cause of mixed solid and cystic anterior mediastinal mass? A. Thymoma B. Germ cell neoplasm C. Hodgkin lymphoma D. Pericardial cyst

3. Which of the following is the most common cause of a thymic mass? A. Thymic cyst B. Thymolipoma C. Thymic carcinoid D. Thymoma 4. If a thymic cyst has been complicated by hemorrhage, how would it most likely appear on T1-weighted magnetic resonance imaging (MRI)? A. Low T1 signal intensity B. High T1 signal intensity C. No T1 signal intensity D. Intermediate T1 signal intensity

Fig. 136.1

137

Case 137 History: Lung nodule. 1. What congenital abnormality is present (Figs.137.1 and 137.2)? A. Pulmonary sling B. Tracheal bronchus C. Cardiac bronchus D. Complete tracheal rings 2. What portion of the lung does a tracheal bronchus usually supply? A. Right upper lobe B. Right middle lobe C. Left upper lobe D. Left lower lobe

Fig. 137.1

138

3. What is the most common clinical presentation of a tracheal bronchus? A. Recurrent infection B. Hemoptysis C. Stridor D. Incidental nding 4. What term describes a tracheal bronchus supplying the entire right upper lobe? A. Cow bronchus B. Pig bronchus C. Sheep bronchus D. Horse bronchus

Fig. 137.2

Case 138 History: Dyspnea, heart disease. 1. Which of the following should be included in the differential diagnosis for this patient (Figs. 138.1, 138.2 and 138.3)? (Choose all that apply.) A. Amiodarone toxicity B. Renal cell carcinoma C. Talcosis D. Bleomycin toxicity 2. Considering the high-attenuation liver parenchyma, what is the most likely cause of these nodules? A. Amiodarone toxicity B. Renal cell carcinoma C. Talcosis D. Bleomycin toxicity

Fig. 138.1

3. What disorder is amiodarone used to treat? A. Acute respiratory distress syndrome (ARDS) B. Cardiac dysrhythmia C. Myocarditis D. Congestive heart failure 4. Approximately what percentage of patients treated with amiodarone develop pulmonary toxicity? A. 75%

Fig. 138.3

Fig. 138.2

139

Case 139 History: Chest pain. 1. Which of the following should be included in the differential diagnosis for this patient (Fig.139.1)? (Choose all that apply.) A. Lymphoma B. Aneurysm C. Pseudoaneurysm D. Thymoma 2. What is the most severe complication of this abnormality? A. Rupture B. Myocardial ischemia C. Fistula formation D. Compression of the superior vena cava (SVC)

Fig. 139.1

140

3. Approximately what percent of patients with venous graft aneurysms contain thrombus? A. 5% B. 15% C. 25% D. 50% 4. What is the best next step in the management of this patient? A. Follow-up radiograph in 3 months B. Cardiac magnetic resonance imaging (MRI) C. Surgical repair D. Coil embolization

Case 140 History: Cough and fever. 1. Which of the following should be included in the differential diagnosis for this patient (Figs.140.1 and 140.2)? (Choose all that apply.) A. Mycoplasma infection B. Staphylococcus aureus infection C. Respiratory syncytial virus (RSV) infection D. Streptococcus pneumoniae infection 2. Which of the following terms describes the nodular and linear branching centrilobular opacities due to small airways disease? A. Acinar nodule B. Tree-in-bud opacity C. Ring shadow D. Mosaic attenuation

3. Which of the following regarding bronchioles is true? A. Bronchioles contain cartilage. B. Normal bronchioles are visible on computed tomography (CT). C. Respiratory bronchioles communicate directly with alveoli. D. Terminal bronchioles participate in gas exchange. 4. Where in the pulmonary lobule are bronchioles located? A. Center B. Interlobular septa C. Subpleural interstitium D. Adjacent to the pulmonary vein

Fig. 140.1

Fig. 140.2

141

Case 141 History: Lung transplantation 2 months ago. Low-grade fever and nonproductive cough. 1. Which of the following should be included in the differential diagnosis for this patient (Figs.141.1 and 141.2)? (Choose all that apply.) A. Tuberculosis B. Drug reaction C. Viral pneumonia. D. Post-transplant lymphoproliferative disorder (PTLD) 2. True or false? Pneumocystis jiroveci pneumonia a common opportunistic infection in transplant recipients. A. True B. False 3. Which of the following is the most common viral pneumonia in solid organ transplant recipients? A. Epstein-Barr virus (EBV) B. Cytomegalovirus (CMV) pneumonia

Fig. 141.1

142

C. Respiratory syncytial virus (RSV) pneumonia D. Adenovirus pneumonia 4. Which of the following regarding renal transplantation is not true? A. Opportunistic infections are uncommon during the rst month after transplant. B. Streptococcus pneumoniae is a common cause of pneumonia after the sixth month following transplant. C. T-cell mediated immunity is most severely depressed during the second through sixth months after transplant. D. Fungal pneumonia is uncommon after the sixth month following transplant.

Fig. 141.2

Case 142 History: Severe pulmonary hypertension. 1. Which of the following should be included in the differential diagnosis for this patient (Figs. 142.1, 142.2 and 142.3)? (Choose all that apply.) A. Diffuse alveolar hemorrhage B. Usual interstitial pneumonia C. Pulmonary veno-occlusive disease (PVOD) D. Noncardiogenic pulmonary edema 2. What is the most likely diagnosis based on the ndings of severe pulmonary hypertension and pulmonary edema? A. PVOD B. Chronic pulmonary thromboembolic disease C. Chronic obstructive pulmonary disease (COPD) D. Primary pulmonary hypertension 3. Which one of the following is true regarding PVOD? A. Prognosis is excellent. B. Most patients are asymptomatic at diagnosis.

C. Venous and venule obstruction results from intimal brosis. D. Central pulmonary veins are dilated on computed tomography (CT). 4. Which of the following is the best next step in the management of this patient with suspected PVOD? A. Cardiac magnetic resonance imaging (MRI) B. Surgical biopsy C. Bronchoalveolar lavage (BAL) D. Fluorodeoxyglucose–positron emission tomography (FDG-PET)/CT

Fig. 142.2 Fig. 142.1

Fig. 142.3

143

Case 143 History: Cough. 1. Which of the following should be included in the differential diagnosis for this patient (Figs.143.1 and 143.2)? (Choose all that apply.) A. Tracheobronchial papillomatosis B. Relapsing polychondritis C. Bacterial tracheitis D. Amyloidosis 2. How can magnetic resonance imaging (MRI) help distinguish amyloid from other causes of tracheal masses? A. Amyloid has high signal intensity on both T1- and T2-weighted images. B. Amyloid has low signal intensity on both T1- and T2-weighted images. C. Amyloid has low signal intensity on T1- and high signal intensity on T2-weighted images. D. Amyloid has high signal intensity on T1- and low signal intensity on T2-weighted images.

Fig. 143.1

144

3. Which virus is most associated with this condition? A. Human immunodeciency virus B. Human papilloma virus (HPV) C. Epstein-Barr virus (EBV) D. Cytomegalovirus (CMV) 4. Which of the following is not a complication of respiratory papillomatosis? A. Life-threatening airway obstruction B. Hemoptysis C. Adenocarcinoma D. Recurrent infection

Fig. 143.2

Case 144 History: Abnormal chest radiograph. Cough. 1. What infection is associated with a mass within a cyst, as shown in the gure (Figs.144.1 and 144.2)? A. Aspergillosis B. Nocardiosis C. Echinococcosis D. Mucormycosis 2. Which term is used to describe air collection within the cyst in the rst gure? A. Crescent sign B. Water lily sign C. Tip-of-the-iceberg sign D. Collar sign

3. What is the signicance of the crescent sign in the rst gure? A. Secondary infection B. Impending rupture C. Healing D. Hemorrhage 4. Which of the following is the most common site of hydatid cysts in humans? A. Lung B. Brain C. Liver D. Spleen

Fig. 144.1 Fig. 144.2

145

Case 145 History: Cough. 1. Which of the following should be included in the differential diagnosis for this patient (Figs.145.1 and 145.2)? (Choose all that apply.) A. Lung abscess B. Lymphangioleiomyomatosis C. Congenital pulmonary airway malformation D. Bronchogenic cyst 2. Which of the following congenital lung lesions is characterized by systemic arterial supply? A. Bronchogenic cyst B. Congenital pulmonary airway malformation C. Pulmonary sequestration D. Congenital lobar overination

Fig. 145.1

146

3. What is the most common location of a bronchogenic cyst? A. Right lower lobe B. Left lower lobe C. Anterior mediastinum D. Middle mediastinum 4. Patients with congenital pulmonary airway malformations are at increased risk for developing which of the following tumors? A. Lymphoma B. Pleuropulmonary blastoma C. Adenocarcinoma D. Small cell lung carcinoma

Fig. 145.2

Case 146 History: Xerostomia and facial swelling. 1. Which of the following should be included in the differential diagnosis for this patient (Figs.146.1 and 146.2)? (Choose all that apply.) A. Usual interstitial pneumonia (UIP) B. Lymphangioleiomyomatosis C. Pulmonary Langerhans cell histiocytosis D. Lymphocytic interstitial pneumonia 2. Lymphocytic interstitial pneumonia is most commonly associated with which of the following? A. Systemic lupus erythematosus B. Sjögren syndrome C. Idiopathic pulmonary brosis D. Acute interstitial pneumonia

Fig. 146.1

3. What is the predominant computed tomography (CT) nding of Pneumocystis jirovecii pneumonia? A. Ground-glass opacity B. Consolidation C. Lung cysts D. Lymphadenopathy 4. Which of the following is not associated with lung cysts? A. Lymphangioleiomyomatosis B. Pulmonary Langerhans cell histiocytosis C. Respiratory bronchiolitis–associated interstitial lung disease D. Birt-Hogg-Dubé syndrome

Fig. 146.2

147

SECTION IV

Answers

Opening Round CASE 1

Imaging Findings

Squamous Cell Carcinoma of the Lung

PA and lateral hest radigraphs (Fig.1.1, Fig.1.2) shw a right lwer lbe mass. The mass and right hilar lymphadenpathy were nrmed n CT (Fig. 1.3, Fig. 1.4). Primary lung arinma mst mmnly presents as a slitary lung ndule r mass. Assiated hilar and mediastinal lymphadenpathy may be apparent n hest radigraphs, althugh CT is superir t radigraphy fr deteting thrai lymphadenpathy. Other assiated ndings an inlude bne destrutin (hest wall invasin r metastasis), pleural effusin, lung metastases, and septal thikening frm lymphangiti arinmatsis. Patients with radigraphs suggestive f lung arinma shuld underg hest CT fr further evaluatin. Flurdexygluse–psitrn emissin tmgraphy/CT (FDG-PET/CT) is used t stage patients with lung aner and is partiularly valuable in deteting ult metastases. Suh upstaging f lung aner urs frequently after PET imaging and an ften alter therapy. Beause f the intrinsially high level f FDG uptake in the brain, magneti resnane imaging (MRI) is used t evaluate fr brain metastasis in patients with lung aner.

1. A and B. The mst mmn auses f a slitary lung ndule r mass are neplasms and infetin. The mst likely neplasti ause is primary lung arinma. Infetins that present as large ndules r masses inlude lung absesses, nardisis, atinmysis, and fungal infetin suh as blastmysis r invasive aspergillsis. Oasinally, mmunity-aquired baterial infetins an present as a rund mass during the early stages f infetin (rund pneumnia). Althugh pulmnary sequestratins an present as medially lated lwer lbe masses n hest radigraphy, lymphadenpathy is nt a typial nding unless the sequestratin is infeted. 2. A. Adenarinma is the mst mmn histpathlgi type f lung aner. Squamus ell arinma and small ell arinma are smewhat less mmn. Carinid aunts fr apprximately nly 1% f all lung malignanies. 3. C. Cigarette smking is the signal largest risk fatr fr develping lung arinma. Other risk fatrs inlude upatinal and envirnmental expsures suh as asbests and silia. Radn may be a fatr ntributing t lung aner in sme patients. Obesity has nt been shwn t be an independent risk fatr fr lung arinma. 4. D. Stage at diagnsis is the best preditr f 5-year survival with lung arinma. The urrent staging system fllws the T(umr), N(de), M(etastasis) mdel and desribes lal, reginal, and systemi extent f malignany.

Comment Differential Diagnosis Primary lung arinma is the leading diagnsti nsideratin. Other entities t nsider inlude infetins partiularly fungal r baterial nditins suh as atinmysis r nardisis.

Lung Cancer Lung aner remains the leading ause f aner-related death in bth males and females. Adenarinma is the mst mmn histpathlgi type f lung aner, fllwed by squamus ell and small ell arinma. Other histpathlgi types are muh less mmn. Cigarette smking remains the leading ause f lung aner, as 85% f lung aners develp in urrent r frmer smkers. Mst lung aners are deteted at later stages when patients beme symptmati. Chest radigraphy has nt been shwn t redue mrtality when used as a sreening test in patients at risk f lung aner. Hwever, the Natinal Lung Sreening Trial shwed a 20% redutin in all-ause mrtality with lw-dse mputed tmgraphy (CT) sreening in patients deemed at highest risk fr develping lung aner. Earlier stage lung aner is assiated with higher survival rates.

148

REFERENCES

Maldnad F, Jett JR. Invasive and nninvasive advanes in the staging f lung aner. Semin Oncol. 2014 Feb;41(1):17–27. Thrai Radilgy: The Requisites, 3rd ed, 405–427.

CASE 2 Spontaneous Pneumothorax 1. C. Playing basketball des nt invlve sufient trauma t prdue a traumati pneumatele. There is n radigraphi evidene f pneummediastinum. Spntaneus pneumthrax typially urs in healthy yung individuals, ften during athleti ativities. Swyer-James-MLed syndrme is haraterized n radigraphy by a hyperluent lung with slightly dereased lung vlume and a small ipsilateral hilum. 2. B. A traumati pneumatele results frm mpressin-dempressin trauma f the hest during partial airway lsure, ausing rupture f small airways. 3. B. Swyer-James-MLed syndrme results frm pstinfetius nstritive brnhilitis, mst mmnly a result f hildhd infetin that is usually viral. There is redued bld w t the affeted lung and air trapping n expiratry imaging, whih manifests as unilateral hyperlueny f the affeted hemithrax. 4. D. Tensin pneumthrax manifests with harateristi displaements that result frm inreased intrathrai pressure in the affeted hemithrax, shifting the mediastinum tward the ppsite side, displaing the ipsilateral hemidiaphragm inferirly, and widening the ipsilateral interstal spaes.

Opening Round

Comment Differential Diagnosis Unilateral hyperluent lung (hemithrax) may result frm pneumthrax, Swyer-James-MLed syndrme, endbrnhial tumr r freign bdy, mastetmy, r Pland syndrme.

Discussion PA hest radigraph (Fig. 2.1 and 2.2) shws abnrmal lueny in the left hemithrax, with ateletasis f the left lung, shift f mediastinal strutures t the right, dwnward displaement f the left hemidiaphragm, and widening f left interstal spaes. Pneumthrax is dened as the presene f air r gas in the pleural spae. Althugh there is a wide variety f auses, spntaneus pneumthrax is the mst mmn etilgy. Affeted patients are typially in their third r furth deade f life. Spntaneus pneumthraes are almst always sendary t rupture f an apial bleb, whih represents an air pket within the elasti bers f the viseral pleura. Suh blebs have been reprted t be detetable n hest radigraphs in apprximately 15% f ases f spntaneus pneumthrax. Hwever, blebs are rarely evident n hest radigraphs fllwing reslutin f the pneumthrax. Cmputed tmgraphy (CT) is muh mre sensitive than radigraphy fr deteting blebs and has been shwn t detet blebs in apprximately 80% f patients fllwing reslutin f spntaneus pneumthraes. The size and number f apial blebs deteted n CT have been shwn t rrelate with the risk fr reurrent pneumthraes and the need fr surgial interventin. The ipsilateral and ntralateral rates f reurrene f spntaneus pneumthrax is apprximately 30% and 10%, respetively. Tensin pneumthrax is a life-threatening nditin. Affeted patients present with linial signs f tahypnea, tahyardia, yansis, sweating, and hyptensin. Radigraphi ndings may inlude ntralateral mediastinal shift, dwnward displaement f the ipsilateral hemidiaphragm, widening f interstal spaes. In severe ases, there may be attening f the nturs f the right heart brder and/r vena avae. REFERENCES

Sahn SA, Heffner JE. Spntaneus pneumthrax. N Engl J Med. 2000;342:868–874. Thrai Radilgy: The Requisites, 3rd ed, 159–192.

CASE 3 Pleural Plaques 1. C. Based n the harateristi radigraphi and mputed tmgraphy (CT) ndings, the mst likely diagnsis is C, asbests-related pleural plaques. Fibrthrax is typially a unilateral press and the pleural thikening and aliatin are harateristially ntinuus rather than disntinuus, and ften invlve and bsure the ardiphreni sulus. Tal pleurdesis may be perfrmed t treat intratable pleural effusin r hrni pneumthrax and ften results in fal r multifal areas f smth r ndular pleural thikening that ften exhibit high attenuatin r aliatin. The ndings typially invlve the psterir and basilar aspets f the treated hemithrax. Calied pleural metastases are unmmn but d ur in patients with stesarmatus malignanies and may mimi pleural plaques. 2. C. Apprximately 25% f patients with the primary pleural malignant mesthelima will als exhibit pleural plaques, an imaging marker f their previus asbests expsure.

149

3. D. The harateristi lateny perid is 30 t 40 years between the time f a persn’s upatinal expsure t asbests and the develpment f mesthelima. 4. E. All of the above. Asbests was ne an ingredient in many different nstrutin materials. In the mid-1960s, sientists nrmed that breathing airbrne asbests bers an ause a variety f lung diseases, inluding mesthelima, lung aner and asbestsis. Many buildings built befre 1980 were nstruted with asbests-ntaining materials inluding rfing materials, ement, r and eiling tiles, pipe and dut insulatin. Asbests was ne nsidered a ritial element in the shipbuilding industry, espeially in the military, beause it uld resist heat and prevent res that wuld be disastrus fr a vessel at sea. It was als regnized as a great insulatr and was resistant t rrsin. Many f the gds that were prdued and pressed in textile mills frm the 1940s t the 1970s were made with asbests bers, whih put many textile wrkers at risk f asbests expsure.

Comment Differential Diagnosis Pleural thikening resulting frm infetin, primary r sendary neplasia, r pleurdesis. Pleural plaques are the mst mmn manifestatin f asbests expsure and typially ur after a lateny perid f apprximately 15 t 20 years. They d nt ause symptms and are usually disvered inidentally, as in this ase. Pathlgially, pleural plaques are mpsed f dense bands f avasular llagen. Pleural plaques may be deteted in patients whse upatin r ther ativities invlved expsure t asbests bers (e.g., nstrutin, bat building). They are typially fund in hest radigraphi and CT examinatins perfrmed 15 t 20 years after suh expsure and have harateristi imaging features. Pleural plaques are nt premalignant lesins, but affeted individuals are at inreased risk fr lung aner, mesthelima, and asbestsis (pulmnary brsis) due t their asbests expsure. Plaques serve as a signpst f asbests expsure. On hest imaging, detetin f bilateral alied pleural plaques alng the diaphragmati pleura (S3.1 and S3.2) is nsidered diagnsti f asbests-related pleural disease. They manifest as bilateral multifal areas f disntinuus pleural thikening, with r withut aliatin, and typially ur adjaent t the 6th t 9th ribs, alng the diaphragmati pleura, and alng the right lwer paravertebral pleura (Fig. 3.1). When mre extensive, the larger plaques may have a distintive en face radigraphi appearane resembling the fringes f a hlly leaf. Pleural plaques may als exhibit inmplete brders n hest radigraphy, an imaging feature that ften urs in pleural and hest wall lesins. REFERENCES

Jamrzik E, etal. Clinial Review: Asbests-related Disease. Intern Med J. 2011;Feb 10 https://di.rg/10.1111/j.1445-5994. 2011. Thrai Radilgy: The Requisites, 3rd ed, 159–192.

CASE 4 Emphysema 1. A, C, and D. Fibrti (hrni) hypersensitivity pneumnitis is haraterized by imaging signs f brsis, namely vlume lss that ften invlves the mid and upper lung znes. The hest radigraph may be nrmal in patients with mild

150

SECTION IV

Answers

entrilbular emphysema and in early ases f lymphangileimymatsis (LAM), but in mderate t advaned ases the lungs may appear hyperinated. Mst patients with asthma have nrmal hest radigraphs, but hyperinatin f the lungs des ur in patients with severe disease and may be transient r xed. 2. B. Swyer-James-MLed syndrme is assiated with pstinfetius brnhilitis urring in hildhd, usually at 2.5 m) n lateral hest radigraphy. Hyperlueny f the upper lung znes urs as result f mderate t severe lung destrutin with dereased perfusin. Rapid tapering and attenuatin f pulmnary vessels is a harateristi radigraphi nding in the upper lung znes f affeted patients. Flattened hemidiaphragms ur beause f hyperinatin and air trapping and are usually mst apparent n the lateral view

Comment PA and lateral views f the hest shw hyperinatin, with attened hemidiaphragms and widening f the retrsternal spae (Fig. 4.1). On the PA view, the upper lung znes appear hyperluent, with a pauity f pulmnary vessels that demnstrate rapid tapering and attenuatin as they emanate frm the hila. The entral pulmnary arterial strutures appear dilated, nsistent with assiated pulmnary hypertensin. Emphysema is dened as permanent, abnrmal enlargement f airspaes distal t the terminal brnhile, ampanied by destrutin f their walls. Radigraphi abnrmalities in patients with emphysema are related t verinatin f the lungs and lung destrutin. The latter is haraterized by redued vasularity r the presene f bullae. Overinatin f the lungs may be haraterized by several ndings, mst ntably attening f the hemidiaphragms best appreiated n the lateral view and an inrease in the retrsternal air spae diameter. Chest radigraphi abnrmalities are usually evident in mderate t severe ases f emphysema, but radigraphs are frequently nrmal in patients with early emphysema. Thin-setin mputed tmgraphy (CT) is superir t hest radigraphs in deteting and haraterizing emphysema and has a high sensitivity and speiity fr establishing the diagnsis. Other nditins that may result in verinatin f the lungs are asthma and ysti brsis.

REFERENCES

Gietema HA, et al. Quantifying the extent f emphysema: fatrs assiated with radilgists’ estimatins and quantitative indies f emphysema severity using the ECLIPSE hrt. Acad Radiol. 2011;18(6):661–671. Raghu G, Remy-Jardin M, Ryersn CJ, et al. Diagnsis f Hypersensitivity Pneumnitis in Adults. An Ofial ATS/JRS/ALAT Clinial Pratie Guideline. Am J Respir Crit Care Med. 2020 Aug 1;202(3):e36–e69. Thrai Radilgy: The Requisites, 3rd ed, 391–404.

CASE 5 Sarcoidosis (Stage I) 1. C. Mediastinal and hilar lymphadenpathy may ur in saridsis, lymphma, metastati disease, infetin, and pneumnisis. In a yung patient with n systemi symptms, as in this ase, the presene f symmetri bilateral hilar and mediastinal lymphadenpathy is mst harateristi f saridsis. 2. E. Head and nek malignanies, melanma, breast aner, and geniturinary malignanies are all assiated with metastases t mediastinal and hilar lymph ndes. Other auses f bilateral symmetri hilar lymphadenpathy inlude lymphma, ther metastases (e.g., renal ell arinma, testiular aner) and pneumnisis (i.e., silisis) in upatinally expsed individuals. 3. C. Patients infeted by histplasmsis may be asymptmati r suffer frm prgressive pulmnary disease and pssible disseminatin; and mediastinal lymphadenpathy is mmn in bth aute and subaute ases. Blastmysis may manifest as aute r hrni pulmnary disease mimiking mmunity-aquired pneumnia and malignany, and may ultimately prgress t disseminated disease with utaneus, geniturinary, and bny lesins. Primary idiidmysis manifests with pulmnary invlvement and affeted patients are ften asymptmati. Symptmati patients usually present with fever, ugh, and hest pain, features that mimi mmunity-aquired pneumnia. Chest radigraphs may shw nslidatin, ndules, r peripheral, slitary thinwalled avities and parapneumni effusin. 4. D. Bilateral asymmetri hilar lymphadenpathy may ur in lymphma, saridsis, and metastati disease (e.g., renal ell arinma, testiular aner, breast aner, lung aner).

Comment Differential Diagnosis Saridsis, lymphma, metastati disease

Discussion Cmpsite image f PA hest radigraph and axial ntrast-enhaned mputed tmgraphy (CT) (Fig. 5.1). The hest radigraph shws right paratraheal and bilateral hilar lymphadenpathy, frming the radigraphi “1,2,3 sign” that is harateristi f saridsis (Stage I). CT images nrm thse areas f lymphadenpathy and reveals additinal invlvement f prevasular and subarinal lymph nde statins. Saridsis is a multisystem hrni inammatry disease f unknwn etilgy, haraterized by widespread nnaseating granulmas. Beause this pathlgi nding may als be seen in a variety f ther nditins, a diagnsis f saridsis requires nrdant radilgi, linial, labratry, pathlgi ndings, and exlusin f ther entities (espeially granulmatus infetins).

Opening Round

151

The hest radigraph is abnrmal in apprximately 90% f patients with saridsis. Bilateral, symmetri hilar lymph nde enlargement is the mst mmn radigraphi abnrmality and is frequently ampanied by bilateral mediastinal lymph nde enlargement. Lung parenhymal disease is usually ndular r retiulndular in appearane, with a prediletin fr the upper and mid-lung znes. On thin-setin CT, sarid granulmas typially manifest as small (2–3 mm) ndules, with a harateristi perilymphati distributin that inludes the peribrnhvasular lymphatis, the interlbular septa, and subpleural lymphatis (peripherally and alng the ssures). Apprximately 20% f patients with saridsis may prgress t have pulmnary brsis with upper lung arhitetural distrtin, vlume lss, and ysti hanges.

mediastinal strutures, vertebral bdy invasin, diret extensin f tumr t the ntralateral pleura, and the presene f distant metastases. The presene f any ne r mre f these ndings preludes surgial resetin. Patients with limited disease may be nsidered andidates fr attempted surgial ure by extrapleural pneumnetmy. Regardless f therapy, hwever, malignant mesthelima is almst always fatal.

REFERENCES

CASE 7

Little BP. Saridsis: verview f pulmnary manifestatins and imaging. Semin Roentgenol. 2015 Jan;50(1):52–64. Thrai Radilgy: The Requisites, 3rd ed, pp 355–376.

CASE 6 Mesothelioma 1. A, B, C, D. The mputed tmgraphy (CT) imaging features f pleural thikening that suggest the presene f malignany inlude the fllwing: >1 m in thikness, ndularity, irumferential grwth pattern within the invlved hemithrax, and invlvement f the mediastinal pleural. 2. C. Pleural metastasis is the mst mmn frm f pleural malignany, whereas mesthelima and pleural invlvement by lymphma are muh less mmn. 3. B. Althugh pleural plaques may be present in the setting f mesthelima, their presene is nt required fr a diagnsis f mesthelima. The earliest manifestatin f mesthelima is unilateral diffuse pleural thikening and a pleural effusin. 4. A. Epitheliid is the mst mmn histlgi subtype f mesthelima. Althugh all are assiated with a pr prgnsis, the sarmatid subtype is typially mst aggressive albeit least mmn.

Comment Differential Diagnosis Pleural metastases, mesthelima, lymphma, invasive thymma.

Discussion Althugh relatively rare, malignant mesthelima is the mst mmn primary neplasm f the pleura. Mst affeted individuals (80% f ases) have a histry f asbests expsure 30 t 40 years befre the diagnsis, typially thrugh their upatin. Males are affeted mre mmnly than females, likely reeting a gender disparity in the upatins assiated with asbests expsure (e.g., nstrutin and shipyard wrkers, re ghters). Affeted patients typially present with mplaints f hest pain and dyspnea. The mst mmn radigraphi nding is the presene f diffuse pleural thikening that is typially ndular and irregular in nguratin and ften irumferential within the invlved hemithrax (Figs. 6.1A and B). In sme ases, diffuse pleural thikening may be ampanied by a redutin in the size f the affeted hemithrax, with assiated ipsilateral shift f the mediastinum. Pleural effusin is ften present. CT and magneti resnane imaging (MRI) are superir t radigraphy in assessing the extent f disease. In many ases, the tw mdalities play mplementary rles in the evaluatin f resetability and may be useful in assessing fr transdiaphragmati extensin, diffuse hest wall invasin, invasin f vital

REFERENCES

Leung AN, Müller NL, Miller RR. CT in differential diagnsis f diffuse pleural disease. AJR Am J Roentgenol. 1990 Mar;154(3):487–492. Miller BH, Rsad-de-Christensn ML, Masn AC, etal. Malignant pleural mesthelima: radilgi pathlgi rrelatin. Radiographics. 1996;16:613–644. Thrai Radilgy: The Requisites, 3rd ed, 159–192

Junction Lines 1. D. The strutures indiated by the arrw and arrwhead are the anterir and psterir juntin lines, respetively. 2. A, B, C, and D. Pneummediastinum may develp in individuals wh engage in inhalatinal substane abuse and is als knwn t ur in the setting f a sustained Valsalva maneuver (e.g., vmiting, straining, weightlifting). It is als assiated with bstrutive lung disease and urs in up t 15% f patients with pulmnary brsis. 3. B. The nept f radilgi mediastinal mpartments was intrdued by Dr. Benjamin Felsn in the 1970s using the lateral hest radigraph t evaluate a deteted mediastinal mass and help frmulate a differential diagnsis. In his methd, the mediastinum was divided int anterir, middle, and psterir mpartments using anatmi landmarks visible n the lateral hest radigraph. Felsn’s anterir mpartment extends frm the sternum t a line drawn alng the anterir aspet f the trahea and alng the psterir aspet f the heart, and the middle mpartment extended frm that same line t a psterir line drawn alng the anterir third f eah thrai vertebral bdy. The psterir mpartment extends psterirly frm that line and inludes the paravertebral regin. This designatin differs frm the denitin f mediastinal mpartments used by anatmists and pathlgists in whih the heart upies the middle mediastinum. 4. B. Ttal ateletasis f either lung is assiated with inreased mpensatry mehanisms as mpared t lbar ateletasis. The anterir juntin line is typially displaed ipsilaterally tward the ateletati lung, and the ntralateral lung hyperinates in a mpensatry fashin and ften rsses the midline tward the ateletati lung. The ipsilateral hemidiaphragm may be elevated, partiularly in ttal ateletasis f the left lung, and there is ften marked ipsilateral shift f the mediastinum, partiularly f the anterir mediastinal mpnents sine the psterir mediastinal strutures are relatively tethered t paraspinal strutures.

Comment Differential Diagnosis Nne.

Discussion The ned-dwn frntal radigraph demnstrates the nrmal appearane f the anterir (arrw) and psterir (arrwhead) juntin lines (Fig. 7.1), frmed by the lse appsitin f viseral and parietal layers f pleura f bth lungs as they apprximate in

152

SECTION IV

Answers

the anterir and psterir aspets f the mediastinum, respetively. The anterir and psterir juntin lines may be seen n 24.5% t 57% and 32% f hest radigraphs, respetively. The anterir prtin f the thrax begins at the thrai inlet and the superir aspet f the anterir juntin line begins at the undersurfae f the laviles and typially urses bliquely frm right t left but may appear mre vertial in alignment as demnstrated in this ase. The psterir prtin f the thrax extends mre superirly than the anterir prtin and thus the psterir juntin line may be seen t extend abve the level f the laviles, as in this illustratin. It typially appears as a straight vertial line, ften visible thrugh the traheal air lumn. Obliteratin r abnrmal nvexity f the anterir juntin line suggests underlying anterir mediastinal disease (e.g., thyrid r thymi mass, lymphadenpathy, lipmatsis) whereas similar ndings related t the psterir juntin line suggest a psterir mediastinal abnrmality (e.g., esphageal mass, lymphadenpathy, arti disease, r neurgeni tumr). Either juntin line may als be displaed by vlume lss r hyperinatin f the surrunding lung A line typially measures 3 m being malignant is 5.23. A lung ndule dubling time between 30 and 400 days has a likelihd rati f 3.40 f being malignant. Ndules that duble in vlume in less than 30 days are mst likely benign, typially infetius. Lung aners ur mre mmnly in the upper lbes than the lwer lbes, with slight right upper lbe predminane. The likelihd rati f an upper lbe ndule being malignant is 1.22.

Comment Differential Diagnosis The hest radigraph demnstrates a densely alied ndule in the left lwer lung, regnized benign pattern. These alied ndules are typially the result f remte granulmatus infetin suh as tuberulsis r histplasmsis.

161

Solitary Pulmonary Nodule There are tw aepted radigraphi riteria fr a benign slitary pulmnary ndule: lak f grwth ver at least 2 years and identiatin f a benign aliatin pattern within a smthly marginated pulmnary ndule. Rughly half f all reseted slitary pulmnary ndules prve t be benign. Clinial indiatrs that suggest a benign diagnsis inlude age yunger than 35 years and histry f expsure t tuberulsis r residene in areas with endemi tuberulsis r fungal infetins. Suh indiatrs are, unfrtunately, insufiently spei t be helpful in mst individual ases. Fr patients with ndules that d nt meet the aepted radigraphi riteria fr benignany, nnntrast CT with thin-setin imaging is usually the preferred methd fr further evaluatin. CT is mre sensitive than nventinal radigraphs fr deteting alium and fat within a ndule. In ertain ases, CT imaging allws a ndent diagnsis f a spei benign entity suh as granulma, hamartma, arterivenus malfrmatin, pulmnary infartin, muid impatin, and pulmnary sequestratin. When CT is nndiagnsti, the methd f further evaluatin depends n patient harateristis and ndule mrphlgy. Nninvasive imaging mdalities inlude ntrast-enhaned CT t assess fr abnrmal ndule enhanement and urdexygluse(FDG)-psitrn emissin tmgraphy (PET)/CT imaging, whih relies n abnrmal gluse analgue (FDG) uptake t distinguish benign frm malignant ndules. Hwever, false psitive and false negative examinatins an ur with FDG-PET/ CT beause f inreased metabli ativity in inammatry ndules and lw metabli ativity in lw-grade malignanies. REFERENCES

Erasmus JJ, Cnnly JE, MAdams HP, Rggli VL. Slitary pulmnary ndules: part I. Mrphlgi evaluatin fr differentiatin f benign and malignant lesins. Radiographics. 2000;20:43–58. Erasmus JJ, MAdams HP, Cnnly JE. Slitary pulmnary ndules: part II. Evaluatin f the indeterminate ndule. Radiographics. 2000;20:59–66. Thrai Radilgy: The Requisites, 3rd ed, 428–440.

CASE 25 Radiation Pneumonitis 1. C and D. The sharp and relatively straight lateral and superir margins, nnanatmi distributin n the lateral view, and distrtin and vlume lss are highly suggestive f radiatin indued injury beause the inammatin urs nly within the radiatin prt. Infetin an ause perihilar dense nslidatin as well, althugh margins are typially less well dened exept when nslidatin abuts a pulmnary ssure. Saridsis an ause perihilar and entral paities, whih asinally appear nslidative, but it is nearly always bilateral and typially symmetri. Idipathi pulmnary brsis is haraterized by peripheral and basal predminant interstitial brsis. In this patient, there is nslidatin, whih is limited t the entral prtin f the right lung. 2. C. Radiatin pneumnitis is usually apparent n the hest radigraph by 6 t 8 weeks fllwing mpletin f therapy. On asin, radiatin pneumnitis an have a rapid r delayed nset. Hwever, this is nt the usual time urse. Numerus fatrs suh as radiatin dse, nmitant hemtherapy, and hst fatrs ntribute t radiatin indued lung injury. Cmputed tmgraphy (CT) an shw shw abnrmalities befre they beme apparent n hest radigraphy. 3. A. Althugh different tumr types an alter the radiatin treatment plan fr a patient, the tumr histlgy itself des nt diretly ntribute t the patient’s risk fr develping

162

SECTION IV

Answers

radiatin-indued lung injury. Sme hemtherapeuti agents suh as gemitabine an predispse a patient t develping radiatin-indued lung disease. Higher radiatin dses inrease the risk f lung injury. Larger tumrs require targeting therapy t a larger vlume f tissue, thus inreasing a patient’s risk f develping radiatin-indued lung disease. 4. D. Radiatin brsis usually develps 6 t 12 mnths after the mpletin f therapy. Radiatin pneumnitis is nt a prerequisite fr develping radiatin brsis, and patients with radiatin pneumnitis might nt develp brsis. Patients wh have radiatin pneumnitis and are symptmati an have a nrmal radigraph, while sme patients with abnrmal radigraphs are asymptmati. Radiatin brsis usually stabilizes 12 t 24 mnths after mpletin f therapy, depending n the tehnique(s) used. A prgressing abnrmality n the hest radigraph after this time shuld raise suspiin fr reurrent neplasm r anther press.

Comment Differential Diagnosis Psteranterir (PA) and lateral radigraphs shw dense perihilar nslidatin in the right lung with assiated vlume lss. Nte the sharp lateral and superir margins n the PA radigraph and the nnanatmi distributin n the lateral view where nslidatin rsses the pulmnary ssures. This nnanatmi distributin f dense nslidatin with vlume lss is typial f radiatin indued injury. Infetin and aspiratin uld have a similar appearane, althugh the margins typially are less well dened. Adenarinma uld be nsidered with hrni nslidatin.

Discussion The hest radigraph (Figure 1) shws dense right lung nslidatin with sharp lateral and superir margins with a nnanatmi distributin, spanning the right lung in the sagittal plane. The gegraphi margins and gemetri shape rrespnd t the eld f irradiatin. Radiatin pneumnitis and brsis an ur in patients wh have reeived denitive radiatin therapy fr lung aner. Radiatin pneumnitis is generally bserved n hest radigraphs within 6 t 8 weeks fllwing mpletin f treatment, but CT might detet subtle abnrmalities earlier than radigraphy (within a few weeks after mpletin f treatment). Suh paities are harateristially sharply demarated and are nt limited by anatmi bundaries suh as ssures. Fibrsis usually develps within 6 t 12 mnths fllwing radiatin therapy. With time, radiatin brsis an ensue. The parenhymal paities generally beme mre linear in nguratin and are usually ampanied by vlume lss and tratin brnhietasis. Several methds have been develped t deliver an adequate dse f radiatin t tumrs while limiting the amunt f expsure t nrmal lung parenhyma. These tehniques inlude limited radiatin prtals, tangential beams, nfrmed therapy, intensity-mdulated radiatin therapy. These methds an result in variable patterns f radiatin-indued lung injury and steretati bdy radiatin therapy (SBRT). Knwledge f the tempral relatinship and type f therapy an help t distinguish radiatin hanges frm infetin and malignany. Flurdexygluse psitrn emissin tmgraphy (FDG PET)/CT an be helpful fr distinguishing radiatin-indued lung disease frm malignany when perfrmed at least 6 t 12 mnths after mpletin f radiatin therapy. Radiatin pneumnitis an be treated expetantly, r patients may be given rtisterids. Severe pneumnitis an require supplementary xygen r mehanial ventilatin.

REFERENCES

Chi YW, Munden RF, Erasmus JJ, etal. Effets f radiatin therapy n the lung: radilgi appearanes and differential diagnsis. Radiographics. 2004;24:985–997. Thrai Radilgy: The Requisites, 3rd ed, 405–427.

CASE 26 Acute Respiratory Distress Syndrome With Barotrauma 1. A, B, and D. ARDS is a frm f aute hypxi respiratry failure with myriad auses inluding infetin, trauma, inhalatinal injury, drug txiity, aspiratin, and llagenvasular disease. The hest radigraph is always abnrmal and usually shws extensive lung paiatin with air brnhgrams. Saridsis mst mmnly presents as symmetri mediastinal and hilar lymphadenpathy with r withut lung invlvement, whih inludes bilateral perilymphati ndules and less mmnly large paities. 2. A and D. The hest radigraph (Fig. 26.1) shws a luent gas lletin in the right hemithrax separating the viseral and parietal pleura laterally. There is als a pneummediastinum. There is air streaking int the next superimpsed n the spinus presses f the upper vertebral bdies. The mputed tmgraphy (CT) (Supplemental Fig. 26.1) shws air in the anterir mediastinum. Pneummediastinum typially nsists f streaks f air in the mediastinum n the frntal radigraph, with gas smetimes utlining strutures suh as the arta. Subutaneus gas is ften a sendary sign f pneummediastinum. Pneumperiardium is muh less mmn than pneummediastinum exept fr patients wh had reent heart surgery. Pneumperiardium usually surrunds ne r bth ventriles as a thin band f gas. N etpi subdiaphragmati gas is evident n this hest radigraph. 3. B. With evlutin f ARDS, rganizatin and healing, ften with brsis, ensue. The lungs beme stiff, and bartrauma an result frm higher pressure ventilatin, whih is required t maintain xygenatin. Pneumthrax r pneummediastinum an develp. Central venus atheter plaement an als ause pneumthrax, althugh inreased used f ultrasund guidane has redued this mpliatin. Pulmnary laeratin an ause pneumthrax, but there are nt ther signs f traumati injury t the hest in this patient. 4. D. Pleural effusin is nt a requirement fr the diagnsis f ARDS. By denitin, Pa2/Fi2 ≤300 is required fr ARDS, and the severity is assessed by the degree f impaired xygenatin. Bilateral lung paity n hest radigraphy is a riterin fr the diagnsis f ARDS. Hypxemi respiratry failure must develp within ne week f an insult.

Comment Differential Diagnosis The AP prtable radigraph shws supprt tubes in expeted latins, diffuse lung paity, and small right pneumthrax as well as pneummediastinum. The differential diagnsis f diffuse lung paity in an autely hypxemi patient is usually limited t ardigeni r nn-ardigeni edema, diffuse alvelar hemrrhage, r infetin; and distintin amng these auses is ften difult n imaging alne.

Discussion ARDS is a severe frm f aute lung injury that is thught t enmpass a variety f distint disrders that share mmn

Opening Round

pathphysilgi and linial features. The denitin f ARDS was updated in 2012 (the Berlin Denitin) t better reet understanding f ARDS and pitfalls in diagnsis. A variety f pulmnary and extrapulmnary nditins, an preipitate ARDS inluding infetin, trauma, inhalatinal injury, drug txiity, aspiratin, and llagen-vasular disease. The linial denitin f ARDS inludes Pao2/Fio2 ≤300, bilateral lung paity n the hest radigraph, aute nset, and exlusin f hydrstati lung edema. Beause f dereased lung mpliane and the need fr prlnged mehanial ventilatin, patients with ARDS an develp bartrauma, inluding subutaneus emphysema, pneumthrax, pneummediastinum, and pulmnary interstitial emphysema. Treatment f ARDS inludes ventilatry supprt and treatment f the underlying ause. The mrtality rate is high, apprximately 50%. Survivrs an have residual pulmnary deits. REFERENCES

ARDS Denitin Task Fre. Aute respiratry distress syndrme: the Berlin Denitin. JAMA. 2012 Jun 20;307(23):2526–2533. Thrai Radilgy: The Requisites, 3rd ed, 226–237.

CASE 27 Cavity From Postprimary Tuberculosis 1. A, B, and D. Lung aner an avitate, usually as the result f nersis. Granulmatsis with plyangiitis (frmerly Wegener’s granulmatsis) an present as a slitary avitary mass lesin with an irregular, thikened wall, althugh multiple lesins are mre mmn. Pstprimary tuberulsis (reativatin r reinfetin) ften manifests as avitary disease. Hwever, the status f the patient’s immune system determines the presentatin f tuberulsis, whether it is primary infetin, reativatin, r reinfetin. Cavities an frm in saridsis but are typially the sequelae f brsis and brnhietasis. Unilateral upper lbe disease in saridsis wuld be highly unusual. 2. C. Squamus ell arinmas have the highest assiatin with avitatin, espeially when they are large. Adenarinmas, small ell lung arinmas, and large ell arinmas an avitate, albeit less frequently than with squamus ell arinmas. 3. A. Wall thikness is nt spei fr the nature f a avitary mass lesin, but wall thikness 5 m (>50% nrmal diameter).Measurements may be made n axial mputed

Fig. 51.1C

Fair Game

are f vasular etilgy. When nrmal artinturs are indistinguishable frm a mediastinal mass, the silhuette sign may assist in lalizing the lesin t the arta. Other radigraphi lues might inlude nvexity f the right superir mediastinum, the hilum verlay sign, traheal deviatin, and lateralizatin f the left paraarti interfae. Peripheral aliatin may be a lue t the presene f an atherslerti aneurysm. Cntrast-enhaned CT and magneti resnane imaging (MRI) play imprtant rles in the diagnsis and surveillane farti aneurysms (Fig. 51.1C). REFERENCES

Cmputed tmgraphy pulmnary angigraphy (CTPA) has beme the mdality f hie fr diagnsis f PE with sensitivity and speiity reahing 96% and 100%, respetively. Cnversely, in patients with lw pre-test prbability fr PE, CTPA may prdue false-psitive rates as high as 42%. Findings f PE inlude mplete lusin r partial lling defets within pulmnary arteries. Oasinally, PE an be deteted as a hyperattenuating lling defet n nn-ntrast mputed tmgraphy (CT). Dual-energy CT maps idine perfusin within nrmal lung tissue, analgus t sintigraphy, and may allw visualizatin f perfusin defets distal t small embli.

Meyershn NM, etal. Rle f mputed CT in the assessment f the thrai arta. Curr Treat Options Cardio Med. 2015;17:35. Thrai Radilgy: The Requisites, 3rd ed, 97–136.

CASE 52 Pulmonary Infarct 1. E 2. B 3. C

Comment Differential Diagnosis Tumr thrmbus, primary pulmnary artery sarma, vasulitis, artifat (respiratry mtin, w, streak), mimis (pulmnary vein, muid impatin).

Discussion Pulmnaryemblus (PE) is a mmn linial prblem requiring prmpt diagnsis and management, as untreated PE is assiated with up t 30% mrtality. Symptms f aute PE are nn-spei and inlude dyspnea and pleuriti hest pain, as are the linial signs f tahypnea, tahyardia, and hemptysis. Clinial deisin supprt and validated linial sring systems (Wells, simplied Wells, and mdied Geneva sres) an assist the liniian in direting apprpriate wrk-up and management. If PE is suspeted in a patient with lw pretest prbability and a negative D-dimer test, an alternate diagnsis ther than PE shuld be nsidered. If the D-dimer test is psitive, then imaging is the next step in diagnsti wrkup. Chest radigraphs are ften the rst imaging perfrmed but may shw n abnrmality. Signs f PE n radigraphy inlude ateletasis, Westermark sign (parenhymal lueny beynd the luded vessel), Fleishner sign (an enlarged entral pulmnary artery), and “knukle sign” (tapered distal vessels due t entral thrmbus.) Pulmnary infartin may manifest as peripheral wedge-shaped nslidatin in the lwer lateral aspet f the invlved hemithrax that abuts the pleura (Hamptn hump) (Fig 52.1), r as ateletasis, an elevated hemidiaphragm, r pleural effusin. Chest radigraphy is primarily used t exlude ther diagnses that might mimi a PE, suh as pneumthrax, pneumnia, r rib frature. A reent hest radigraph is required t interpret any subsequent ventilatin-perfusin sintigraphy san. Ventilatin sintigraphy utilizes inhaled raditraers xenn 133 r tehnetium-99m (99mT) diethylenetriamine-pentaaeti aid whih are distributed thrugh the lungs. 99mT maraggregated human albumin is injeted intravenusly t map the lung perfusin. Diagnsis f PE is made when tw r mre segmental r larger wedge-shaped perfusin defets are fund in areas f nrmal ventilatin (V/Q mismath).

179

Fig. 52.2

Fig. 52.3

180

SECTION IV

Answers

On CT, pulmnary infarts typially manifest as peripheral, ften wedge-shaped nslidatin that extends t the pleural surfae (Figs. 52.23 and 52.3). Grund-glass paity and bubble-like luenies may be seen—representing hemrrhage and infarted tissue—and beme mre ndular as they reslve. CTPA may identify patients with hemdynami mprmise wh are at inreased risk fr fatal r nnfatal adverse events. Imaging signs f right heart strain (right ventrile–left ventrile rati f >1.0), straightening r leftward bwing f the interventriular septum, ntrast reux int the hepati veins, and dilated mainpulmnaryartery. CTPA an als detet alternative diagnses suh as pneumnia, pneumthrax, periarditis, and arti dissetin. Pitfalls in CTPA interpretatin inlude respiratry mtin, image nise, w-related artifat, and streak artifat frm nentrated ntrast material within the superir vena ava. Mimis f PE inlude slw w in vessels (espeially within areas f nslidatin r ateletasis) and muus-plugged brnhi. Treatment inludes antiagulatin, inferir vena ava lter, systemi thrmblysis, atheter–direted mehanial r pharmalgi thrmblysis, and surgial embletmy. Mstpulmnaryembli reslve withut sequelae. Others may result in inmplete reslutin and hrni thrmbembli disease. REFERENCES

Kirsh J, etal. Amerian Cllege f Radilgy Apprpriateness Criteria ® Aute Chest Pain—Suspeted Pulmnary Emblism. J Am Coll Radiol. 2017;14(5):S2–S12. Kligerman SJ, etal. Radilgist perfrmane in the detetin f pulmnary emblism: features that favr rret interpretatin and risk fatrs fr errrs. J Thorac Imaging. 2018;33(6):350–357. Thrai Radilgy: The Requisites, 3rd ed, 238–258.

CASE 53 Lymphoma 1. E 2. D 3. C

plane as the hila. There may be traheal deviatin r narrwing frm mass effet. CT features typially inlude a midline mass f hmgeneus sft tissue attenuatin. Masses tend t be large and are dened as “bulky” if ver 10 m r if greater than ne-third f the intrathrai diameter is upied, prtending pr prgnsis. If ysti hange and nersis are present, the mass may appear mre hetergeneus. Lymphma tends t exert signiant mass effet n and displae adjaent mediastinal strutures. Invasin f adjaent hest wall r lung parenhyma may ur. Midline latin f the mass favrs lymphma, while an ff-midline latin might suggest an alternate diagnsis suh as thymma. Treated lymphma will derease in size and attenuatin with asinal aliatin. Caliatins are rare in untreated lymphma. Treatment respnse n MRI may manifest as inreased T2 signal in areas f tumr nersis and eventually, lw T2 signal indiating brsis. 18FDG-PET-CT plays an imprtant rle in staging and restaging f lymphma, with mplete respnse nsidered with reslutin f FDG ativity whether there is derease in lesin size. Whle-bdy and PET-MRI may be emplyed fr serial imaging in surveillane fr lymphma with added benet f n inizing radiatin. REFERENCES

Barringtn SF, etal. Rle f imaging in the staging and respnse assessment f lymphma: nsensus f the Internatinal Cnferene n Malignant Lymphmas Imaging Wrking Grup. J Clin Oncol. 2014;32(27):3048–3058. Shah KJ, et al. Diffuse large B-ell lymphma in the era f preisin nlgy: hw imaging is helpful. Korean J Radiol. 2017;18(1):54–70. Thrai Radilgy: The Requisites, 3rd ed., p. 113.

CASE 54 Traumatic Aortic Injury (Transection) 1. A, B, and E 2. D 3. E

Comment Differential Diagnosis

Comment

Mediastinal Hdgkin lymphma, thymi epithelial neplasm, mediastinal germ ell neplasm, mediastinal giter, lymph nde metastasis.

Differential Diagnosis

Discussion Primarymediastinal lymphma (PML) arises frm the thymus r lymph ndes, typially manifesting as a prevasular r (less mmnly) viseral mediastinal mass. Classiatin f PML is divided int Hdgkin (HL) r nn-Hdgkin (NHL). HL is dened by the presene f Reed-Sternberg ells and inludes lassial HL and ndular lymphyte predminant subtypes. Subtypes f primary mediastinal NHL inlude diffuse large B-ell lymphma, lymphblasti lymphma, and primary thymi large B-ell lymphma. Clinial symptms are nn-spei inluding ugh r tahypnea. Oasinally, mass effet might ause dysphagia r superir vena ava (SVC) syndrme. Fever and weight lss an als be present. Radigraphi features f lymphma mst mmnly inlude mediastinal wideningn the frntal prjetin and retrsternal lear spae lling n the lateral prjetin. A hilum overlay sign may be seen as preservatin f hilar vasulature interfaes, indiating the mass is nt in the same anterpsterir

Dutus bump, arti spindle, brnhial artery infundibulum, arti pseudaneurysm, penetrating atherslerti uler, pseudartatin.

Discussion Prmpt exlusin f traumati arti injury (TAI) after blunt hest trauma is ritial as suh injuries have a greater than 70% in-eld mrtality and up t 50% 24-hur mrtality if left untreated. TAI urs mst mmnly at sites where the arta is relatively xed, namely the rt, isthmus, and desending arta at the level f the diaphragmati hiatus. Patients wh survive lng enugh t reeive hspital are typially have injuries at the arti isthmus, just distal t the left sublavian artery rigin, near the ligamentum arterisum. Prpsed injury mehanisms inlude (1) rapid differential deeleratin f the xed prtins f the arta relative t ther mediastinal strutures, (2) shear stress n the wall, (3) sseus pinh sendary t psterir translatin f the sternum, and (4) hydrstati r water-hammer phenmenn aused by sudden inreases in intrathrai pressure resulting in tears. Rt injuries are assiated with ther serius ardivasular

Fair Game

181

esphagus, r nasgastri tube; and a left apial ap, indiating hemrrhage extending int the apial extrapleural spae. Cmputed tmgraphy angigraphy (CTA) is the mst apprpriate study fr evaluatin f TAI, yielding rapid, high-reslutin depitin f the injury and assiated ndings with a sensitivity and speiity f mre than 90% fr detetin f TAI. Diret CT signs f TAI inlude arti ntur abnrmality, intimal ap, intraluminal thrmbus, pseudaneurysm, and frank ntrast extravasatin. Preise CT haraterizatin f the lesin is imprtant fr treatment planning (stent versus surgery) and shuld inlude infrmatin related t the length f defet, arti diameter abve and belw, and distane frm the great vessel rigins. Detetin f a periarti hematma is an indiret sign f TAI, demnstrating bld in ntat with the arta (Fig.54.2). Mediastinal bld with a preserved fat plane arund the arta suggests a different sure f bleeding – frm mediastinal veins r interstal r internal mammary arteries. Fratures f the rst r send ribs shuld prmpt lse examinatin f the arta and arh vessels t exlude traumati vasular injury. Treatment f TAI typially inludes surgial r endvasular graft repair (Fig. 54.3). REFERENCES

Fig. 54.2

Steenburg SD, etal. Aute traumati arti injury: imaging evaluatin and management. Radiology. 2008;248:748–762. Cullen EL, et al. Traumati arti injury: CT ndings, mimis, and therapeuti ptins. Cardiovasc Diagn Ther. 2014;4(3):238–244. Thrai Radilgy: The Requisites, 3rd ed, 279–288.

CASE 55 Lipoid Pneumonia 1. A, B, C, D, E 2. C 3. D

Comment Differential Diagnosis Lung adenarinma (muinus), lymphma, alvelar prteinsis, alvelar sarid, hrni esinphili pneumnia, rganizing pneumnia.

Discussion

Fig. 54.3

mpliatins, inluding rnary arterial injury, hemperiardium, and tampnade. Supine hest radigraphy is the rst imaging mdality used in the trauma setting, mainly t exlude aute life-threatening nditins like tensin pneumthrax (Fig. 54.1). Several lues may assist in the diagnsis f TAI n hest radigraphy. These inlude widening f the vasular pedile (>8 m r 25% f the width f the thrax); lss f nrmal arti arh and desending arti nturs; rightward displaement f the trahea,

Lipid pneumnia (LP) is an unmmn ause f hrni lung paiatin n hest radigraphy and mputed tmgraphy (CT). The etilgies inlude exgenus and endgenus auses f lipid aumulatin in alveli. Exgenus sures inlude aspiratin f animal fats, mineral il, r vegetable ils. Endgenus LP results frm phagytsis f lipids by intraalvelar marphages that ur sendary t seretins that may aumulate beynd a pint f endbrnhial bstrutin, fat strage diseases, r alvelar prteinsis. Exgenus LP may have an aute nset if a large quantity f petrleum-based prdut is intrdued t the lung, resulting in an aute pneumnitis. Chrni LP typially fllws frm repeated, lng-standing aspiratin f animal fats r ther ils and manifests similarly t aute LP. Predispsing nditins t hrni LP inlude lder age, intelletual develpmental disrders, rpharyngeal dysphagia, gastresphageal reux, r ther fatrs whih may ause aspiratin (e.g., Zenker divertiulum). Chrni use f mineral il laxative is als lassially assiated. Radigraphi manifestatins vary arding t the ause, with aute exgenus lipid pneumnia prduing grund-glass and nslidative paities ften in a lbar and basilar distributin. Chrni LP may manifest as a nn-reslving nslidatin

182

SECTION IV

Answers

n hest radigraphy. On CT, LP is haraterized by the demnstratin f lw attenuatin within fal r multifal areas f parenhymal nslidatin equal t that f marspi fat (-30 HU) (Figs. 55.1 and 55.2). The areas f nslidatin are ften irregular r spiulated, frming ndular r mass-like paities that may mimi lung aner. CT detetin f fat attenuatin within a ndule r mass must be differentiated frm ther fat-ntaining lung lesins (e.g., hamartma, fat-ntaining metastases [sarmas].) Chrni LP may be assiated with arhitetural distrtin r ther evidene f brsis due t hrni inammatin. Pneumateles may develp in areas f previus pneumnitis and beme mpliated by pneummediastinum r pneumthrax if they rupture. A “razy-paving” pattern may als be seen n CT with nn-reslving grund-glass paity with superimpsed interlbular septal thikening. Lipid pneumnia may be avid n urdexygluse (FDG)-psitrn emissin tmgraphy (FDG PET) if there is assiated inammatin. Chrni airspae paities must be differentiated frm ther auses suh as pulmnary infetin, alvelar prteinsis, lung aner, and lymphma.

and ateletasis. Fifty perent f patients with PCD will have situs abnrmalities (hetertaxy r situs inversus) (Fig. 56.3). REFERENCES

Knwles MR, et al. Primary iliary dyskinesia. Clin Chest Med. 2016;37(3):449–461. Thrai Radilgy: The Requisites, 3rd ed, 137–158, 103–209.

CASE 57 Invasive Aspergillus 1. 2. 3. 4.

A, B, and C C C D

Comment Differential Diagnosis

REFERENCES

Betanurt SL, etal. Lipid pneumnia: spetrum f linial and radilgi manifestatins. AJR Am J Roentgenol. 2010;194:103–109. Marhiri E, etal. Exgenus lipid pneumnia. Clinial and radilgial manifestatins. Respiratory Medicine. 2011;105:659–666. Thrai Radilgy: The Requisites, 3rd ed, 355–376, 405–427.

CASE 56 Kartagener Syndrome (Primary Ciliary Dyskinesia) 1. B 2. A, C, D 3. B

Comment Differential Diagnosis Cysti brsis, allergi brnhpulmnary aspergillsis, pst-infetius brnhietasis, mmn variable immune deieny, human immundeieny virus (HIV).

Discussion Kartagener syndrme is a subset f the primary iliary dyskinesia (PCD) spetrum f ilipathies haraterized by a lassi triad f brnhietasis, situs inversus, and sinusitis. Patients with PCD inherit a geneti defet that leads t impaired iliary struture and funtin in an autsmal reessive pattern. Early symptms f brnhitis, sinusitis, titis media, and rhinitis ften lead t early hildhd presentatin, whih may be mpliated by reurrent pneumnia. Patients may present later in life with redued fertility beause f impaired spermatzn iliary funtin. Brnhietasis assiated with this disrder is typially less severe than that f ysti brsis. Imaging features inlude situs inversus and brnhietasis, with r withut superimpsed infetin (Fig. 56.1). Brnhietasis in this nditin typially affets the right middle and lwer lbes. Centrilbular ndules and tree-in-bud paities indiate the small airways affeted by muus and infetin. Msai attenuatin and air-trapping may be seen. Maxillfaial mputed tmgraphy (CT) will shw sequelae f aute and hrni sinusitis inluding musal thikening and muperisteal reatin, with r withut superimpsed air-uid levels (Fig. 56.2). Other assiated ndings inlude brnhial wall thikening, lung hyperinatin,

Other pneumnia (baterial, mybaterial, r fungal), granulmatsis with plyangiitis, pulmnary infartin, lung aner.

Discussion Invasive pulmnary aspergillsis is the mst mmn fungal infetin t affet immunmprmised patients, usually thse with severe neutrpenia. Ppulatins at risk fr this ptentially fatal infetin inlude transplant reipients, hematlgi malignanies, r thse reeiving hrni highdse sterids. Radigraphi manifestatins f invasive aspergillus inlude slitary r multiple ill-dened pulmnary ndules r masses, and nuent nslidatin (Fig. 57.1). When Aspergillus fumigatus rganisms invade pulmnary bld vessels (angiinvasive frm), lalized small t medium sized pulmnary arteries beme luded leading t peripheral infartin. CT ndings f pulmnary infartin inlude peripheral wedge-shaped subpleural nslidatin with surrunding grund-glass paity (CT halo sign) in whih the hal represents hemrrhage (Fig. 57.2). This sign is nt spei fr Aspergillus but is highly suggestive in the apprpriate linial setting. Other invasive fungal rganisms suh as Mucor and Candida speies may als manifest with hals n CT. Early regnitin f the hal sign may prmpt the additin f anti-fungal treatment, thereby reduing mrbidity and mrtality. After nset f therapy, areas f nslidatin may avitate and frm the air crescent sign, indiating revery f neutrphil funtin and respnse t treatment. REFERENCES

Walsh S, et al. Imprtane f the reversed hal sign fr the diagnsis f angiinvasive pulmnary aspergillsis. Respir Med. 2014;108(8):1240. Prasad A, etal. Pulmnary aspergillsis: what CT an ffer befre it is t late! J Clin Diagn Res. 2016;10(4):TE01–TE05. Thrai Radilgy: The Requisites, 3rd ed, 155.

CASE 58 Granulomatosis With Polyangiitis 1. 2. 3. 4.

A, B, C, D, and E E C D

Fair Game

Comment Differential Diagnosis Septi embli, lung absesses, tuberulsis, trahebrnhial papillmatsis, pulmnary metastases.

Discussion Antineutrphil ytplasm antibdy (ANCA)-assiated vasulitis is the mst mmn f the primary small vessel vasulitides that inlude granulmatsis with plyangiitis (GPA) (frmerly Wegener granulmatsis), esinphili granulmatsis with plyangiitis (E-GPA), and mirspi plyangiitis. GPA represents a multisystem nertizing vasulitis resulting in the lassi linial triad f upper airway disease with lung and renal invlvement (glmerulnephritis). The upper respiratry trat is invlved in almst all patients and may inlude symptms and signs f titis, sinusitis, rhinrrhea, epistaxis, r airway stensis. Lungs and kidneys are invlved in 90% and 80% f patients, respetively. The presene f ytplasmi ANCA (ANCA) suggests the diagnsis. Treatment typially inludes ylphsphamide and sterids. Chest radigraphi and CT ndings assiated with GPA inlude multiple pulmnary ndules and masses, seen in up t 90% f patients with pulmnary disease (Fig. 58.1). The distributin f ndules and masses is usually bilateral, lwer lung zne, and subpleural. Ndules may alese int larger masses >10 m in diameter, with predispsitin t avitatin when greater than 2 m, frming thik-walled irregular avities and dereasing in size with treatment (Fig. 58.2). On ntrastenhaned CT, mst ndules r masses demnstrate entral lw attenuatin, indiating nersis. Less mmnly grund-glass paity may surrund the ndule (CT halo sign). Other CT ndings inlude reverse CT halo and feeding vessel signs, the latter representing an artery, leading int a ndule r mass. Cnslidatin and grund-glass paities are the send mst mmn manifestatin after ndules and masses, reeting pneumnitis r alvelar hemrrhage related t vasulitis. Airway abnrmalities inlude traheal and brnhial wall thikening. Cnentri inammatry wall thikening an result in airway stensis, with subgltti traheal invlvement being mst typial. The differential diagnsis fr subpleural ndules and masses f GPA inludes septi embli and absess, neplasms (inluding hematgenus metastases and lymphma), rganizing pneumnia, and Kapsi sarma. REFERENCES

Feragalli B, etal. The lung in systemi vasulitis: radilgial patterns and differential diagnsis. Br J Radiol. 2016;89(1061):20150992. Thrai Radilgy: The Requisites, 3rd ed, 238–258.

CASE 59 Lymphangioleiomyomatosis 1. A-C. The CT images shw diffuse ysti lung disease thrughut the lungs. Of the listed hies, the nly hie whih is nt a knwn ause f diffuse ysti lung disease is Hermansky-Pudlak syndrme, whih is an inherited ause f pulmnary brsis and ulutaneus albinism, mre mmn amng peple frm Puert Ri. 2. B. The ysts in Langerhans ell histiytsis (LCH) are usually upper lung prepnderant with sparing f the stphreni angles. Cysts an have thiker walls than in ther ysti lung diseases and may alese t frm bizarre shapes. The ysts ften arise frm entrilbular ndules whih

183

prdue small airway bstrutin. Cmbined with a histry f smking, these ndings are essentially diagnsti f LCH. 3. A. Lymphangileimymatsis (LAM) may ause pleural effusins whih are hylus (hylthrax) in up t 1/3 f patients. The mst mmn auses f hylthrax are trauma t the thrai dut and lymphma. It wuld be unusual fr the ther listed nditins t lead t hylthrax. 4. C. Hepatellular arinma is nt assiated with tuberus slersis mplex (TSC). Cmmn manifestatins f TSC inlude a LAM-like ysti pattern f lung disease, rhabdmymas in the heart, renal angimylipmas, a myriad f skin lesins, brain lesins, slerti bne lesins and multindular multifal pneumyte hyperplasia. The latter entity prdues multiple small pulmnary ndules.

Comment LAM is haraterized by abnrmal prliferatin f immature smth musle ells. Thin-walled lung ysts in LAM may rupture, leading t spntaneus pneumthraes. Lymphati bstrutin may result in hylus pleural effusins. Vasular endthelial grwth fatr-D (VEGF-D) levels are ften elevated in LAM but nt in ther ysti lung diseases and an be used t supprt the diagnsis and t mnitr prgressin f disease. Reently, sirlimus has been shwn t stabilize lung funtin in LAM. Reent evidene als suggests that sirlimus may be beneial in the treatment f ther manifestatins f TSC. The differential diagnsis fr diffuse ysti lung disease is nt as brad as in ther diffuse lung diseases. Mst ases are either due t TSC, LAM, r LCH. In mst ases, the CT imaging manifestatins f LAM and TSC in the lungs are indistinguishable, althugh multindular multifal pneumyte hyperplasia (MMPH) urs almst exlusively in TSC with r withut manifestatins f LAM. The features f MMPH n CT nsist f multiple small ndular lesins usually 5 mm r less in diameter whih an be slid r subslid in attenuatin. Obviusly, if ther nn-pulmnary manifestatins f TSC are evident, the diagnsis an be made readily. LAM and LCH are usually quite different in their imaging appearane. Cysts in LAM are unifrmly thin-walled and evenly spaed thrughut the lungs. In LCH, the ysts may be asymmetri r bizarrely shaped beause ysts may alese ver time. Small ndules may be present in LCH whih wuld be very unusual in LAM. In LCH, the diffuse lung disease lassially spares the stphreni angles. Regarding demgraphis, all LAM patients (and vast majrity with TSC) are wmen, usually f hild-bearing age. LCH als tends t affet yung adults wh are smkers but is nt limited t wmen. Less mmn auses f diffuse ysti lung disease inlude lymphyti interstitial pneumnitis (LIP), Birt-Hgg-Dube syndrme (BHD), and amylidsis. Helpful diagnsti pearls fr these nditins: LIP: Almst always sendary rather than idipathi. In adults, mst ften assiated with Sjgren syndrme. May als ur in immundeieny syndrmes. BHD: Assiated with renal tumrs (mst ften hrmphbi renal ell arinma) and skin lesins (brflliulmas mst mmn). Autsmal dminant inheritane. Amylidsis: May ur with LIP and Sjgren syndrme. Prdues ndular amylid fi whih may be partially alied/ ssied. REFERENCES

Seaman DM, Meyer CA, Gilman MD, MCrmak FX. Diffuse ysti lung disease at high-reslutin CT. AJR Am J Roentgenol. 2011;Jun;196(6):1305–1311. Thrai Radilgy: The Requisites, 3rd ed, 355–376.

184

SECTION IV

Answers

CASE 60

CASE 61

Tuberculosis

Pneumocystis jirovecii Pneumonia

1. A-C. The mputed tmgraphy (CT) images shw large nerti lymph ndes in the mediastinum.), Tuberulus lymphadenitis, lymphma, and metastati disease (usually frm a squamus ell arinma primary) are the mst mmn auses fr nerti lymphadenpathy. Saridsis is a very mmn ause f large bulky lymph ndes in the mediastinum and hilar regins; hwever, nersis within lymph ndes wuld be unusual. 2. D. Cnslidatin is a mmn nding f tuberulsis in hildren. Fibravitary disease is seen in ative tuberulsis and brnhietasis may ur in ative disease r as late nsequene f remte tuberulsis. Bth are mre frequently identied in adults. Grund-glass paity is nt a majr nding in mst ases f tuberulsis. 3. A. Lymphadenpathy in tuberulsis (TB) is mmn in hildren and thse with human immundeieny virus– aquired immundeieny syndrme (HIV-AIDS) with CD4 unts belw 200/mm^3. 4. A. TB pleural effusins are very unmmn in infants and unmmn in hildren. Up t 15% f adults may have pleural effusins as a manifestatin f TB; this may be the sle and initial manifestatin f TB in sme ases. Therefre, a high level f linial suspiin is neessary. A pleural bipsy is required fr diagnsis.

1. A, B, C, and D. The CT images shws diffuse grundgrund glass paity with superimpsed retiulatin septal thikening and intralbular lines nsistent with a razy paving pattern. All the answer hies an manifest this pattern n hest CT. Crazy paving n hest CT is lassially desribed in pulmnary alvelar prteinsis; hwever, given the relative rarity f this nditin, ther mre mmn disease presses are usually respnsible fr the pattern in mst linial praties. Anther rare ause f the razy paving pattern is hrni lipid pneumnia, whih urs in thse wh are at risk fr aspiratin r inhalatin f lipid-laden substanes (mineral ils, il-based nse drps, lip glss, et). 2. B. Up t 1/3 f HIV patients with Pneumocystis jirovecii pneumnia (PJP) will have assiated air ysts; very ften there is an upper lung prepnderane. The prevalene f ysts in PJP is lwer in nn-HIV afited patients. Interestingly, ysts may reslve with treatment. As in ther ysti lung diseases, these patients are at risk fr spntaneus pneumthraes. Signiant pleural effusins r lymphadenpathy is unusual in PJP. In the setting f PJP, tree-in-bud ndules n hest CT suggest superimpsed infetin in the small airways frm anther mirbial agent r frm aspiratin. 3. A. Lymphangileimymatsis (LAM) may ause pleural effusins whih are hylus (hylthrax) in up t 1/3 f patients. The mst mmn auses f hylthrax are trauma t the thrai dut (ften pst-surgial) and lymphma. It wuld be unusual fr the ther listed nditins t lead t hylthrax. 4. A. Trimethprim-sulfamethxazle (Batrim) is rst line therapy and prphylaxis fr PJP.

Comment The terms primary and pst-primary TB ntinue t be used in the medial literature. It was previusly believed that primary and pst-primary TB uld be differentiated n linial and imaging grunds. Hwever, DNA ngerprinting evidene has shwn that primary and pst-primary TB annt be differentiated reliably n imaging (i.e., time frm initial infetin t the ineptin f ative disease annt predit the imaging manifestatins f TB). Hwever, the imaging manifestatins f TB in adults, hildren, and HIV-psitive patients differ. Lymph nde enlargement is a harateristi feature f TB in hildren but is muh less mmn in adults. Lymph nde enlargement may ur alne r in assiatin with parenhymal nslidatin, thugh islated nslidatin in hildren is unusual. On ntrast-enhaned CT sans f patients with mediastinal and hilar tuberulus lymphadenitis, enlarged ndes ften demnstrate a lw-attenuatin enter and peripheral rim enhanement. Histlgially, suh ndes have been shwn t demnstrate entral nersis and a highly vasular, inammatry apsular reatin. Althugh lw-attenuatin ndes are harateristi f TB, they are nt spei fr this entity. Suh ndes may als be enuntered in atypial mybaterial and fungal infetins. Neplasti lymph ndes (e.g., metastati squamus ell arinma and lymphma) may als demnstrate this appearane. Regarding TB in HIV-psitive patients, the radigraphi appearane varies depending n the patient’s CD4 unt. In patients with CD4 unts abve 200/mm3, a pst-primary pattern is typially seen. In patients with CD4 unts belw 200/ mm3, yu will usually bserve a primary pattern, inluding lw-attenuatin lymph ndes and nslidatin. REFERENCES

Jeng YJ, Lee KS. Pulmnary tuberulsis: up-t-date imaging and management. AJR Am J Roentgenol. 2008 Sep;191(3):834–844. Thrai Radilgy: The Requisites, 3rd ed, 323–334

Comment The inidene f PJP has dereased as highly ative antiretrviral therapy (HAARTT) and hemprphylaxis has beme standard treatment in HIV-psitive patients. Hwever, PJP ntinues t be the mst mmn pprtunisti infetin in peple with AIDS. In additin t HIV-psitive patients, PJP may als affet ther immunmprmised individuals (in the setting f stem ell r slid transplant, hemtherapy fr malignany, hematlgial malignany, r hrni rtisterid therapy). The lassi hest radigraphi presentatin f PJP nsists f bilateral perihilar r diffuse symmetri paities, whih may be nely granular, retiular, r grund glass in appearane. Imprtantly, the hest radigraph may be nrmal at the time f presentatin in a signiant minrity f ases f PJP. CT, partiularly HRCT, is mre sensitive than hest radigraphs fr deteting PJP and thus may be helpful in evaluating symptmati patients with nrmal r equival radigraphi ndings. The lassi CT nding in PJP is extensive grund-glass attenuatin, whih rrespnds t the presene f intraalvelar exudate, nsisting f uid, rganisms, and debris. It is ften distributed in a pathy r gegraphi fashin, with a prediletin fr the entral, perihilar regins f the lungs. Grund-glass attenuatin is asinally ampanied by thikened septal lines, and fi f nslidatin may als be evident in severe ases. In up t a third f ases f PJP, grund-glass paities are ampanied by ysti lung disease. Suh ysts have an upper lbe predminane and demnstrate varying sizes and wall thiknesses. REFERENCES

Kanne JP, Yandw DR, Meyer CA. Pneumystis jirvei pneumnia: high-reslutin CT ndings in patients with and withut HIV infetin. AJR Am J Roentgenol. 2012 Jun;198(6):W555–W561. Thrai Radilgy: The Requisites, 3rd ed, 310–322.

Fair Game

CASE 62 Metastatic Osteosarcoma 1. A, B, C, and D. There are multiple bilateral alied mediastinal and hilar lymph ndes. All the listed nditins an ause alied mediastinal and hilar lymph ndes and wuld be viable diagnsti nsideratins althugh silisis wuld be unusual in suh a yung patient. 2. D. Lymphma frequently alies after hemtherapy and radiatin therapy. Caliatin within lymph ndes affeted by lymphma befre therapy is highly unusual. 3. D. Of the listed nditins, histplasmsis is the least likely t ause eggshell aliatin. 4. A. Garland’s triad (als knwn as the 1-2-3 sign and desribed n standard radigraphy) is dened as lymphadenpathy lalized t the right paratraheal and bilateral hilar regins. This symmetri pattern f lymphadenpathy is mst ften assiated with saridsis.

Comment Axial maximum intensity prjetin image frm hest mputed tmgraphy (CT) shws multiple hyperdense mediastinal and hilar lymph ndes as well as sattered hyperdense pulmnary ndules within the lungs nsistent with ssied metastati disease in the setting f stesarma. Calied lymph ndes are usually benign, and they are ften related t granulmatus presses, suh as tuberulsis (TB), histplasmsis, r saridsis. Neplasti auses f alied lymph ndes are less mmn. They inlude metastases frm muinus adenarinmas and lymphma. Regarding lymphma, aliatin is frequently seen fllwing radiatin therapy, but it is rarely enuntered in untreated ases. Ossied lymph ndes are a rare manifestatin f metastati stesarma. Suh ndes appear like alied lymph ndes. In patients with stesarma, the presene f lymph nde metastases indiates a pr prgnsis. Lymphati invlvement is usually ampanied by metastases within the lung whih is the mst mmn site f metastati disease. Lung metastases frequently demnstrate ssiatin. REFERENCES

Se JB, Im JG, G JM, Chung MJ, Kim MY. Atypial pulmnary metastases: spetrum f radilgi ndings. Radiographics. 2001 Mar–Apr;21(2):403–417. Thrai Radilgy: The Requisites, 3rd ed, 428–440.

CASE 63 Septic Infarcts 1. A, B, and C. The mputed tmgraphy (CT) images shw bilateral avitary ndules with peripheral prepnderane and pleural effusins. The differential diagnsis fr avitary ndules inludes metastati disease, septi infarts, multifal nerti pneumnia, and small vessel vasulitis. Cavitary ndules are nt a feature f adult respiratry distress syndrme; mrever, pleural effusins are als unmmn in aute respiratry distress syndrme (ARDS) unless there is superimpsed heart failure. 2. B. In a patient with hrni avitary ndules, subutaneus ndular lesins, and jint pain, rheumatid arthritis is the mst likely etilgy. Althugh there are many radilgi manifestatins f tuberulsis in the thrax, this nstellatin f ndings is nt suggestive f that diagnsis. Granulmatsis with plyangiitis (Wegner granulmatsis) is nt

185

typially assiated with jint pain r subutaneus ndules. Invasive aspergillsis is an aute nditin in neutrpeni patients. 3. C. Indwelling entral venus atheters, prstheti devies, triuspid endarditis (ften frm intravenus drug use), skin infetin, and peridntal disease are all mmn auses f septi infarts/embli. 4. A. Diffuse grund-glass paity with mild superimpsed ndularity are typial ndings in fat emblism. Risk fatrs fr fat emblism inlude pelvi and lng bne fratures, hemglbinpathies, burns, and panreatitis. Usually, there is a 1- t 2-day delay in the appearane f abnrmalities in the lungs beause mst f the pulmnary inammatry respnse fllws the breakdwn f fat glbules int free fatty aids.

Comment Axial image frm hest CT shws bilateral peripheral predminant avitary ndules with pleural effusins. Septi infarts mst ften riginate frm right-sided triuspid endarditis r frm infeted thrmbi within systemi veins. Other sures inlude skin infetins, peridntal disease, entral venus atheters, and paemaker wires. On hest radigraphs and CT sans f patients with septi infarts, yu may bserve prly dened ndular paities and areas f wedge-shaped parenhymal paiatin. Suh paities are usually peripheral in latin and abut the pleural surfae. They have a prediletin fr the lwer lbes. Cavitatin is frequently bserved, partiularly n CT sans. A harateristi nding n CT is the identiatin f feeding vessels leading t the ndules (arrws in the send gure) and wedge-shaped parenhymal paities. Thus, the CT nding f avitating ndules with feeding vessels is highly suggestive f septi infarts. REFERENCES

Khashper A, Disepla F, Ksiuk J, Qanadli SD, Mesurlle B. Nnthrmbti pulmnary emblism. AJR Am J Roentgenol. 2012 Feb;198(2):W152–W159. Thrai Radilgy: The Requisites, 3rd ed, 289–309.

CASE 64 Bronchopleural Fistula 1. C. The riginal hest radigraph shws near mplete paiatin f the left hemithrax after left pneumnetmy nsistent with uid lling the left pneumnetmy spae. The fllw-up hest radigraph shws new gas/uid levels within the upper left hemithrax, essentially diagnsti f a brnhpleural stula sendary t a brnhial stump dehisene. 2. B. The shrter brnhial stump after right pneumnetmy as well as the mre tenuus bld supply f the right main brnhus relative t the left main brnhus renders brnhpleural stulas fllwing pneumnetmy mre mmn n the right side. 3. A. Right middle lbe trsin after right upper lbetmy is the mst mmn type f lbar trsin. This surgial emergeny may result in hemrrhagi infartin r nersis f the invlved lbe due t entral kinking f vessels and lymphatis. 4. D. The next step in management f a suspeted brnhpleural stula wuld be brnhspy t nrm its presene. In additin, nulear mediine xenn sans uld als be pursued t nrm the diagnsis and pinpint the latin f the stula.

186

SECTION IV

Answers

Comment Brnhpleural stula is a relatively unmmn but serius mpliatin fllwing pneumnetmy, with a prevalene f up t 5% and a mrtality rate f apprximately 20%. Majr predispsing fatrs relate t perative auses f brnhial ishemia, suh as a lng brnhial stump, ligatin f the brnhial arteries t prximally, and disruptin f brnhial bld supply frm extensive lymph nde dissetin. Additinal risk fatrs inlude preperative radiatin therapy, sterid therapy, malnutritin, and resetin thrugh infeted r anerus tissue. Fllwing pneumnetmy, the mediastinum is nrmally shifted tward the side f resetin, and the pneumnetmy spae gradually lls with uid ver time. Brnhpleural stula shuld be nsidered when any f the fllwing are bserved: (1) the pneumnetmy spae fails t ll with uid; (2) there is an abrupt dwnward shift in the air-uid level in the pneumnetmy spae identied n an upright radigraph; (3) there is a new lletin f air in a previusly paied pneumnetmy spae; r (4) there is ntralateral shift f the mediastinum assiated with any f the abve ndings. The diagnsis an be nrmed with a xenn ventilatin study, whih will demnstrate xenn ativity in the pneumnetmy spae r brnhspy. REFERENCES

Chae EJ, Se JB, Kim SY, D KH, He JN, Lee JS, Sng KS, Sng JW, Lim TH. Radigraphi and CT ndings f thrai mpliatins after pneumnetmy. Radiographics. 2006 Sep–Ot;26(5):1449–1468. Thrai Radilgy: The Requisites, 3rd ed, 259–278.

CASE 65 Superior Sulcus Tumor 1. B. Squamus ell arinma is the mst mmn ell type f lung aner t present in the superir sulus (apex f the lung) with invasin f mediastinal and hest wall strutures. 2. C. Stage III B r stage IV lung aner patients are nsidered inperable. In the TNM staging system, hest wall invasin is nsidered T3 disease. T3 disease is nt sufient t establish stage III B status. Therefre N3 disease (ntralateral mediastinal/hilar lymphadenpathy r supralaviular/anterir salene lymphadenpathy) wuld be the nly setting in whih the patient wuld be nnperable. 3. B. Invasin f the mediastinum, heart, great vessels, trahea, reurrent laryngeal nerve, esphagus, vertebral bdy, r arina and separate tumr ndule(s) in anther ipsilateral pulmnary lbe is nsidered T4 disease and nnperable regardless f ther N and M ndings. 4. C. Standard therapy fr superir sulus tumrs inlude hemtherapy and raditherapy (CRT) with subsequent surgery 3 t 5 weeks after mpletin f CRT.

Comment A lung aner arising at the extreme apex f the lung is referred t as a superir sulus tumr r Panast tumr. Affeted patients typially present with symptms f shulder pain in 44% t 96%, Hrner’s syndrme (ptsis, misis, anhidrsis) in 14% t 50%, and weakness and atrphy f intrinsi musles f the hand in 8% t 22%. Chest radigraphs shw asymmetri paity in either apex (Figs. 65.1 and 65.2). Chest mputed tmgraphy (CT) better haraterizes superir sulus tumrs and an delineate inltratin f the adjaent mediastinal fat and ther strutures Fig. 65.3. Magneti resnane imaging (MRI) is mre aurate

than CT fr determining the resetability f a superir sulus tumr wing t its superir ability t evaluate tumr extensin int the vertebral bdy, neural framina, spinal rd, and brahial plexus. Psitrn emissin tmgraphy (PET)-CT allws fr detetin f ndal and distant metastases and is thus helpful fr staging. Like ther nn-small ell lung aners, superir sulus tumrs are staged using the TNM staging system. Operable andidates are usually treated with preperative radiatin and hemtherapy fllwed by surgery. REFERENCES

Bruzzi JF, Kmaki R, Walsh GL, etal. Imaging f nn–small ell lung aner f the superir sulus. Part 1: anatmy, linial manifestatins, and management. Radiographics. 2008;28:551–560. Thrai Radilgy: The Requisites, 3rd ed, 405–427.

CASE 66 Tuberculosis (Post-Primary Type) 1. A, B, and C. The mputed tmgraphy (CT) images shw a large avitary lesin in the right lung. TB (pst-primary pattern), fungal pneumnia, and primary lung aner are all pssible diagnses. Pulmnary infarts are usually smaller, invlve a peripheral prtin f the lung parenhyma abutting the pleural surfae, and seldm avitate unless infeted. 2. D. Squamus ell arinma is the mst frequent lung aner ell type assiated with avitatin. 3. C. Tree-in-bud paities were rst desribed in the setting f TB. They indiate endbrnhial spread f disease int the small airways (brnhiles). Typially, the brnhiles are nt visualized n hest CT unless there is impatin f material (pus, bld, tumr, et.) within them. 4. A. There is a dual arterial bld supply t the lungs mprised f the pulmnary and brnhial arteries. Lalized disruptin f pulmnary arterial w urs with pulmnary emblism. If there is superimpsed ngestive heart failure, the brnhial arterial supply is als affeted. This mbinatin inreases the likelihd f develpment f fal pulmnary infart within the territry f the emblism.

Comment The terms “primary” and “pst primary” tuberulsis are inaurate. DNA ngerprinting has demnstrated that in adults, there is n signiant differene in the imaging features between reently aquired tuberulsis and remtely aquired tuberulsis. Hwever, knwledge f the patterns assiated with the traditinal lassiatin is still imprtant. The primary pattern f tuberulsis manifests as nslidatin, ften in the mid r lwer lung znes, with lymphadenpathy. This pattern is mre mmn in hildren and in human immundeieny virus (HIV) psitive patients with lw CD4 unts. The pst primary pattern f tuberulsis is haraterized by upper lung paities ften with assiated avitary r bravitary disease. Chest mputed tmgraphy (CT) axial image demnstrates a avitary lesin with assiated fal areas f tree-in-bud paities in the right lung. The patient had knwn tuberulsis. Based n imaging ndings, this wuld be mst nsistent with a pst primary pattern f tuberulsis. Tree-in-bud paities suggest endbrnhial spread f disease. Thugh riginally desribed in the setting f tuberulsis, and thught t be pathgnmni fr this nditin, we nw knw that this imaging pattern is quite mmn in ther inammatry nditins invlving the small airways (brnhiles) inluding aspiratin and nn-tuberulus mybaterial pneumnia. Any nditin, whih leads t impatin f the brnhiles an prdue the tree-in-bud pattern. Rarely, the tree-in-bud pattern is due t small metastati fi

Fair Game

within pulmnary arteries, whih mirrr the branhing pattern f the airways with whih they are in prximity. REFERENCE

Jeng YJ, Lee KS. Pulmnary tuberulsis: up-t-date imaging and management. AJR Am J Roentgenol. 2008 Sep;191(3):834–844.

CASE 67

anatmi-metabli rrelatin. Reent data indiates that diffusin weighted images (DWI) n MRI ffer better speiity than PET-CT. REFERENCES

Kligerman S, Abbtt G. A radilgi review f the new TNM lassiatin fr lung aner. AJR Am J Roentgenol. 2010;194:562–573. Thrai Radilgy: The Requisites, 3rd ed, 405–427.

Lymphadenopathy From Lung Cancer

CASE 68

1. A, B, D, and E. The subarti (AP windw), lw paratraheal, para-arti, and hilar lymph ndes are all FDG avid nsistent with ndal metastases in these statins. FDG PET CT image shws that the right upper lbe ndule, the paratraheal, para-arti, and prevasular lymph ndes are all FDG-avid, nsistent with ndal metastases (Figs. 67.1 and 67.2). FDG PET CT image mre inferirly demnstrates a highly FDG-avid right hilar lymph nde (Fig. 67.3). Para-arti and prevasular lymph ndes are nt enlarged. N FDG avid high paratraheal lymph ndes are visualized in the prvided images. The distintin between high paratraheal and lw paratraheal lymph ndes is the tp f the arti arh. 2. C. Cntralateral mediastinal lymphadenpathy (N3 disease) makes this patient’s disease stage III B whih is nsidered unresetable. Islated T3 r N2 disease is ptentially resetable. Obviusly, metastati disease is a ntraindiatin fr surgery (M1) but there is n denite evidene f distant metastases in the prvided images. 3. A. The right upper lbe is mst mmnly affeted by lung aner. Interestingly, in the setting f pulmnary brsis, lung aner is mst ften lated within the lwer lbes. 4. D. Spiulated margins have a high assiatin with lung aner and wuld prdue a true psitive result with PET/CT in mst ases. Lesins smaller than 1 m may be difult t image reliably using PET imaging unless fal metabli ativity is high. Carinid tumrs ften have lw FDG avidity beause f their slw grwth rate. Lung aners with grund glass attenuatin (typially adenarinma) are assiated with lw FDG ativity.

Subsolid Nodule (Adenocarcinoma) 1. A, B, C. and D. The magnied CT image in Fig. 68.1 demnstrates a fal grund-glass ndule within the superir segment f the left lwer lbe. All f the listed nsideratins may manifest in this manner. On sagittal r rnal refrmats, sar may be distinguished as having a at r linear nguratin. 2. B. A mixed grund-glass and slid pulmnary ndule is mst nerning fr primary lung aner (mst ften adenarinma) (Fig. 68.2). Slid ndules with entral avitatin may be malignant but are frequently f infetius rigin (e.g., septi embli, fungal pneumnia) r inammatry (e.g., granulmatsis with plyangiitis, rheumatid ndules). 3. D. Pure grund-glass ndules measuring 6 mm r smaller require n further dediated CT fllw-up per urrent Fleishner Siety guidelines (see Referene). 4. B. Centrilbular grund-glass ndules are mmnly fund in the setting f hypersensitivity pneumnitis, respiratry brnhilitis, and pulmnary hemrrhage. Metastati disease presenting with suh a CT pattern wuld be highly unusual.

Comment On thin-setin CT, a malignant slitary pulmnary ndule (SPN) may be f sft tissue (slid), pure grund-glass attenuatin, r a mbinatin f bth grund-glass and slid mpnents (i.e., part-slid r mixed attenuatin). Of thse patterns, the part-slid ndule is the mst likely t be malignant (Fig.68.2),

Comment Lymph nde enlargement is a mmn ause f a mediastinal r hilar mass and shuld be suspeted whenever a spherial r vid mass r masses are identied within a knwn anatmi lymph nde latin. There are a variety f infetius, inammatry, and neplasti auses f thrai lymph nde enlargement. Neplasti etilgies inlude primary lung aner, metastati disease, and lymphma. Bth CT and magneti resnane imaging (MRI) rely n the anatmi features f lymph ndes, mst ntably lymph nde size (shrt axis larger than 1 m), t distinguish between malignant and benign lymph ndes. This strategy is limited by a lw sensitivity and speiity. Thus, in patients with primary lung aner, enlarged ndes must be bipsied fr staging purpses. FDG-PET imaging, whih relies upn metabli (gluse metablism) rather than anatmi features, is the mst aurate nninvasive imaging test fr assessing mediastinal lymph ndes. False negatives are unmmn and usually related t small lymph nde size (mirspi metastases.) Hwever false psitives are mre frequent beause f the FDG avidity seen in inammatry ndes. The auray f FDG-PET an be further enhaned by using an integrated PET-CT sanner, whih imprves visual quality and quantitative auray f PET images, while ptimizing

187

Fig. 68.2

188

SECTION IV

Answers

and the sft tissue attenuatin ndule is the mst likely t be benign. Arding t urrent guidelines, slitary subslid ndules require n further imaging fllw up if they measure less than 6 mm. A grund-glass ndule larger than 6 mm an be reevaluated with CT at 6 t 12 mnths t nrm persistene. If it persists, annual CT fllw-up every 2 years fr a minimum f 5 years is then suggested. Part-slid ndules greater than r equal t 6 mm in ttal dimensin are fllwed by CT at 3 t 6 mnths t nrm persistene. If unhanged r if the slid mpnent remains retiular abnrmality)—the answer in this ase d. Cnslidatin in brnhpulmnary segment(s)/lbe(s) e. Prfuse mirndules (bilateral, predminantly upper lbes) f. Disrete ysts (multiple, bilateral, away frm areas f hneymbing) g. Diffuse msai attenuatin/air-trapping (bilateral, in three r mre lbes) 3. C. UIP is mst mmnly basilar and peripheral in distributin. It is haraterized by retiular paities whih extend t the pleural surfae. Subpleural sparing wuld be highly unusual in UIP and muh mre supprtive f a NSIP diagnsis. Hwever, a substantial minrity f UIP (up t 25%) may nt be basilar predminant in znal distributin. 4. D. NSIP is ften seen in llagen vasular disease, as a drug reatin r it may be the histlgi pattern identied by the pathlgist in sme patients with hypersensitivity pneumnitis. Idipathi NSIP is muh less mmn.

Comment Nnspei interstitial pneumnitis (NSIP) is lassied as ne f the idipathi interstitial pneumnias. Pathlgially, it is haraterized by temprally and spatially hmgeneus interstitial inammatin with r withut brsis. Traditinally, NSIP has been ategrized as either ellular, brti, r mixed ellular and brti. On CT, NSIP is typially basilar predminant but has a variable pattern in the axial plane (peripheral, diffuse, r entral). Grund-glass paity is almst always present with variable degrees f superimpsed retiulatin and tratin brnhietasis as well as brnhiletasis. Often the degree f tratin brnhietasis appears t be ut f prprtin t the degree f nmitant lung disease. A signiant degree f subpleural hneymbing is unusual. Hwever, a small minrity f ases f NSIP eventually prgress t UIP. Therefre a mbined NSIP and UIP CT pattern may asinally be bserved. In ntrast t UIP, whih is mst ften idipathi, NSIP is almst always sendary t an underlying nditin--mst mmnly llagen vasular disease, mediatin/drugs, r hypersensitivity pneumnitis (HP). The esphagus is dilated in this ase, nsistent with esphageal dysmtility in this patient with underlying histry f llagen vasular disease. The main imaging differential diagnsis fr NSIP inlude UIP and HP. Differentiatin f NSIP frm UIP and HP may be difult if nt impssible in sme ases. The mst helpful differentiating ndings f NSIP frm UIP are substantial grund glass abnrmality and subpleural sparing. A substantial degree f grund-glass paity in UIP withut a superimpsed nditin wuld be unusual. In additin, subpleural sparing in UIP. UIP lassially riginates in the subpleural lung and prgresses mre entrally. In many ases f brti HP, the distributin f lung disease is upper lung predminant, whih wuld be unusual in UIP and rare in NSIP. Als, there is very ften substantial air trapping in HP, whih wuld be unusual in islated UIP r NSIP. In patients with NSIP features f llagen vasular disease may be evident n imaging inluding esphageal dilatin (esphageal dysmtility), pulmnary arterial dilatin (pulmnary hypertensin), and pleural r periardial effusin/ thikening.

Fair Game

REFERENCES

Kelsh TL, Chung JH, Lynh DA. Radilgi evaluatin f idipathi interstitial pneumnias. Clin Chest Med. 2015 Jun;36(2): 269–282. Thrai Radilgy: The Requisites, 3rd ed, 355–376.

CASE 75 Ascending Aortic Aneurysm 1. A, B, C, and D. All f the listed nditins are ptential auses fr the develpment f asending arti aneurysms. 2. D. Typially, thrai arti aneurysms are nsidered fr repair when they are larger than 5 t 6 m r are grwing at a rapid rate. 3. D. Biuspid arti valves are assiated with asending arti aneurysms, intraranial arterial aneurysms, Turner syndrme, and arti artatin. Partial anmalus pulmnary venus return is nt mmnly assiated with biuspid arti valves. 4. B. The desriptin f the imaging ndings is nsistent with annularti etasia, whih is mst ften assiated with Marfan syndrme r Ehlers-Danls syndrme.

Comment An aneurysm is dened as an abnrmal dilatin f a vessel. Regarding the asending arta, there is sme variability in diameter with inreasing age, but a diameter f greater than 4 m is generally nsidered abnrmal. Axial image frm ntrast enhaned hest mputed tmgraphy (CT) shws a very large asending arti aneurysm measuring 5.7 m. A small right pleural effusin is nted. Aneurysms may be lassied based n the integrity f arta wall (true vs. false), latin, and shape. Regarding shape, fusifrm aneurysms are haraterized by ylindrial dilatin f the entire irumferene f the arta, and saular aneurysms are haraterized by a fal utpuhing f the arta. Fusifrm aneurysms are mst assiated with atherslersis, whereas saular aneurysms are mst ften traumati r infetius in etilgy. Thrai arti aneurysms are less mmn than abdminal arti aneurysms. Up t ne furth f thrai arti aneurysms exist with an abdminal arti aneurysm Althugh aneurysmal dilatin f the asending arta is frequently aused by atherslersis, this press usually invlves ther prtins f the arta as well. Annularti etasia refers t the presene f dilated sinuses f Valsalva with effaement f the sintubular juntin, resulting in a pear-shaped asending arta that tapers t a nrmal- aliber arti arh. This disrder may be idipathi r assiated with nnetive tissue disrders suh as Ehlers-Danls and Marfan’s syndrmes. Syphilis, ne a relatively mmn ause f asending arti aneurysms, is nw rare. Asending arti aneurysms may als ur in the setting f biuspid arti valves, irrespetive f any funtinal valvular w abnrmality (stensis r regurgitatin) suggesting an inherent weakness f the arti wall in these patients. The majr mpliatin f aneurysms is rupture. The risk fr rupture is related t the size f the aneurysm. Fr this reasn, eletive surgial repair is generally remmended when aneurysms exeed 5 t 6 m in diameter. REFERENCES

Agarwal PP, Chughtal A, Matzinger F, Karerni EA. Multidetetr CT f thrai arti aneurysms. Radiographics. 2009;29:537–552. Thrai Radilgy: The Requisites, 3rd ed, 97–136.

191

CASE 76 Carcinoid Tumor 1. D. Cmputed tmgraphy (CT) images demnstrate a fal endbrnhial tumr in the prximal right lwer lbe brnhus with a fus f arse peripheral aliatin. In a yung patient, arinid tumr is the mst likely diagnsis. 2. B. The mst mmn benign pulmnary tumr is a hamartma. The ther hies are less mmn benign tumrs f the lung. 3. C. Apprximately 30% f arinid tumrs demnstrate aliatin. 4. D. DIPNECH urs almst exlusively in middle-aged r elderly wmen. On CT, the mbinatin f small, welldened pulmnary ndules (representing arinid tumrs and tumrlets) in the mid and lwer lung znes and substantial air trapping is suggestive f the diagnsis, espeially in a patient with refratry asthma-like symptms.

Comment Brnhial arinid tumrs are unmmn neurendrine neplasms that ur entrally (80%) mre mmnly than peripherally (20%). Affeted patients are usually in the third t seventh deade f life and typially present with ugh, hemptysis, and pstbstrutive pneumnia. On hest radigraphs, arinids typially appear as a entral, hilar, r perihilar mass that may be assiated with pstbstrutive ateletasis, pneumnia, muid impatin, r brnhietasis. On CT, arinids typially demnstrate well-dened margins and slightly lbulated brders. Carinids are usually lated lse t the entral brnhi, usually near airway bifuratins. Caliatin is bserved in apprximately 30% f ases n CT but is nt usually evident n nventinal radigraphs. Mst lesins demnstrate intense ntrast enhanement. A minrity f arinids present as a slitary pulmnary ndule (SPN) in the periphery f the lung. Typial arinid tumrs in the periphery f the lungs usually grw at a slw rate. Atypial arinids, whih aunt fr 10% f all arinids, ur mst ften in the lung periphery. These lesins are usually large at the time f presentatin and grw at a faster rate than typial arinids. Althugh typial arinids rarely metastasize, atypial arinids exhibit metastases in up t half f patients. Therapy f arinid tumrs nsists f surgial resetin, with a mre aggressive surgial apprah fr atypial lesins. Adjuvant hemtherapy has als been emplyed with sme suess in patients with advaned atypial arinid tumrs. Typial arinids have an exellent prgnsis, with a 5-year survival f apprximately 90%. In ntrast, atypial arinids are assiated with a 5-year survival f apprximately 70%. Beause arinid tumrs have a high number f smatstatin reeptrs, sintigraphi imaging with the radilabeled smatstatin analgue tretide may be helpful fr deteting ult tumrs. Cnversely, urdexygluse–psitrn emissin tmgraphy (FDG-PET) imaging is less useful in this setting beause f a high rate f false-negative results fr typial arinid tumrs. REFERENCES

Bensn RE, Rsad-de-Christensn ML, Martínez-Jiménez S, Kunin JR, Pettavel PP. Spetrum f pulmnary neurendrine prliferatins and neplasms. Radiographics. 2013 Ot;33(6):1631–1649. Little BP, Junn JC, Zheng KS, Sanhez FW, Henry TS, VeeraraghavanS, Berkwitz EA. Diffuse idipathi pulmnary neurendrine ell hyperplasia: imaging and linial features f a frequently delayed diagnsis. AJR Am J Roentgenol. 2020 De;215(6):1312–1320. Thrai Radilgy: The Requisites, 3rd ed, 391–404

192

SECTION IV

Answers

CASE 77 Boerhaave Syndrome 1. A, B, and C. The hest radigraph (Fig. 77.1) demnstrates a large amunt f etpi gas entered within the inferir aspet f the mediastinum. Small pleural effusins with bibasilar ateletasis are als present. Axial ntrast enhaned hest mputed tmgraphy (CT) image (Fig. 77.2) demnstrates a large amunt f pneummediastinum arund the distal esphagus and arta as well as small pleural effusins in this patient with Berhaave syndrme. The differential diagnsis fr pneummediastinum inludes freful exhalatin against a lsed glttis, asthma, esphageal r large airway injury, and extensin f retrperitneal r retrpharyngeal gas. Extensin f air frm a pneumthrax int the mediastinum is nt mmn. The mst likely diagnsis given the nentratin f gas arund the dilated esphagus is esphageal rupture in this patient with histry f vmiting, the Berhaave syndrme. 2. B. Pneummediastinum in the setting f asthma is thught t be due t the Maklin effet. In the setting f small airway bstrutin, alvelar rupture allws gas t trak alng the pulmnary interstitium entrally int the hila and eventually the mediastinum. 3. D. Esphageal rupture in the setting f Berhaave syndrme typially urs in the left lwer aspet f the esphagus. 4. C. Mediastinitis is the mst serius mpliatin f esphageal rupture with high mrtality reprted. Treatment ranges frm medial management t endspi therapy and/r surgery depending n the severity f the rupture and whether it is ntained. Hwever, in all ases, nthing shuld be allwed by muth and the patient shuld be plaed n brad-spetrum intravenus antibitis, prtn pump inhibitr, and any lalized uid lletins drained. Empyema partiularly in the left pleural spae is nt unmmn.

Comment Esphageal perfratin is a mmn ause f aute mediastinitis and may ur sendary t a variety f mehanisms. Berhaave syndrme refers t transmural perfratin f the distal esphagus that results frm repeated episdes f vmiting. Rupture typially urs psterirly, near the left diaphragmati rus. Patients with esphageal perfratin typially present with symptms f fever, leukytsis, dysphagia, and retrsternal hest pain, whih ften radiates int the nek. Pneummediastinum is a frequent hest radigraphi nding as in this ase. It an be subtle, and air may be difult t appreiate n the standard hest radigraph. Air r gas in the sft tissues f the nek an be a helpful sign. Additinal hest radigraphi ndings may inlude diffuse mediastinal widening, pneumthrax, pleural effusin, and empyema. When the diagnsis f esphageal perfratin is delayed, additinal mpliatins may inlude mediastinal absess, esphagpleural stula, and esphagbrnhial stula. A diagnsis f suspeted esphageal perfratin an be made fllwing the administratin f water-sluble ntrast medium. Suh a study demnstrates extravasatin f ntrast at the site f perfratin, but false negatives ur in up t 10% f ases. CT may be helpful in ases fr whih urspy is nndiagnsti. It may als be helpful t delineate the latin and extent f uid lletins in ases that have prgressed t mediastinal absess frmatin and empyema. It is imprtant t be aware A delay f greater than 24 hurs in the diagnsis f this mpliatin

is assiated with high mrbidity and mrtality rates. Thus, prmpt diagnsis and treatment are ritial. REFERENCES

Yung CA, Menias CO, Bhalla S, Prasad SR. CT features f esphageal emergenies. Radiographics. 2008 Ot;28(6):1541–1553. Katabathina VS, Restrep CS, Martinez-Jimenez S, Riass RF. Nnvasular, nntraumati mediastinal emergenies in adults: a mprehensive review f imaging ndings. Radiographics. 2011 Jul–Aug;31(4):1141–1160. Nrtn-Gregry AA, Kulkarni NM, O’Cnnr SD, Budve JJ, Zrn AP, Desuhes SL. CT esphaggraphy fr evaluatin f esphageal perfratin. Radiographics. 2021 Mar–Apr;41(2):447–461. Thrai Radilgy: The Requisites, 3rd ed, 97–136, 226–237.

CASE 78 Bronchiectasis 1. A, B, and C. The axial CT image demnstrates ndings nsistent with brnhietasis. The differential diagnsis wuld inlude nntuberulus mybaterial pneumnia, allergi brnhpulmnary aspergillsis, and ysti brsis. There is n evidene f pulmnary brsis t suggest usual interstitial pneumnitis. Tratin brnhietasis is a feature f pulmnary brsis but ther ndings f brsis suh as retiulatin, arhitetural distrtin, and/r hneymbing are absent. 2. D. In the setting f severe brnhietasis, differentiatin f brnhietasis frm ysti lung disease may be difult. A helpful CT pstpressing tehnique is minimum intensity prjetin. With the use f thik minimum intensity prjetin slies, the ntinuity f ysti air spaes with mre nrmal appearing entral brnhi is readily apparent in ases f brnhietasis. 3. A. Cylindrial, variid, and ysti (saular) brnhietasis are all subtypes f brnhietasis Spherial brnhietasis isnt a subtype. 4. B. Up t 5% f adults with ysti brsis will develp hepati irrhsis. With the inrease in life span f patients with ysti brsis, mrbidity and mrtality frm hrni liver disease is likely t inrease.

Comment The term bronchiectasis refers t abnrmal, irreversible dilatin f the brnhi. The denitive pathlgi desriptin f brnhietasis was reprted by Reid and is based n the mrphlgy f the brnhi and the number f brnhial subdivisins that are present. In ylindrial brnhietasis, the brnhi are minimally dilated and have a straight, regular ntur (Fig. 78.2). The average number f brnhial subdivisins frm the hilum t the lung periphery is 16 (17 t 20 is nrmal). In variid brnhietasis, the brnhi demnstrate a beaded appearane with sequential dilatatin and nstritin (Fig. 78.3). The average number f brnhial divisins is 8. In ysti brnhietasis, the brnhi have a ballned appearane (Fig. 78.4). The average number f brnhial divisins is nly 4. Brnhietasis an be distinguished frm ysti lung disease by applying the fllwing imaging riteria: 1. When dilated brnhi urse perpendiular t the sanning plane, a pulmnary artery r arterile will always urse adjaent t it (signet ring sign). In ntrast, true lung ysts, suh as thse assiated with LAM, are lated randmly in the lung parenhyma. 2. Dilated brnhi urse parallel t the sanning plane, yu will bserve that the ysti spaes nnet with ne anther. 3. Cysti brnhietasis is ften assiated with air-uid levels, a nding that is nt generally bserved in ysti lung disease.

Fair Game

193

Fig. 78.2

Fig. 78.4

REFERENCES

Millirn B, Henry TS, Veeraraghavan S, Little BP. Brnhietasis: Mehanisms and imaging lues f assiated mmn and unmmn diseases. Radiographics. 2015 Jul–Aug;35(4):1011–1030. Thrai Radilgy: The Requisites, 3rd ed, 137–158.

CASE 79 Giant Bulla 1. A and B. The imaging ndings are highly suggestive f very large bulla. Althugh, a right-sided pneumthrax uld mimi this appearane. Hwever, the absene f a distint viseral pleural line as well as gradual attenuatin f lung vessels at the juntin f the superir aspet f the right lung and mid aspet f the right lung are muh mre suggestive f a bulla rather than a pneumthrax. A prtin f the inferir wall f the bulla an be identied inferirly and medially. 2. B. A bulla is dened as a gas ntaining spae in the lung ≥ 1m with a thin well-dened wall. Bth bullae and blebs represent subpleural ysti air spaes, whih are usually well-dened. 3. D. Of the listed mdalities, hest CT is the mst sensitive tl t detet early emphysema. 4. D. Althugh basilar predminant panlbular emphysema is almst always assiated with alpha-1 antitrypsin deieny, in a minrity f patients, previus IV Ritalin use is the underlying ause f this pattern.

Comment Fig. 78.3

The hest radigraph in this ase demnstrates a large area f absent pulmnary vessels in the right lung with sparing f the right

194

SECTION IV

Answers

base It displaes the remainder f the lung inferirly and appears t have a well-dened wall inferirly and medially. A large pneumthrax is a nsideratin. Hwever, the absene f a viseral pleural line and gradual transitin frm the nrmal lung parenhyma t an area f absent pulmnary vessels demarated by a wall indiates the presene f a large bulla rather than a pneumthrax. Bullae may develp in assiatin with any type f emphysema, but they are mst mmnly assiated with paraseptal and entrilbular emphysema. Hwever, they are nt always assiated with diffuse emphysema. Bullae usually enlarge ver mnths t years, but the grwth rate is quite variable. Oasinally, bullae an beme quite large and may be fal in distributin. Large bullae may mprmise respiratry funtin. The resulting syndrme has been referred t by varius terms, inluding bullus emphysema, vanishing lung syndrme, and primary bullus disease f the lung. This entity urs mst ften in yung men and is haraterized by large, prgressive upper lbe bullus disease. Althugh it may ur in nnsmkers, mst affeted patients are smkers. Cmputed tmgraphy (CT) is the preferred mdality fr the assessment f patients with suspeted bullus emphysema. CT is helpful fr delineating the number, size, and latin f bullae. It an als assess the degree f mpressin f underlying nrmal lung and determine the presene and severity f emphysema in the remaining prtin f the lung parenhyma. In symptmati patients, surgial resetin f bullae an result in marked imprvement in pulmnary funtin. The greatest benet frm surgery is bserved in patients with a large bulla (upying 50% r mre f a hemithrax) and a mderate redutin in fred expiratry vlume in 1 send (FEV1). In ntrast, patients with severe generalized emphysema tend t d prly and are thus nt ideal andidates fr bulletmy. REFERENCES

Lynh DA, Austin JH, Hgg JC, Grenier PA, Kauzr HU, Bankier AA, Barr RG, Clby TV, Galvin JR, Gevenis PA, Cxsn HO, Hffman EA, Newell Jr JD, Pistlesi M, Silverman EK, Crap JD. CT-denable subtypes f hrni bstrutive pulmnary disease: a statement f the Fleishner Siety. Radiology. 2015 Ot;277(1):192–205. Thrai Radilgy: The Requisites, 3rd ed, 19–60, 391–404.

CASE 80 RML and RLL atelectasis 1. D. The psteranterir (PA) radigraph demnstrates a fal paity in the entral and inferir aspet f the right lung with bsuratin f the right hemidiaphragm and right heart brder. The upper brder is sharply dened, and that brder extends medial t the interlbar artery in the right hilum. The upper brder is prdued by the depressed minr ssure laterally and the majr ssure medially (arrw) (the minr ssure an never extend medial t the lateral brder f the interlbar artery s the lwer lbe must be llapsed.) There is mild right lung vlume lss suggested by rightward shift in the mediastinum and trahea. These ndings are typial f mbined right middle and right lwer lbe llapse. 2. C. Obstrutin f the brnhus intermedius wuld ause mbined right middle and right lwer lbe ateletasis. The brnhus intermedius arises frm the right mainstem brnhus distal t the takeff f the right upper lbe brnhus. 3. D. Althugh all listed answer hies uld ause mplete right lung ateletasis, in a reently intubated patient, muus plugging is mst likely. If the endtraheal tube is advaned t far, it mst ften extends int the right mainstem brnhus rather than the left mainstem brnhus due t the right mainstem brnhus’s mre btuse angulatin

relative t the trahea; this wuld result in left lung ateletasis rather than right lung ateletasis. In the utpatient setting, mplete left lung ateletasis is very sendary t a entral lung arinma. 4. D. Hepatellular arinma des nt mmnly metastasize t the entral airways. Cmmn distant primary malignanies whih metastasize t the trahea and entral brnhi inlude melanma, renal ell arinma, breast aner, and ln aner.

Comment The radigraphi appearane is typial f mbined right middle and lwer lbe llapse (Fig. 80.1). There is a well-dened paity in the entral aspet f the right lung with assiated silhuetting f the right hemidiaphragm and right heart brder with right lung vlume lss. The sharp upper brder is prdued by the minr and majr ssures whih are bth depressed. Cmbined right middle and lwer lbe llapse an ur when a tumr bstruts the brnhus intermedius. This mbinatin urs mre frequently than mbined right upper and right middle lbe llapse beause the brnhi t these lbes are remte frm ne anther. When the latter mbinatin urs, the appearane is idential t left upper lbe llapse. In this patient, the mbined lbar llapse urred sendary t endbrnhial metastati disease. Als nte the presene f pulmnary metastases, best visualized in the left lung. Endbrnhial metastases are unmmn and are fund in less than 5% f patients at autpsy. Presenting symptms may inlude ugh, wheeze, and hemptysis. Cughing may infrequently result in expetratin f tumr fragments; rarely, this is the rst indiatin f metastati disease. Radigraphi ndings in the setting f partial airway bstrutin inlude ligemia and air trapping. In the setting f mplete brnhial bstrutin, ndings inlude lbar, segmental, r subsegmental ateletasis and pstbstrutive pneumnitis. Ahilar r entral mass may als be evident. REFERENCES

Se JB, Im JG, G JM, etal. Atypial pulmnary metastases: spetrum f radilgi ndings. Radiographics. 2001;21:403–417. Thrai Radilgy: The Requisites, 3rd ed, 19–60.

CASE 81 Lipoma 1. A and C. Axial image frm a nn-ntrast hest CT demnstrates a large fatty mass in the left axilla. Crnal nnntrast T1-weighted MRI demnstrates diffuse hyperintense signal thrughut the mass whih extends superirly int the nek harateristi f a lipma. The imaging ndings are essentially diagnsti f a lipma. There is a small amunt f wispy sft tissue within this lesin; therefre, lipsarma uld be nsidered in the differential diagnsis but is muh less likely. 2. D. On T1 weighted MRI, nn-prteinaeus r hemrrhagi uid is usually T1 hypintense. The ther listed materials (fat, melanin, methemglbin) are all T1 hyperintense. 3. C. The greater degree f sft tissue within a fatty mass, the greater the likelihd f lipsarma. 4. C. Lipmatus hypertrphy f the interatrial septum represents a benign fatty inltratin f the atrial septum. This lesin is typially dumbbell shaped that lassially spares the fssa valis. Thugh this lesin is usually asymptmati, it may be a ause f symptmati arrhythmias and vasular bstrutin. In 80% f patients, there is inreased FDG uptake n PET-CT, likely beause f metablially ative brwn fat.

Fair Game

Comment The CT and MRI images demnstrates a fatty mass in the left axilla that ntains a minimal amunt f sft tissue attenuatin material. T1 hyperintensity is typial f fatty lesins. The mass extends int the lwer aspet f the left nek. Lipmas may ur in a variety f latins in the thrax, inluding the mediastinum, hest wall, extrapleural spae, esphagus, heart, airway, and rarely, the lung parenhyma. Althugh lipmas typially appear as well-marginated lesins haraterized by hmgeneus fat attenuatin, sft tissue elements may be bserved. In suh ases, it may nt be pssible t distinguish lipma frm thymlipma r lw-grade lipsarma. The pliability and lak f invasiveness f lipmas may aid in their differentiatin frm lipsarmas; fr example, lipmas typially drape arund adjaent vessels, ribs, and mediastinal strutures withut invading them. Lipsarmas typially ntain a larger sft tissue mpnent, have irregular margins, and frequently invade adjaent mediastinal and hest wall strutures. Thus, the presene f well-dened margins and lak f invasiveness favr a diagnsis f lipma ver lipsarma. REFERENCES

Gaerte SC, Meyer CA, Winer-Muram HT, etal. Fat-ntaining lesins f the hest. Radiographics. 2002;22:S61–S78. Thrai Radilgy: The Requisites, 3rd ed, 159–192.

CASE 82 Apical Cap Secondary to Extrapleural Hematoma 1. A and D. Bld frm interstal vessel injury, rib frature, r disseting mediastinal hematma (ruptured arta) an aumulate between the parietal pleura and ribs, frming an apial ap. Mst aute hemthraes aumulate dependently. Hwever, pleural bld uld ause an apial ap if the patient had pre-existing pleural adhesins. Pulmnary ntusin manifests as lung nslidatin r grund-glass paity in a nnanatmi distributin. Asbests-related pleural thikening usually extends t the stphreni angle. The apial pleura is typially spared. The ap in this ase is lnger in dimensin than a lassi pleural plaque. 2. C. Bne destrutin adjaent t a pleural ap is mst suggestive f a primary lung arinma (superir sulus tumr). Hilar lymphadenpathy and pleural effusin an be aused by extrathrai tumr metastases r inammatin. Mediastinal widening may be prdued by infetin (mediastinitis) r hematma bth f whih an trak int the apial extrapleural spae. 3. C. Many patients develp apial brsis and pleural sarring with age. Mst f these patients are asymptmati. The apial aps usually have irregular margins, are less than 5 mm thik, and are typially symmetri. The prevalene inreases with age. Apial aps frm extensin f mediastinal hematma are typially unilateral and mre mmnly ur n the left. These may be the result f a traumati arti injury. Radiatin fr breast arinma typially is limited t the anterir hest wall. Radiatin t these areas an auses brsis in the adjaent ipsilateral lung apex and result in a unilateral apial ap. Head and nek arinmas ften metastasize t ervial and supralaviular lymph ndes and an prdue an apial ap when the lymph nde metastases are large. These metastases are mre likely t be unilateral than bilateral. Als, radiatin t the ipsilateral nek in suh ases may als prdue an apial ap.

195

4. A. CT f the hest an best haraterize the abnrmality and delineate its full extent. Althugh MRI an prvide a lt f infrmatin, a CT san is a better rst-line test t further haraterize the abnrmality beause f its superir spatial reslutin and ability t rapidly image the entirety f the hest and nek. Ultrasund may be useful t assess pleural and extrapleural abnrmalities, but it is limited in its ability t fully delineate the extent f disease. Furthermre, deep tissues may nt be visible. FDG-PET des nt have a rle in evaluating the autely ill patient.

Comment Differential Diagnosis AP radigraph (Fig. 82.1) shws a left apial ap. There is a subtle displaed frature f the left rst rib, indiating that extrapleural hematma r lulated hemthrax is the mst likely ause. Crnal refrmatted ntrast-enhaned CT image (Fig. 82.2) nrms the presene f bld in the upper left hemithrax. The bld abuts the inner margin f the adjaent ribs, indiating an extrapleural latin. The relatively smth margins and the presene f an aute rib frature argue against neplasm.

Discussion The term apical cap has been used t desribe the presene f paity lated in r adjaent t the extreme apex f the lung n hest radigraphs (see Fig. 82.1). On hest radigraphs f nrmal, asymptmati patients, biapial smth r irregular paities lated at the apies f the lungs, usually measuring less than 5 mm in diameter, are ften present. The lwer margin is usually sharply delineated but ften has an undulating brder. Apial aps are thught t represent the result f nnspei subpleural sarring and apial pleural thikening, and they are usually f n linial signiane. The prevalene f apial aps inreases with age. This nding when islated des nt represent ld healed granulmatus disease (i.e., tuberulsis). A variety f entities an result in an enlarged apial ap. The varius auses f a unilateral enlarged ap inlude lymphma r absess extending dwn frm the nek, primary lung arinma (superir sulus r Panast tumr), and mediastinitis r hematma traking int the apial extrapleural spae. Regarding bilaterally enlarged apial aps, they may be assiated with radiatin brsis (e.g., fr Hdgkin lymphma), mediastinal lipmatsis, and vasular abnrmalities suh as artatin f the arta with enlarged llateral vessels. In this ase, the presene f a smthly marginated enlarged left apial ap is the result f an extrapleural hematma. The apial ap is smthly marginated, reeting the extrapleural latin. Extrapleural hematmas typially d nt require treatment. Ative interstal arterial bleeding may be treated with emblizatin. It is imprtant t distinguish hemthrax frm extrapleural hematma, as the frmer usually requires perutaneus drainage t avid the subsequent develpment f a brthrax. REFERENCES

MLud TC, Isler RJ, Nvelline RA, etal. The apial ap. AJR Am J Roentgenol. 1981;137:299–306. Thrai Radilgy: The Requisites, 3rd ed, 159–192.

196

SECTION IV

Answers

CASE 83

CASE 84

Bleomycin Lung Toxicity

Primary Lung Adenocarcinoma With N2 Nodal Disease

1. A and D. Blemyin lung txiity, and fungal infetin an present as pathy nslidatin in a patient n hemtherapy. Cngestive heart failure typially presents with diffuse lung edema and pleural effusins. Cllagen vasular disease is usually nt assiated with peripheral nslidatin. 2. A. Apprximately 4% f patients reeiving blemyin develp lung txiity. 3. A. Althugh blemyin is used t treat sme patients with lymphma, lymphma itself des nt inrease a patient’s risk f blemyin indued lung txiity. Cnurrent raditherapy, renal insufieny, and advaned age all raise a patient’s risk fr develping blemyin indued lung txiity. 4. C. Mst patients imprve within a few weeks, nt a few days, althugh a small number f patients may take muh lnger t rever, inluding up t tw years. Blemyin must be disntinued t prevent wrsening lung injury, and mst patients are treated with rtisterids, whih are slwly tapered as the patient’s linial nditin imprves.

Comment Differential Diagnosis Axial and rnal mputed tmgraphy (CT) images shw pathy peripheral nslidatin in the right lung. Cnslidatin in a patient treated with hemtherapy an result frm infetin r drug reatin.

Discussion Blemyin is an antitumr agent that is used t treat lymphmas, testiular arinmas, and ertain squamus ell arinmas. Lung txiity urs in apprximately 4% f patients and is the prinipal dse-limiting fatr fr this agent. Fibrsis is the mst serius pulmnary mpliatin but rganizing pneumnia reatin and aute hypersensitivity reatin an ur. Affeted patients typially present with an insidius nset f dyspnea, nnprdutive ugh, and asinal fever. Pulmnary funtin tests reveal a dereased DLco, a sensitive measure fr early blemyin lung injury. Chest radigraphs may be nrmal r might shw retiular paities in a basilar and subpleural distributin. Peripheral and peribrnhial nslidatin an als develp. Thin setin CT is mre sensitive than radigraphs fr deteting lung abnrmalities and an shw harateristi ndings even when the hest radigraph is nrmal. Cnslidatin (Figs. 83.1 and Fig. 83.2) an represent drug-indued rganizing pneumnia, whih an als be mass like r ndular in appearane. Ndules an vary in size frm 5 mm t 3 m. Infetin, partiularly fungal infetin is the ther majr diagnsis t nsider. Organizing pneumnia is usually subpleural in a peripheral distributin and/r in a peribrnhial distributin. In ntrast t metastases, air brnhgrams are ften present within the ndules. Early detetin f blemyin-indued lung injury is imprtant beause prmpt disntinuatin f blemyin an result in imprved lung funtin and healing in patients with early stages f disease. In patients with mre-advaned disease, the prgnsis is variable. Althugh sme patients respnd t sterids, thers develp prgressive brsis, whih an lead t respiratry failure and death. REFERENCES

Rssi SE, Erasmus JJ, MAdams HP, et al. Pulmnary drug txiity: radilgi and pathlgi manifestatins. Radiographics. 2000;20:1245–1259. Thrai Radilgy: The Requisites, 3rd ed, 355–376.

1. A, B, and D. The radigraph shws right hilar and right paratraheal lymphadenpathy, whih an result frm primary lung arinma r metastases, inluding renal ell arinma and testiular arinma amng thers. Primary tuberulsis an als present with hilar and mediastinal lymphadenpathy, althugh this appearane is mre mmn in hildren than adults. Silisis an lead t hilar and mediastinal lymphadenpathy. Hwever, n underlying pneumniti ndules are present n the standard radigraph. Als, lymphadenpathy in silisis is usually bilateral in distributin. 2. C. N2 dentes metastasis t ipsilateral mediastinal r subarinal lymph ndes. N0 dentes n lymph nde metastasis, N1 dentes ipsilateral intrapulmnary, brnhial, r hilar lymph nde metastasis, and N3 dentes any supralaviular, salene, r ntralateral mediastinal r hilar lymph nde metastasis. 3. D. Cmbined with T1-3, N1 disease determines the designatin f stage IIIA. The patient is staged as IIIB when N2 disease urs with a T3 r T4 lesin. Any axillary lymph nde metastasis is staged as M1. In the absene f a T3 r T4 lesin and n distant metastases, sme patients with N2 lymph nde metastases may be andidates fr surgial resetin typially after neadjuvant hemtherapy. FDG-PET is sensitive fr lymph nde metastases and is superir t mputed tmgraphy (CT), but the false psitive rate is high. Surgial r brnhspi (EBUS) ndal bipsy is usually required fr adequate ndal staging. 4. C. Ipsilateral pleural metastasis is staged as M1a and is therefre a ntraindiatin fr resetin. Chest wall invasin is staged as T3 and is ptentially resetable. A tumr metastasis t the same lbe is staged as T3 and ptentially resetable. A entral lesin resulting in bstrutive pneumnia f the entire lung is staged as T2 and may be resetable.

Comment Differential Diagnosis The PA hest radigraph (Fig. 84.1) shws right hilar enlargement and a right lwer paratraheal mass, nsistent with lymphadenpathy. Cntrast-enhaned CT image shws right hilar (Fig. 84.2) and right lwer paratraheal (Fig. 84.3) lymphadenpathy. The differential diagnsis is brad but mst mmnly wuld inlude primary lung arinma, metastases (testiular and renal, espeially), lymphma, and granulmatus infetins suh as tuberulsis and histplasmsis.

Discussion In patients with nn–small ell lung aner, the ndal status prvides imprtant infrmatin fr determining prgnsis and planning apprpriate therapy. Lymph ndes are ategrized int seven spei znes: supralaviular, upper, artipulmnary, subarinal, lwer, hilar-interlbar, and peripheral. N hanges were made fr the N designatin in the 8th editin f the TNM lassiatin system. Arding t the TNM lassiatin system, ndal invlvement is graded frm N0 t N3 as fllws: • N0 = n demnstrable metastases t reginal lymph ndes • N1 = metastasis t lymph ndes in the ipsilateral peripheral r hilar-interlbar regins • N2 = metastasis t ipsilateral mediastinal ndes (upper, artipulmnary, lwer r subarinal)

Fair Game

• N3 = metastasis t any supralaviular ndes, r t ntralateral mediastinal (upper, artipulmnary, lwer), hilar-interlbar, r peripheral regins. CT and magneti resnane imaging (MRI) play an imprtant but limited rle in the assessment f the ndal status in patients with lung arinma. These imaging studies rely primarily n anatmi features f lymph ndes, mst ntably lymph nde size. Shrt axis diameter >1 m is generally nsidered abnrmal. This strategy is assiated with sensitivities in the range f 60% t 79% and speiities in the range f 60% t 80%. Thus, fr staging purpses, enlarged ndes must be evaluated by bipsy. The primary rle f these imaging examinatins is t identify the latin f enlarged ndes. This infrmatin allws apprpriate bipsy predures t be planned. In reent years, FDG-PET imaging has been shwn t be superir t CT and MRI in the assessment f mediastinal lymph nde metastases. This tehnique relies n physilgi (gluse metablism) rather than anatmi features t identify abnrmal lymph ndes. Thus, it has the ptential t identify neplasti invlvement within small ndes and t distinguish enlarged, hyperplasti ndes frm neplasti nes. Hwever, the number f false-psitive and false-negative studies (partiularly in small ndes with mirspi metastases) requires that lymph nde sampling still be perfrmed in mst ases. REFERENCES

Sharma A, Fidias P, Hayman LA, etal. Patterns f lymphadenpathy in thrai malignanies. Radiographics. 2004;24:419–434. Carter BW, Lihtenberger 3rd JP, Benveniste MK, de Grt PM, Wu CC, Erasmus JJ, Trung MT. Revisins t the TNM staging f lung aner: ratinale, signiane, and linial appliatin. Radiographics. 2018 Mar–Apr;38(2):374–391. Thrai Radilgy: The Requisites, 3rd ed, 19–60, 450–459.

CASE 85 Superior Vena Cava Syndrome from Lung Cancer 1. A. Lung aner an invade r metastasize t the mediastinum and mpress r lude the superir vena ava. Althugh there are numerus dilated vessels in the right hemithrax, they ntain undiluted ntrast, whih indiates that they are in ntinuity with the vein int whih the ntrast was administered and nt deriving w frm an arterivenus malfrmatin, whih shuld have bld mixed with ntrast. Extravasated ntrast llets in the sft tissues and des nt ll multiple vessels. 2. C. Lung aner is a mmn disease and ften results in diret mediastinal invasin r mediastinal adenpathy bth f whih an ause SVC bstrutin. Lng-term intravenus devies suh as paemaker leads an ause SVC stensis r thrmbsis. Hwever, these devies are nt the mst mmn auses f SVC syndrme. Fibrsing mediastinitis is assiated with SVC syndrme but is a relatively rare disease. Radiatin therapy an ause brsis in the mediastinum and result in SVC stensis, but its urrene is unusual. 3. D. Dural venus sinus thrmbsis is nt a typial manifestatin f SVC syndrme. Impaired venus return frm the head and nek an result in head and fae edema, extremity edema, and visual disturbanes. 4. D. Targeted radiatin therapy t the mediastinum an lead t rapid imprvement f the symptms f SVC syndrme. Chemtherapy may be useful in hemsensitive tumrs; hwever, it is nt the preferred rst-line treatment fr SVC syndrme related t nn–small ell lung arinma. Surgial resetin is generally nt indiated beause patients with

197

SVC syndrme related t nn–small ell lung arinma generally have unresetable disease. Surgial bypass may be used in sme ases fr palliatin where ther therapies have failed. SVC stenting may be useful in sme patients wh fail initial therapy. Hwever, data are smewhat limited.

Comment Differential Diagnosis Cntrast-enhaned CT image (Fig. 85.1) shws a sft tissue mass inltrating the mediastinum with bliteratin f the superir vena ava. Numerus hest wall and mediastinal llateral vessels are present lled with undiluted ntrast. The inltrating sft tissue mass is mst nsistent with lung arinma. Other neplasti auses wuld inlude lymphma and thymi arinma. Fibrsing mediastinitis an als ause SVC syndrme. This mst mmnly presents with a large, partially alied ndal mass in the mediastinum.

Discussion Superir vena ava (SVC) syndrme is aused by bstrutin f the SVC by external mpressin, intraluminal thrmbsis, neplasti inltratin, r a mbinatin f these presses. Mst ases result frm neplasm, mst mmnly lung arinma (espeially small ell arinma). Lymphma, thymi arinma, and metastati arinma are additinal malignant auses. There are a variety f benign auses, inluding lng-term intravenus devies (e.g., Hikman atheters and permanent paemakers) and brsing mediastinitis. Chest radigraphs ften shw a mass in the right paratraheal regin that may be ampanied by distentin f the azygs vein. In the setting f brsing mediastinitis, the right paratraheal mass is ften an enlarged alied nde. In patients wh develp thrmbsis f the SVC beause f an indwelling atheter, lateral displaement f the atheter may be apparent. Cmputed tmgraphy (CT) r magneti resnane imaging (MRI) an nrm the diagnsis f SVC bstrutin. On CT, the diagnsis is based n dereased r absent ntrast paiatin f the SVC in njuntin with paiatin f llateral vessels. Bth ndings are neessary t make a reliable diagnsis. Cntrast-enhaned CT with multiplanar refrmatin and 3D renstrutins is highly aurate at deteting the presene and level f SVC bstrutin. It is als valuable in determining the ause f bstrutin and fr delineating the llateral venus irulatin. The presene f llateral venus vessels shuld prmpt a searh fr a entral venus bstrutin. Treatment fr SVC syndrme related t lung aner is primarily with radiatin therapy. Chemtherapy may be used in sme patients espeially thse with small ell arinma. Surgial bypass is generally reserved fr patients with benign auses f SVC bstrutin r fr palliatin in thse wh fail radiatin and hemtherapy fr malignany. REFERENCES

Eran S, Karaman A, Okur A. The superir vena ava syndrme aused by malignant disease: imaging with multidetetr rw CT. Eur J Radiol. 2006;59:93–103. Carter BW, Lihtenberger 3rd JP, Benveniste MK, de Grt PM, Wu CC, Erasmus JJ, Trung MT. Revisins t the TNM staging f lung aner: ratinale, signiane, and linial appliatin. Radiographics. 2018 Mar–Apr;38(2):374–391. Thrai Radilgy: The Requisites, 3rd ed, 450–459.

198

SECTION IV

Answers

CASE 86 Diffuse Alveolar Hemorrhage From Vasculitis 1. A and C. The differential diagnsis fr widespread nslidatin in the autely ill patient inludes pulmnary hemrrhage, pulmnary edema, diffuse infetius pneumnia, massive aspiratin, and aute lung injury. Langerhans ell histiytsis is a smking related disease haraterized by upper lung predminant ndules and ysts. Silisis is haraterized by well-dened ndules with an upper lbe predminane with r withut prgressive massive brsis. Aute silisis hwever an prdue diffuse lung disease whih is mstly grund glass in appearane. The pathlgy is that f alvelar prteinsis. 2. C. Pulmnary vasulitis is the mst mmn ause f diffuse alvelar hemrrhage. Hemphilia is rare. Drug indued pulmnary hemrrhage is rare and an ur with therapeuti drugs suh as erltinib r illiit drugs suh as rak aine. Infetin rarely auses diffuse alvelar hemrrhage. 3. B. The mst mmn lung manifestatin f mirspi plyangiitis is diffuse alvelar hemrrhage. Diffuse alvelar hemrrhage related t granulmatsis with plyangiitis (Wegener’s granulmatsis) nly urs in 10% t 15% f patients. Diffuse alvelar hemrrhage frm esinphili granulmatsis with plyangiitis (frmerly Churg-Strauss syndrme) is rare. Takayasu arteritis primarily affets the arta and its branhes. Large pulmnary artery invlvement des ur, but diffuse alvelar hemrrhage is nt a feature. 4. A. The diagnsis f diffuse alvelar hemrrhage is nrmed by brnhalvelar lavage, whih shws prgressive inrease in return f bld in lavage uid. Transbrnhial and surgial bipsy an shw hemsiderin-laden marphages r areas f hemrrhage but are nt the best hies fr diagnsis. Sputum analysis has lw yield and may shw red bld ells but des nt nrm diffuse alvelar hemrrhage.

Comment Differential Diagnosis The prtable anterpsterir (AP) radigraph (Fig. 86.1) shws diffuse lung nslidatin. Cmputed tmgraphy (CT) shws dense bilateral nslidatin psterirly and pathier grundglass paity anterirly (Fig. 86.2). The differential diagnsis fr diffuse lung paity in the autely ill patient is brad and inludes diffuse pneumnia frm infetin, ardigeni and nn-ardigeni edema, hemrrhage, massive aspiratin, r aute lung injury. Hemptysis, when present, and dereased hematrit favr diffuse alvelar hemrrhage.

Discussion Diffuse alvelar hemrrhage (DAH) is haraterized by bleeding int the alvelar spaes and results frm disruptin f the alvelar-apillary basement membrane. The linial presentatin is aute, and patients usually present with ugh, fever, dyspnea, and hemptysis. Hwever, apprximately ne-third f patients with DAH d nt have hemptysis. Althugh a wide variety f insults an lead t DAH, the histpathlgi injury an be gruped int apillaritis, bland hemrrhage, and diffuse alvelar damage (DAD). Capillaritis an result frm systemi vasulitis (espeially anti-neutrphil ytplasmi antibdy assiated), antiglmerular basement membrane antibdies, llagen vasular disease, r be idipathi. Causes f DAD are numerus and inlude infetin, systemi sepsis, reatin t drugs, trauma, and

llagen vasular disease. Bland DAH may develp sendary t left heart failure, antiagulatin, and hemrrhagi disrders. Radigraphi ndings f DAH vary by the extent and severity f DAH. Radigraphs f patients with mild DAH may be nrmal r near nrmal, while thse f patients with severe disease may shw extensive nslidatin. CT ndings are similar, ranging frm mild grund-glass paity t extensive multilbar nslidatin. The distributin f paities tends t be mre entral and perihilar, but the ndings are ften indistinguishable frm ther auses f diffuse lung paity suh as edema, infetin, and alvelar damage. Brnhalvelar lavage is the referene standard fr establishing a diagnsis f DAH. Inreased bld prduts in returned lavage uid is diagnsti f DAH. Hemsiderin-laden marphages an als be identied in lavage uid. Treatment inludes management f underlying auses (when knwn) and supprtive therapy as needed. REFERENCES

Krause ML, et al. Update n diffuse alvelar hemrrhage and pulmnary vasulitis. Immunol Allergy Clini North Am. 2012 Nv;32(4):587–600. Thrai Radilgy: The Requisites, 3rd ed, 238–258, 335–354.

CASE 87 Usual Interstitial Pneumonia (UIP) Caused by Idiopathic Pulmonary Fibrosis (IPF) 1. B. Subpleural and basal predminant retiulatin with hneymbing is highly diagnsti f usual interstitial pneumnia. Desquamative interstitial pneumnia is haraterized by extensive grund-glass paity, smetimes with ampanying mild ne retiulatin and asinal small ysts. Nnspei interstitial pneumnia is usually haraterized by grund-glass paity with r withut retiulatin in basal and peribrnhial distributin. Lymphid interstitial pneumnia an be quite variable but is usually haraterized by grund-glass paity, ndules, and ysts. 2. D. Fr patients with a denite UIP pattern f brsis n CT, bipsy is rarely warranted. Surgial bipsy is nt withut risk and rarely will hange the diagnsis with this pattern. Transbrnhial and CT-guided needle bipsy are nt sufient t make a pathlgi diagnsis f UIP, primarily beause f the small sample size and relative peripheral latin f brsis. A multidisiplinary apprah t diagnsis is imprtant with invlvement f the radilgist, pulmnlgist, and pathlgist. 3. A. A usual interstitial pneumnia (UIP) pattern f brsis is mst strngly assiated with rheumatid arthritis. Nnspei interstitial pneumnia (NSIP) is the mst mmn pattern f diffuse lung disease in patients with prgressive systemi slersis. NSIP and rganizing pneumnia are the mst mmn patterns f diffuse lung disease in patients with plymysitis. Lymphid interstitial pneumnia (LIP) is assiated with Sjögren syndrme. 4. D. Asbests an result in pulmnary brsis (asbestsis), typially with a usual interstitial pneumnia (UIP) pattern f brsis. Cal and silia ause pneumnisis haraterized by small perilymphati ndules predminantly in the upper lungs. Ndules an alese t frm large paities, termed prgressive massive brsis. Beryllium auses a granulmatus respnse in the lungs indistinguishable frm saridsis.

Fair Game

Comment Differential Diagnosis Axial (Figs. 87.1 and 87.2) and rnal refrmatted (Fig. 87.3) CT images shw subpleural and basal predminant retiulatin with hneymbing and tratin brnhietasis, nsistent with UIP pattern f pulmnary brsis. This pattern n CT is highly spei fr UIP and mst mmnly represents IPF.

Discussion UIP is a pattern f hrni diffuse lung injury haraterized by restritive physilgy and redued diffusing apaity. Mst patients with UIP have n identiable ause, termed IPF. Other auses f UIP histlgy and CT pattern inlude llagen vasular disease (partiularly rheumatid arthritis), familial brsis, drug txiity, and asbests expsure (asbestsis). Older age and male sex are risk fatrs fr IPF. Patients present with prgressive dyspnea, dry ugh, restritive pulmnary physilgy, and redued diffusin apaity. Digital lubbing may be present, and arse end expiratry rakles are heard n expiratin, partiularly in the lwer lung znes. The diagnsis f UIP is usually made by hest CT when the harateristi pattern f subpleural and basal predminant retiulatin with hneymbing is present. In mst ases, this CT pattern and linial evaluatin alne are enugh t establish a diagnsis f IPF. Oasinally, a surgial lung bipsy may be required either beause f nfunding ndings n hest CT r linial evaluatin. Histpathlgially, UIP is haraterized by spatially and temprally hetergeneus areas f brsis and lbular llapse with harateristi brblasti fi at the leading edge f brsis. Mirspi hneymbing may be present, and inammatin is minimal r absent. UIP, regardless f ause, is assiated with a pr prgnsis. Many patients with IPF die f their disease within 3 t 4 years f diagnsis. Cmpliatins inlude prgressive respiratry deline, aute exaerbatins, and lung aner. Treatment ptins fr IPF are limited t tw nvel drugs, whih have been shwn t imprve prgressin free survival but nt verall survival REFERENCES

Ryu JH, Mua T, Daniels CE, et al. Idipathi pulmnary brsis: evlving nepts. Mayo Clin Proc. 2014 Aug;89(8):1130–1142. Raghu G, Remy-Jardin M, Myers JL, et al. Amerian Thrai Siety, Eurpean Respiratry Siety, Japanese Respiratry Siety, and Latin Amerian Thrai Siety. Diagnsis f idipathi pulmnary brsis: an fial ATS/ERS/JRS/ALAT linial pratie guideline. Am J Respir Crit Care Med. 2018 Sep 1;198(5):e44–e68. Thrai Radilgy: The Requisites, 3rd ed, 355–376.

CASE 88 Intramuscular Hematoma From Supratherapeutic Warfarin Therapy 1. A and C. Fi f high attenuatin in an intramusular mass are typial in aute hematmas. An intramusular metastasis suh as renal ell arinma an spntaneusly hemrrhage, as well. Sft tissue absesses will have a lw attenuatin entral prtin. High attenuatin fi are nt typial. Elastbrmas ur inferir t the sapular tip and usually feature sft tissue and fat attenuatin. 2. B. High attenuatin, nnalied fi within a hetergeneus lletin are typial f bld prduts. Sme tumrs ntain high attenuatin fi, but mst are equal r slightly lwer attenuatin than skeletal musle. The fi f high attenuatin

199

are nly mderately greater than thse f skeletal musle, making alium very unlikely. Fat is f lw attenuatin. 3. A. Intramusular hematma is nt a risk fatr fr malignant degeneratin. Intramusular hematma an impede bld w t the affeted musle and result in ishemi mypathy. Neurpathy an result frm an intramusular hematma hrnially mpressing peripheral nerves. Intramusular hematma an ause pressure nersis f adjaent bne, espeially when the hematma is quite large. 4. D. Spntaneus intramusular hematma an be a mpliatin f hemphilia. Aspirin prlngs bleeding time but des nt predispse t spntaneus intramusular hematma. Extravasatin f IV ntrast medium an ur during pwer injetin but des nt ause spntaneus intramusular hematma. Vitamin K supplementatin wuld interfere with warfarin therapy, reduing its antiagulatin effet.

Comment Differential Diagnosis Anterpsterir (AP) hest radigraph demnstrates hazy paiatin f the left hemithrax with enlargement f the sft tissues in the left lateral hest wall (Fig. 88.1). The CT image shws a large, well-marginated, hetergeneus left psterlateral hest wall mass entered within the musulature (Fig. 88.2). The high-attenuatin fi in this mass n unenhaned CT represent bld prduts and are nsistent with aute intramusular hematma, a mpliatin f supratherapeuti warfarin therapy. One uld nsider a hemrrhagi metastasis, suh as renal ell arinma.

Discussion Crss-setinal imaging studies an be helpful in the identiatin f the site and extent f intramusular hematmas in patients with either traumati injury r spntaneus hemrrhage. Beause f its relative lw st and lak f inizing radiatin, ultrasngraphy is usually the preferred test fr this purpse. In patients with large hematmas, CT and magneti resnane imaging (MRI) are asinally helpful in determining the extent and the age f hemrrhage and identifying the effet f the hemrrhage n adjaent rgans. Therapy is usually supprtive. Hwever, large hematmas may require surgial interventin if ishemi injury r neurpathy is suspeted. Mysitis ssians an ur in up t 10% f patients with intramusular hematma and an ause pain and dereased funtin. REFERENCES

MKenzie G, Raby N, Rithie D. Pitrial review: Nn-neplasti sft tissue masses. Br J Radiol. 2009 Aug;82(981):775–785. Thrai Radilgy: The Requisites, 3rd ed, 159–192.

CASE 89 Scimitar Syndrome 1. A and C. The large urvilinear paity in the right lwer lung is mst likely a vasular struture, likely an anmalus pulmnary vein. A vein draining a pulmnary arterivenus malfrmatin is likely withut identiatin f a similar size artery feeding the malfrmatin. Cngenital pulmnary airway malfrmatin is typially nt assiated with a large vasular anmaly, and the muele f brnhial atresia usually has a smaller diameter. 2. B. Beause the pulmnary vein drains int the right atrium, a left-t-right shunt is mst likely present. A left-t-left shunt

200

SECTION IV

Answers

urs when an anmalus pulmnary vein drains t the left atrium. A right-t-right shunt wuld ur when a pulmnary artery drained int anther artery r the right atrium. A rightt-left shunt urs with a pulmnary arterivenus malfrmatin, where bld bypasses the nrmal pulmnary apillary bed. 3. B. Simitar syndrme is assiated with a hst f ngenital abnrmalities, and hypplasti lung with abnrmal airway branhing is the mst mmn. There is n apparent inreased inidene f lung aner, kidney abnrmalities, r musular dystrphy. 4. D. Simitar syndrme in an asymptomatic adult is generally an inidental nding that requires n management. Resetin, endvasular iling, r brnhspy are nt indiated fr this patient.

Comment Differential Diagnosis Psteranterir (PA) hest radigraph (Fig. 89.1) shws a large, urvilinear struture ursing frm the right infrahilar regin t the medial right lung base, reeting the simitar vein, named beause f its likeness t a Turkish swrd. The right lung is slightly hypplasti, and the right main brnhus is smaller than usual. Crnal maximum intensity prjetin (MIP) frm magneti resnane (MR) angigram f the hest (Fig. 89.2) shws the urvilinear struture draining int the suprahepati inferir vena ava. Differential nsideratins wuld inlude pulmnary vein varix r meandering pulmnary vein. Differential auses fr the slightly hypplasti right hemithrax uld inlude pulmnary underdevelpment, interruptin f the right pulmnary artery, pulmnary vein stensis, and Swyer-JamesMaLed syndrme.

Discussion Simitar syndrme (venlbar syndrme) is a frm f partial anmalus pulmnary venus drainage, where the simitar vein drains all r part f the ipsilateral lung int the systemi venus irulatin, typially the inferir vena ava. In tw-thirds f ases, the simitar vein drains the entire lung and in ne-third f ases, nly the inferir half f the lung drains thrugh the simitar vein. In additin t anmalus venus drainage, ther anmalies are present t varying extents. Hypplasia f the ipsilateral lung urs in nearly all ases with varying degrees f abnrmal lbatin, airway branhing, and dextrpsitin f the heart. The ipsilateral pulmnary artery is usually smaller than nrmal, and systemi arterial supply t a prtin f the affeted lung urs in up t tw-thirds f patients. Other defets inlude seundum type atrial septal defet, diaphragmati hernia, and hrseshe lung. The pathgenesis f simitar syndrme is unlear, and the develpmental abnrmalities that lead t the anmalus anatmy are nt well desribed. Simitar syndrme has tw distint linial manifestatins. The infantile frm has a high assiatin with signiant ngenital ardivasular anmalies inluding ventriular septal defet, arti artatin and hypplasia, tetralgy f Fallt, trunus arterisus, and abnrmal rigin f the left rnary artery. Beause f the assiated malfrmatins, mrtality and mrbidity rates are high. In ntrast t the infantile frm, the adult frm f simitar syndrme is ften asymptmati r presents nly with milder symptms frm shunting r ther anmalies. Asymptmati patients typially d nt require treatment. Cmputed tmgraphy (CT) r MR angigraphy an better delineate the anatmy f simitar syndrme. MR has the

advantage f being able t quantify shunts whereas CT an better delineate lung abnrmalities. REFERENCES

Gudjnssn U, Brwn JW. Simitar syndrme. Semin Thorac Cardiovasc Surg Pediatr Card Surg Annu. 2006:56–62. Thrai Radilgy: The Requisites, 3rd ed, 193–209.

CASE 90 Pericardial Effusion 1. A and B. Myardial infartin with left ventriular failure is the mst mmn ause f periardial effusin. Sersitis related t systemi lupus erythematsus and ther llagen vasular diseases an ause pleural and periardial effusin. Periardial effusin is nt a typial nding f relapsing plyhndritis, whih primarily affets the trahea and brnhi. Aute pulmnary emblism an ause pleural effusin, but periardial effusin des nt ur. 2. D. The lateral view shws a lw-density vertial line behind the sternum (mediastinal fat) and anther lw-density vertial line psterir t it (epiardial fat). These tw lines are separated by uid density whih represents the periardial effusin. The appearane resembles an Ore kie. 3. C. Cardia MRI is extremely sensitive fr deteting periardial effusins. The entire periardium an be imaged in multiple planes. Chest radigraphs have very lw sensitivity fr periardial effusin, espeially with vlumes less than 200 mL. Transthrai and transesphageal ehardigraphy are highly sensitive fr periardial effusin but an miss effusins lulated psterirly beause f limited austi windws. 4. D. Dressler syndrme refers t pleural and periardial effusins, whih develp 2 t 10 weeks after aute myardial infartin. Dressler syndrme is nt related t rheumatid arthritis, viral myarditis, and radiatin indued injury.

Comment Differential Diagnosis The psteranterir (PA) radigraph shws and enlarged ardia silhuette (Fig. 90.1). The lateral radigraph shws a vertially riented sft tissue stripe anterir t the heart brdered by tw stripes f fat attenuatin (Fig. 90.2), representing a periardial effusin. Unenhaned mputed tmgraphy (CT) image (Fig. 90.3) shws the full extent f the periardial effusin and illustrates the anterir mediastinal and epiardial fat, whih enable the effusin t be visible n the lateral radigraph. Pleural effusins are als present. The differential diagnsis fr periardial effusin is brad and inludes infetin, trauma, radiatin therapy, drug txiity, llagen vasular diseases, metabli disrders, and neplasms.

Discussion The lateral radigraph shws a urvilinear retrsternal stripe braketed by fat attenuatin, als referred t as the dublelueny, sandwih r Ore kie sign. This sign refers t widening (>4 mm) f the sft tissue paity f the periardium between the luent stripes that represent fat lated anterir (mediastinal) and psterir (epiardial) t the periardium. This sign has a relatively lw sensitivity but a high speiity fr deteting periardial effusin. Chest radigraphy is assiated with a relatively pr sensitivity fr deteting periardial effusins. It has been estimated

Fair Game

that 200 mL f periardial uid must be present t reliably make the diagnsis radigraphially. In ntrast, CT and ehardigraphy are highly sensitive fr deteting periardial effusin and is the studies f hie fr sreening patients with suspeted periardial effusin. MRI may be helpful fr haraterizing mplex periardial uid lletins. Furthermre, MRI prvides detailed imaging f the entire periardium in ntrast t ehardigraphy, whih an be limited by austi windws. The mst mmn ause f periardial effusin is myardial infartin with left ventriular failure. Dressler syndrme refers t the develpment f periardial and pleural effusins 2 t 10 weeks fllwing aute myardial infartin. Suh effusins an be hemrrhagi, partiularly in patients wh have reeived antiagulatin therapy. Large periardial effusins an require drainage t prevent r relieve ardia tampnade. Sme effusins will reslve withut treatment. Periardial thikening an result in nstritive periarditis. REFERENCES

Kligerman S. Imaging f Periardial Disease. Radiol Clin North Am. 2019 Jan;57(1):179–199. Thrai Radilgy: The Requisites, 3rd ed, 226–237.

CASE 91 Idiopathic Bronchiectasis 1. A and B. Ciliary dyskinesia predispses patients t reurrent sinpulmnary infetins, and brnhietasis an result frm reurrent airway inammatin. Allergi brnhpulmnary aspergillsis urs primarily in patients with asthma and ysti brsis and leads t brnhietasis beause f hrni brnhial inammatin. Tratin brnhietasis and brnhiletasis an develp in the setting f interstitial brsis. Hwever, this is a sendary phenmenn and nt related t an intrinsi brnhial abnrmality. In additin, the dilated peripheral airways will be within areas f brsis. These etati airways may appear varise in mrphlgy. Lymphangileimymatsis is haraterized by the develpment f lung ysts with smth, thin walls. Brnhietasis is nt a mpnent f the disease. Cigarette smking leads t hrni airway inammatin and brnhial wall thikening but generally des nt result in brnhietasis. 2. D. Infetin is the mst mmn ause f brnhietasis, espeially infetins during hildhd. Cngenital brnhietasis is unmmn and is ften the result f deieny in brnhial artilage (e.g., Williams-Campbell syndrme). Interstitial pneumnia is unmmn and an be assiated with tratin brnhietasis in areas f brsis. Lymphangileimymatsis is haraterized by the develpment f lung ysts with smth, thin walls. Brnhietasis is nt a mpnent f the disease. 3. C. Brnhial wall thikening is a nnspei nding n mputed tmgraphy (CT) and an be enuntered in presses suh as brnhitis, lymphangiti arinmatsis, and lung edema. Brnhial diameter greater than that f the adjaent artery, lak f nrmal brnhial tapering, and brnhi visible in peripheral 1 m f lung are nsidered diret signs f brnhietasis. 4. D. Pleural effusin is nt a diret nsequene f brnhietasis. Hwever, a parapneumni effusin an develp frm brnhietasis-related pneumnia. Brnhietasis is ne f the mst mmn auses f hemptysis. The hrnially inamed brnhi are ften friable and prne t hemrrhage. Ateletasis frm brnhietasis an result frm muid impatin, pr learane f seretins, and brnhmalaia. Chrni brnhietasis limited t a lbe may result in

201

mplete ateletasis f that lbe withut brnhial bstrutin. Dilated air brnhgrams are a prminent feature. Pr muiliary learane in dilated and inamed brnhi predispses affeted patients t reurrent pneumnia.

Comment Differential Diagnosis Axial (Fig. 91.1) and rnal (Fig. 91.2) CT images shw mild brnhietasis haraterized by ylindrial dilatin f peripheral airways, brnhial wall thikening, and muus plugging. Als nte the presene f tree-in-bud paities and msai attenuatin, reeting assiated small airways disease. The auses f brnhietasis are myriad and inlude infetin (inluding atypial mybaterial), immune deieny, iliary dyskinesia, ngenital artilage deieny in the airways, hrni aspiratin, allergi brnhpulmnary aspergillsis, ysti brsis, small airways disease, and thers.

Discussion Brnhietasis is dened as abnrmal, irreversible dilatin f the brnhi. Brnhietasis an arise sendary t a wide variety f ngenital and aquired abnrmalities. Cysti brsis is the mst mmn assiated ngenital abnrmality, and prir infetin, espeially hildhd infetins, is the mst mmn aquired abnrmality. Chest radigraphs are ften nrmal in patients with mild degrees f brnhietasis but asinally reveal parallel, thikened brnhial walls, als referred t as a tram-trak appearane. With ysti brnhietasis, radigraphs an reveal lusters f air-lled ysts, ften with uid levels. CT is highly sensitive and spei fr diagnsing brnhietasis. Findings inlude a brnhial wall diameter greater than its adjaent artery (resulting in a signet-ring sign when the dilated brnhus and ampanying artery are viewed in rss setin), identiatin f brnhi within the peripheral entimeter f the lung, lak f nrmal brnhial tapering, brnhial wall thikening, and strings r lusters f ysts. Beause brnhial wall thikening may als be seen in ther frms f airway disease, it shuld nt be used as a sle riterin fr diagnsing brnhietasis. Cmpliatins f brnhietasis inlude reurrent infetins, hemptysis, muus impatin, and ateletasis. Patients are usually treated nservatively with antibitis as needed. Severe hemptysis an require brnhial artery emblizatin r resetin f affeted lung. REFERENCES

Bnavita J, Naidih DP. Imaging f brnhietasis. Clin Chest Med. 2012;Jun;33(2):233–248. Millirn B, Henry TS, Veeraraghavan S, Little BP. Brnhietasis: mehanisms and imaging lues f assiated mmn and unmmn diseases. Radiographics. 2015 Jul–Aug;35(4):1011–1030. Thrai Radilgy: The Requisites, 3rd ed., pp. 137–158.

CASE 92 Achalasia 1. B and D. Ahalasia results in esphageal bstrutin and dysmtility. The esphagus prximal t the hypertni segment an be lled with air, liquid, r fd debris. Esphageal arinma an ause esphageal bstrutin, leading t prximal distensin and retained fd, liquid, and gas. Hwever, the degree f esphageal dilatin illustrated in this ase is rare in esphageal arinma and usually indiates a mre hrni nditin.

202

SECTION IV

Answers

Tumr ells an als invade the nerves f the lwer esphageal sphinter, preventing nrmal relaxatin during swallwing (pseudahalasia). Mst esphageal dupliatin ysts are uid lled and typially are rund, masslike strutures. Resultant high-grade bstrutin wuld be highly unusual. An esphageal divertiulum is a fal utpuhing r rund paraesphageal mass and des nt ause diffuse dilatin f the esphagus. 2. A. The tubular nature f the gas-lled struture is typial f the esphagus. Althugh an enlarged azygs vein displaes the azygesphageal ntur, it des nt ntain gas. The gaslled struture extends abve the expeted latin f the azygs arh is at the level f the trahebrnhial angle. The right atrium is nt enlarged. Its nturs are anterir t the azygesphageal reess. The left atrium is nrmal in this patient. 3. C. An esphagram an prvide infrmatin regarding esphageal mtility, unlike the ther tests whih are listed. A CT an shw esphageal dilatin, thikening, and narrwing, but it annt evaluate esphageal mtility. MRI has a very limited rle in the evaluatin f esphageal disrders. The lateral radigraph an prvide further supprt that the abnrmal struture is the esphagus, but it will nt prvide any funtinal infrmatin. 4. A. Prgressive systemi slersis (slerderma) typially results in dilatin f the esphagus and dysmtility with a patulus lwer esphageal sphinter. Ahalasia results frm failure f the lwer esphageal sphinter t relax. Chagas disease is aused by Trypanosoma cruzi, whih an invade and destry the nerve plexus respnsible fr relaxatin f the lwer esphageal sphinter. It urs primarily in Suth Ameria. Tumr invasin f the nerve plexus respnsible fr relaxatin f the lwer esphageal sphinter (pseudahalasia) an mimi ahalasia.

Differential Diagnosis The psteranterir (PA) hest radigraph (Fig. 92.1) shws a dilated, tubular struture ntaining a uid level ursing thrugh the mediastinum and displaing the nrmal azygesphageal ntur, representing the esphagus. Ahalasia is the mst likely ause f severe esphageal dysmtility and dilatin. Other auses inlude Chagas disease, pseudahalasia (neplasti invasin f the nerves f the lwer esphageal sphinter), and prgressive systemi slersis (slerderma).

Discussion The azygesphageal interfae is prdued by the juxtapsitin f aerated lung in the right lwer lbe and the sft tissue paity f the right lateral margin f the azygs vein, esphagus, r bth. On nrmal hest radigraphs, the azygesphageal interfae begins at the level f the azygs arh and extends inferirly t the level f the diaphragm. It nrmally prdues a nave slpe as it urves slightly tward the left. Abnrmalities f either the azygs vein (e.g., azygs ntinuatin f the inferir vena ava) r esphagus (e.g., ahalasia) an result in rightward displaement f this interfae. Subarinal masses suh as brnhgeni ysts and lymph nde enlargement an als result in fal rightward displaement f the azygesphageal interfae, usually prduing a rightward nvexity in the subarinal regin. The hest radigraph (Fig. 92.1) shws diffuse, marked rightward displaement f the azygesphageal ntur. When an air-lled, distended esphagus displaes the azygesphageal ntur, a stripe is frmed rather than an interfae. This patient has a histry f ahalasia. In mst patients with this disrder, the esphagus ntains a large amunt f retained seretins. Therefre, the radigraph typially shws a displaed azygesphageal interfae rather than a stripe. Retained seretins

and fd an als result in a disrete uid level within the distended esphagus. Treatment fr ahalasia inludes a Heller mytmy t release the lwer esphageal sphinter. Sme patients require balln dilatin. Sendary gastresphageal reux an develp after mytmy. REFERENCES

Whitten CR, Khan S, Munneke GJ, Grubni S. A diagnsti apprah t mediastinal abnrmalities. Radiographics. 2007;27:657–671. Thrai Radilgy: The Requisites, 3rd ed., pp. 19–60, 97–136.

CASE 93 Cavity Caused by Coccidioidomycosis 1. A and C. Granulmatsis with plyangiitis (Wegener’s) an ause lung ndules and avities. Cavities ften have thik irregular walls; hwever, with learing, the walls an beme thin. A slitary avitary lesin is a mmn manifestatin f hrni idiidmysis. Cavities an frm in advaned saridsis but usually d s in the setting f tratin brnhietasis and brsis. There are n ndings n the images t suggest saridsis. Septi infarts are usually multiple, peripheral and predminate in the lwer lbes. They an avitate and usually have thik walls and the brad surfae usually abuts the pleura. Mst are smaller than the avity in this patient. 2. D. Coccidioides is endemi t the Amerian Suthwest in additin t Mexi, Central Ameria, and Suth Ameria. 3. B. Lung avities are typial f the hrni frm f idiidmysis and nt the aute pneumni frm. Mst infeted patients are asymptmati, and disseminated disease is rare. Mst patients with the pneumni frm have spntaneus reslutin f their infetin. 4. B. Mst patients with disseminated idiidmysis have multiple tiny ndules, typially 5 t 10 mm in diameter. Multifal lung nslidatin, diffuse grund-glass paity, and septi pulmnary infarts are nt typial features f disseminated idiidmysis.

Comment Differential Diagnosis The psteranterir (PA) radigraph (Fig. 93.1) shws an vid mass just lateral t the left hilum. Cavitatin is mre nspiuus n mputed tmgraphy (CT) (Fig. 93.2). Nte the thik wall f the avity but the rather smth interir. Sme satellite ndules are als present. Malignant neplasm and infetin shuld be strngly nsidered. Granulmatsis with plyangiitis an als present as a avitary lesin but mre ften lesins are multiple.

Discussion Cidiidmysis infetin is aused by inhalatin f spres f Coccidioides immitis r Coccidioides posadasii, sil inhabitants endemi t desert areas. Althugh mst patients are asymptmati fllwing expsure, sme experiene a mild, ulike illness. Radigraphi ndings vary depending n the stage f infetin. Fllwing initial inhalatin f the spres, there is a lal pneumni respnse, whih is haraterized radigraphially as an area f nslidatin. Suh nslidatin usually invlves less than an entire lbe, is ften lated in the lwer lbes, and usually reslves spntaneusly withut therapy. Chrni pulmnary idiidmysis is haraterized radigraphially by slitary r multiple pulmnary ndules and avities.

Fair Game

Suh avities, as shwn in this ase, an have variable wall thikness and are usually radilgially indistinguishable frm ther auses f avitary lesins. In a minrity (10% t 15%) f patients, idiidmysis is assiated with harateristi thin walled (grape skin) avities. Suh avities an rapidly hange in size, presumably due t a hek-valve mmuniatin with the brnhial tree. Disseminated idiidmysis is rare; it appears radigraphially as multiple ndules. The ndules usually range in size frm 5 mm t 1 m in diameter, but smaller miliary ndules are bserved in sme ases. The urse f disseminated idiidmysis is variable: it may be hrni and insidius r rapidly fatal. The infetin is usually fatal in patients wh are immunmprmised. Like ther infetins, idiidmysis an ause a false-psitive result n urdexygluse–psitrn emissin tmgraphy (FDG-PET) studies. This an ur in the aute r hrni phase f infetin. Mst patients require n treatment. Antifungal therapy may be used in patients whse disease fails t lear spntaneusly. REFERENCES

Lindell RM, Hartman TE:. Fungal infetins. In: Müller NL, Silva CI, eds. Imaging of the Chest. Philadelphia: Saunders; 2008:362–365. Kunin JR, Flrs L, Hamid A, Fuss C, Sauer D, Walker CM. Thrai endemi fungi in the United States: imprtane f patient latin. Radiographics. 2021 Mar–Apr;41(2):380–398. Thrai Radilgy: The Requisites, 3rd ed., pp. 289–309.

CASE 94 Hydrostatic Pulmonary Edema 1. A and C. Hydrstati pulmnary edema (edema sendary t ngestive heart failure r vlume verlad) is the mst mmn ause f interlbular septal thikening. Smth interlbular septal thikening is nt a typial feature f usual interstitial pneumnia, whih is haraterized by irregular lines (retiulatin), reeting interstitial brsis. Smth interlbular septal thikening is nt a typial nding f saridsis. Hwever, small ndules may be present alng the interlbular septa in patients with saridsis. 2. A. Kerley B lines n the hest radigraph rrespnd t thikened interlbular septa n CT. Other hest radigraphi ndings inlude thikening f the interlbar ssures that is due t edema in r thikening f the subpleural interstitium n bth sides f the ssure. Brnhvasular bundle thikening n the hest radigraph rrespnds t edema in r thikening f the peribrnhvasular interstitium. Fluffy lung nslidatin des nt rrespnd t interlbular septal thikening n CT. 3. A. HRCT requires the use f thin llimatin (≤1.5 mm). Prne imaging may be used in sme patients as part f an interstitial lung disease prtl. Multiplanar refrmatins an be generated frm vlumetri CT imaging. Hwever, they are nt a required mpnent f HRCT. A high spatial frequeny renstrutin kernel prvides edge sharpening, better delineating the small brnhpulmnary strutures n HRCT. 4. B. Repeat hest imaging, either with a radigraph r CT, fllwing diuresis an easily nrm whether septal thikening is the result f hydrstati edema. FDG-PET might shw uptake thrughut the lung if the septal thikening reets diffuse lymphangiti arinmatsis. Hwever, the results are nnspei. Transbrnhial bipsy and surgial bipsies are f urse invasive. They may be a gd ptin if septal thikening persists despite diuresis, and the linial presentatin suggests hrni interstitial lung disease.

203

Comment Differential Diagnosis The primary differential diagnsis fr smth interlbular septal thikening is pulmnary edema and lymphangiti arinmatsis. Edema tends t be symmetri and gravitatinally dependent, whereas lymphangiti arinmatsis mre ften is randmly distributed. Pleural effusins an be present with either nditin. Other auses f septal thikening inlude pulmnary venus bstrutin, strage diseases and amylidsis, as well as lymphati and lymphprliferative disrders. Septal thikening an als be a sendary nding in patients with infetin and pulmnary hemrrhage.

Discussion Althugh the diagnsis f ngestive heart failure is usually made n the basis f typial linial and radigraphi ndings, asinally patients with unsuspeted ngestive heart failure are imaged with HRCT f the hest in searh f a ause f dyspnea. Hydrstati edema is als an asinal inidental nding in patients wh are being sanned fr ther purpses. Thus, it is imprtant t be aware f the typial HRCT features f hydrstati edema. On CT f patients with hydrstati edema (Fig. 94.1), a mbinatin f grund-glass paity, smthly thikened septal lines, peribrnhvasular interstitial thikening, inreased vasular aliber, and thikened ssures may be present. Small pleural effusins, ften n the right, are als ften present. Signs f brsis suh as hneymbing, tratin brnhietasis, and arhitetural distrtin are absent. Interestingly, patients with aute ngestive heart failure have als been reprted t have asinally enlarged mediastinal lymph ndes whih are edematus and haziness f the mediastinal fat als aused by edema. Crrelatin between imaging ndings and linial data is usually sufient t nrm the diagnsis. When the diagnsis is in dubt linially, a fllw-up study after diuresis is asinally helpful t nrm reslutin f abnrmalities and t exlude hrni interstitial lung disease. REFERENCES

Strt ML, Kee ST, Glden JA, Webb WR. Hydrstati pulmnary edema: high-reslutin CT ndings. AJR Am J Roentgenol. 1995;165:817–820. Thrai Radilgy: The Requisites, 3rd ed., pp. 226–237.

CASE 95 Internal Mammary Lymph Node Enlargement 1. B and C. Lymphma and breast arinma are mmn auses f internal mammary lymphadenpathy. Arti artatin an ause enlargement f the internal mammary (thrai) vessels, resulting in a lbulated retrsternal paity. Hwever, the arta is nrmal in this patient, and arti artatin wuld nt aunt fr anterir mediastinal abnrmality. Althugh lymphadenpathy is a mmn manifestatin f tuberulsis, invlvement f the internal mammary lymph ndes is unmmn in the absene f disseminated disease. 2. B. Ipsilateral axillary lymph ndes are the mst mmn site f lymph nde metastases in patients with breast arinma. Internal mammary lymph ndes are a mmn site f lymph nde metastases in breast aner, but they are nt the mst mmn. Hilar and mediastinal lymph ndes are less-mmn sites f lymph nde metastases frm breast arinma. 3. B. Internal mammary lymph ndes are apparent n the lateral hest radigraph nly when they are quite large.

204

SECTION IV

Answers

4. C. Primary mediastinal lymphma, whih is nned t the anterir mediastinum, is mst mmnly diffuse large B ell lymphma (als referred t as Primary mediastinal B cell lymphoma). This is a relatively reently desribed tumr whih an lsely resemble Hdgkin disease. Classial Hdgkin lymphma an ur in the anterir mediastinum but is less mmn. Flliular lymphma less mmnly is islated t the anterir mediastinum. Small lymphyti lymphma is usually systemi affeting the spleen and multiple lymph nde statins thrughut the bdy.

Comment

is mmn, but hemptysis nly urs in up t 10% f patients. Air emblism and bipsy trak seeding with tumr ells are extremely rare mpliatins. 4. C. Use f a utting needle fr re needle bipsy imprves the diagnsti yield f transthrai needle bipsy (TTNB) fr a spei benign diagnsis. Inreasing vlume f aspirate will nt likely imprve the diagnsti yield f TTNB fr a benign lesin. On-site ytpathlgy an be useful fr imprving the diagnsti yield f ne needle aspiratin fr malignant lesins. Hwever, establishing a benign diagnsis based n ne-needle aspiratin alne an be very difult. Use f CT urspy des nt imprve the diagnsti yield f TTNB fr benign lesins.

Differential Diagnosis

Comment

The lateral radigraph (Fig. 95.1) shws a lbulated paity lling the retrsternal lear spae. Cntrast-enhaned mputed tmgraphy (CT) image (Fig. 95.2) shws bilateral internal mammary lymphadenpathy and a sft tissue mass in the anterir mediastinum. Althugh breast arinma metastases are the mst mmn ause f internal mammary lymphadenpathy, lymphma wuld be mre mmn in this male patient.

The CT image (Fig. 96.1) shws a TTNB predure. The CT image nrms the intralesinal latin f the bipsy needle. Regarding planning a TTNB predure, a prebipsy CT san shuld be btained. The shrtest, mst vertial rute shuld be hsen, and the path f the needle shuld avid interlbar ssures, pulmnary vessels, bullae, and areas f severe emphysema. TTNB is a relatively safe and aurate predure fr btaining bipsy speimens f lung ndules and masses. The sensitivity fr malignant ndules is greater than 90%, and the auray fr differentiating amng varius ell types f lung aner is rughly 80%. A majr limitatin f TTNB using ne-needle aspiratin is a relatively lw sensitivity (10%–40%) fr making a spei benign diagnsis. Hwever, this an be signiantly imprved by using re needle bipsy devies. Suh devies prvide histlgi speimens, whih imprve the auray f diagnsing benign entities suh as granulmas, hamartmas, and rganizing pneumnia. It is imprtant t remember that a bipsy fr negative malignany is nt diagnsti unless a spei benign diagnsis has been established. Indeed, abut 30% f lesins with nnspei negative bipsy results ultimately prve t be malignant. Thus, when faed with nnspei negative bipsy result, ne shuld nsider repeating the bipsy with a re needle bipsy. Alternatively, with n-site ytpathlgy, FNA an be perfrmed thrugh a guide needle, and if the FNA speimen is insufient, re needle bipsy an be frmed at the same time. With the inreasing imprtane f mleular markers fr aner diagnsis and treatment, re needle bipsy bemes mre imprtant fr tissue sampling. Cre needle bipsy is als remmended fr bipsy f lesins with a suspeted diagnsis f lymphma t prvide sufient tissue fr lassiatin f lymphma.

Discussion Enlarged internal mammary lymph ndes are generally nt visible n hest radigraphs until they are nsiderably enlarged. On a psteranterir (PA) hest radigraph f a patient with enlarged internal mammary ndes, a fal parasternal paity, whih is usually seen at the level f the rst three interstal spaes and less mmnly at the furth r fth level, may be present. On a lateral radigraph, a lbulated retrsternal paity may be present, as in this ase. A lbulated retrsternal paity may als be present in patients with dilated internal mammary vessels. Fr example, artatin f the arta is assiated with llateral internal mammary arteries, and superir vena ava bstrutin is assiated with llateral internal mammary veins. The frmer is assiated with a harateristi appearane f the arta and asinal rib nthing, and the latter is usually assiated with a large mass in the right paratraheal regin. REFERENCES

Sharma A, Fidias P, Hayman LA, etal. Patterns f lymphadenpathy in thrai malignanies. Radiographics. 2004;24:419–434. Thrai Radilgy: The Requisites, 3rd ed., pp. 97–136.

CASE 96 CT-Guided Transthoracic Needle Biopsy Procedure 1. B. Emphysema is the greatest risk fatr fr develping pneumthrax fllwing transthrai needle bipsy f the lung. Apprximately 50% f patients with emphysema will develp pneumthrax fllwing bipsy in ntrast t apprximately 20% t 30% f patients withut emphysema. Older age and lwer lbe latin d nt signiantly affet pneumthrax rate. Bipsy f subpleural lesins is less likely t result in pneumthrax beause the needle may nt pass thrugh aerated lung. 2. D. The sensitivity f transthrai needle bipsy fr malignant lesins is greater than 90%. There is n signiant differene between re needle bipsy and ne needle aspiratin in mst aners. Hwever nn-slid ndules have a higher yield with re needle bipsies. 3. B. Up t 30% f patients underging transthrai needle bipsy will develp a pneumthrax. Mst pneumthraes reslve withut treatment. Mild perilesinal hemrrhage

REFERENCES

Cham MD, Lane ME, Henshke CI, Yankelevitz DF. Lung bipsy: speial tehniques. Semin Respir Crit Care Med. 2008;Aug;29(4):335–349. Thrai Radilgy: The Requisites, 3rd ed., pp. 460–472.

CASE 97 Bronchial Atresia 1. A and C. Pulmnary arinids an grw in a brnhial lumen and result in muid impatin and a brnhele (a irumsribed dilatin f a brnhus). Typially, an endbrnhial mass expanding the airway is visible. Brnhial atresia an result in muid impatin and brnhele frmatin. Cnstritive brnhilitis affets the small airways and results in diffuse r hetergeneus lw-attenuatin lung frm air trapping. This is present in the upper lbe in this ase, but nstritive brnhilitis is nt assiated with a brnhele. Lung absesses are typially rund and ntain liquid with

Fair Game

r withut gas. A lung absess wuld nt explain the adjaent hyperlueny and the walls f absesses are usually thik. 2. D. Air an enter lung distal t an bstrutin thrugh the anals f Lambert and pres f Khn. Brnhpleural stula results in air entering the pleural spae frm adjaent, usually inamed, r nerti, lung. An anmalus brnhus usually des nt prvide aeratin t lung distal t a muele. 3. A. The left upper lbe, espeially the apipsterir segment, is the mst mmn site f brnhial atresia. Hwever, brnhial atresia an affet any lbe and rarely an be multisegmental r multilbar. 4. C. Mst patients with brnhial atresia require n treatment. Resetin is rarely indiated. There is n rle fr endbrnhial therapy.

Comment Differential Diagnosis Axial and rnal (Figs. 97.1 and 97.2) mputed tmgraphy (CT) images shw a tubular struture in the left upper lbe ntaining an air-uid level. The part f the lung supplied by this brnhus is hyperluent due t air trapping. This appearane is mst nsistent with brnhial atresia. Endbrnhial neplasms an als ause brnhele frmatin. The differential diagnsis f brnhial atresia (BA) inludes ther auses f muid impatin inluding allergi brnhpulmnary aspergillsis (ABPA), benign and malignant endbrnhial tumrs (arinid, r rarely lung aner), ysti brnhietasis related t previus infetin, and tuberulus brnhstensis.

Discussion BA is a rare and benign nditin haraterized by fal atresia f a segmental, subsegmental, r lbar brnhus. The left upper lbe, espeially the apipsterir segment, is mst mmnly invlved. The airways distal t the pint f atresia are nrmal but the bstrutin t prximal drainage auses aumulatin f muus distal t the pint f atresia, with resultant muid impatin and frmatin f a brnhele. An imprtant assiated feature is hyperinatin f the distal lung parenhyma thrugh llateral air drift and air trapping. BA manifests radilgially as a rund, vid, r branhing struture (brnhele lled with muus) assiated with distal hyperinatin, but it may be verlked n the hest radigraph. Althugh its imaging features are harateristi, BA is mmnly misdiagnsed, ften mistaken fr an arterivenus malfrmatin. Beause mst ases f BA d nt require surgial resetin, regnitin f its harateristi imaging features is essential fr nservative management. REFERENCES

Kinsella D, Sissns G, Williams MP. The radilgial imaging f brnhial atresia. Br J Radiol. 1992;65:681–685. Biyyam D, Chapman T, Fergusn M, etal. Cngenital lung abnrmalities: embrylgi features, prenatal diagnsis, and pstnatal radilgi-pathlgi rrelatin. Radiographics. 2010;30:1721–1738. Thrai Radilgy: The Requisites, 3rd ed., pp. 193–209.

CASE 98 Hypersensitivity Pneumonitis 1. C and D. The differential diagnsis fr grund-glass attenuatin entrilbular ndules is primarily hypersensitivity pneumnitis (HP) and respiratry brnhilitis. HP is muh mre likely given the extent f ndules and the patient’s symptms. Saridsis and silisis are haraterized by the presene f well-dened ndules

205

in a mid and upper lung predminantly. The ndules are usually slid and lated in a perilymphati distributin. 2. B. Bth the pulmnary artery and brnhile are in the entral re f the pulmnary lbule. The pulmnary veins are nt lated in the entral re f the pulmnary lbule but in the interlbular septa. Pulmnary lymphatis are lated primarily in the interlbular septa as well as in the brnhvasular bundles. 3. D. Chrni esinphili pneumnia is nt assiated with igarette smking and ften urs in asthmati patients. Respiratry brnhilitis, pulmnary Langerhans ell histiytsis, and desquamative interstitial pneumnia ur primarily in igarette smkers. 4. D. Hypersensitivity pneumnitis results primarily frm a ell-mediated delayed-type hypersensitivity reatin (type IV). IgE and yttxi antibdies d nt play a signiant rle in the pathgenesis f hypersensitivity pneumnitis. Immune mplexes play a smaller rle in the pathgenesis f HP.

Comment Differential Diagnosis Cmputed tmgraphy (CT) images (Figs. 98.1 and 98.2) shw diffuse entrilbular grund-glass attenuatin ndules with sme areas f nuent grund-glass paity. A few lw attenuatin lbules are present, suggesting a mpnent f air trapping. The leading diagnsti nsideratin fr a CT with this appearane is hypersensitivity pneumnitis. Respiratry brnhilitis shuld be nsidered in a persn wh smkes, althugh the extent f grund-glass ndules is typially muh less. Oasinally, infetin, espeially Pneumocystis jirovecii an have a similar appearane but nuent grund glass paities predminate rather than grund glass ndules.

Discussion The presene f entrilbular grund-glass ndules n mputed tmgraphy (CT) shuld prmpt nsideratin f the diagnsis f nnbrti HP. Affeted patients typially present with dyspnea and hrni ugh, and the linial piture may be nfused with ther frms f diffuse lung disease. Often, the radilgist is the rst member f the health are team t suggest the pssibility f HP, and the linial team an then investigate the patient’s envirnmental and upatinal expsures t seek a ausative antigen. Differential diagnsti nsideratins fr entrilbular grund-glass ndules shuld inlude respiratry brnhilitis, a disease that affets igarette smkers, and, in the prper linial setting, infetin, inluding Pneumocystis jirovecii pneumnia, althugh this manifestatin is less mmn than thers. Centrilbular ndules in hypersensitivity pneumnitis usually are prly dened and may be assiated with pathy areas f grund-glass attenuatin. The ndings tend t predminate in the upper lung znes, and air trapping may be present n expiratry CT imaging. Treatment f subaute hypersensitivity pneumnitis nsists f remving the ffending antigen frm the patient’s envirnment and ften presribing rtisterids. REFERENCES

Hypersensitivity Pneumnitis: A Cmprehensive Review. J Investig Allergl Clin Immunl. 2015;25(4):237–250 Thrai Radilgy: The Requisites, 3rd ed., pp. 355–376.

CASE 99 Solitary Pulmonary Nodule: Lung Cancer Screening 1. A, B, and C. A small nnalied lung ndule uld represent primary lung arinma, a metastasis, r a nnalied

206

SECTION IV

Answers

granulma. Althugh infetin (typially granulmatus, i.e., tuberulsis [TB] r fungal) an present as multiple r a slitary ndules, primary baterial infetin des nt usually present as a well-dened small ndule. 2. C. A 7 mm slid ndule n a baseline sreening san wuld be lassied as Lung-RADS 3 (prbably benign) ategry. Lung-RADS 1 inludes n ndules r denitely benign ndules. Lung-RADS 2 inludes slid ndules at baseline